Sol2022 Merged

Descargar como pdf o txt
Descargar como pdf o txt
Está en la página 1de 114








K>/DW/DdDd/
d/s/^ϮϬϭϴ

LVIII Olimpiada Matemática Espñola



Z>^K/DdDd/^WHK>


Concurso Final Nacional


1 y 2 de abril de 2022
PROBLEMAS Y SOLUCIONES

Problema 1
La estrella de seis puntas de la gura es regular: todos los ángulos interiores de
los triángulos pequeños son iguales. A cada uno de los trece puntos señalados se
le asigna un color: verde o rojo. Demuestra que siempre habrá tres puntos del
mismo color que son vértices de un triángulo equilatero.

Solución: Sin pérdida de generalidad, podemos suponer que el punto central,


0, de la gura está pintado de rojo. Si hubiera dos de los vértices del hexágono
(de vértices 1, 2, 3, 4, 5, 6) consecutivos (el 1 es siguiente del 6) pintados de rojo,
junto con 0, tendríamos un triángulo equilátero con los tres vértices rojos. Si no
hay dos vértices consecutivos del hexágono pintados de rojo, tiene que haber al
menos tres de los seis pintados de verde. Si son tres rojos y tres verdes, puesto
que no puede haber dos rojos consecutivos, los tres rojos y los tres verdes se van
alternando; luego los tres rojos (por ejemplo, 1, 3 y 5) son vértices de un triángulo
equilátero. Por lo tanto, tiene que haber cuatro (o más) de los seis vértices del
hexágono pintados de verde. Teniendo en cuenta que que no puede haber dos rojos
consecutivos, y tampoco puede haber tres verdes en lugares alternos, la única
posibilidad que queda es que haya dos rojos diametralmente opuestos (pongamos
que son el 2 y el 5) y los demás verdes. Finalmente, si 7 es rojo, 2, 5 y 7 forman
triángulo equilátero de vértices rojos. Y si 7 es verde, 1, 6 y 7 forman triángulo
equilátero de vértices verdes.

Problema 2
Sean a, b, c, d cuatro números reales positivos. Si se cumple
1 1 1 1 1 1
a+b+ =c+d+ y + + ab = + + cd
ab cd a b c d
demuestra que al menos dos de los valores a, b, c, d son iguales.

Solución 1: Sea u = a + b + ab1 y v = a1 + 1b + ab. Denotamos r = ab1 . Entonces


tenemos que a + b + r = u, ab + br + ra = v y abr = 1. Por las identidades de
Cardano-Vieta, a, b y r son las tres raíces del polinomio p(x) = x3 − ux2 + vx − 1.
Por la misma razón, c, d y cd1 son estas mismas tres raíces. Como p(x) solo tiene
tres raíces, los valores a, b, c, d no pueden ser todos distintos.

Solución 2: Procedamos por contradicción, suponiendo que existen cuatro


valores diferentes (a, b, c, d) que cumplen las igualdades del enunciado. Podemos
considerar que c y d están jadas y hallar a partir de ahí los valores de (a, b) que
cumplen el sistema. Una rápida inspección nos da 6 soluciones, que supondremos,
por ahora, que son diferentes:
       
1 1 1 1
(c, d) , c, , (d, c) , d, , ,c , ,d .
cd cd cd cd

En todos estos casos hay al menos dos variables iguales entre las cuatro. Si pro-
bamos que estas son las únicas soluciones del sistema habremos concluido.
Consideremos las variables auxiliares
1 1
x = a + b, y = + .
a b
y denotemos
1 1
z = c + d, t =
+ .
c d
Es fácil comprobar que cada valor de la pareja (x, y) da lugar, a lo sumo, a dos
parejas de soluciones de la forma (a, b) y (b, a). El sistema inicial se reescribe
como
y t x z
x+ =z+ , y+ =t+ .
x z y t
De la primera ecuación tenemos que
 t 
y =x z+ −x ,
z
y por tanto encontrar el valor de x determina unívocamente el de y . Es suciente
ver entonces que hay a lo sumo tres opciones para el valor de x, dado que eso
dejaría a lo sumo tres opciones para el par (x, y) y seis para (a, b), que ya las
conocemos. Sustituyendo en la segunda ecuación y eliminando denominadores,
tenemos que
 t 2  z  t 
x z+ −x − t+ z + − x + 1 = 0.
z t z
Como es una ecuación de tercer grado, hay a lo sumo tres soluciones y hemos
concluido.
El único caso que queda por analizar es el derivado de que alguna de las seis
soluciones iniciales coincidan, y es inmediato comprobar que eso es equivalente a
c2 d = 1 o cd2 = 1. Por simetría, es suciente considerar el primer caso, en el que
d = c12 , con c 6= 1. En ese caso, se pueden calcular explícitamente las soluciones
de la ecuación en x, que son x = c + c12 (doble) y x = 2c. Se comprueba que
la solución doble da lugar a las parejas (c, c12 ) y ( c12 , c), mientras que la solución
x = 2c da el par (c, c). En ambos casos se tiene la conclusión deseada.

Problema 3
Sea ABC un triángulo, con AB < AC , y sea Γ su circuncírculo. Sean D, E y
F los puntos de tangencia del incírculo con BC , CA y AB , respectivamente. Sea
R el punto de EF tal que DR es una altura del triángulo DEF y sea S el punto
de corte de la bisectriz exterior del ángulo ∠BAC con Γ. Probar que AR y SD
se cortan sobre Γ.

Solución 1: Sea X el segundo punto de corte de AR con Γ. Por comodidad,


pongamos β = ∠ABC y γ = ∠BCA. Se tiene entonces que
BX sin ∠BAX FR F D sin(γ/2) BD sin β sin(γ/2) cos(γ/2) BD
= = = = = ,
XC sin ∠CAX RE DE sin(β/2) DC sin(β/2) cos(β/2) sin γ DC
donde en la primera igualdad se ha usado el teorema del seno en los triángulos
ABX y ACX ; en la segunda que AF E es isósceles; en la tercera el teorema del
seno en F DR y DRE ; y en la cuarta nuevamente el teorema del seno en BDF y
CDE .
Sea ahora Y el segundo punto de corte de SD con Γ. Tenemos que Y D es una
bisectriz del ángulo ∠BY C y por tanto podemos aplicar el teorema de la bisectriz
y deducir que
BY BD
= .
YC DC
Por tanto, tenemos que X e Y son dos puntos sobre el arco de BC que no
contiene a A y que satisfacen BX
XC
= BY
YC
, lo que implica necesariamente que
X = Y (dado que X e Y están sobre el círculo de Apolonio de B y C , que corta
en un único punto al arco BC que no contiene a A). Esto demuestra lo que pedía
el enunciado.

Solución 2: Sea T el punto diametralmente opuesto a S en Γ. Vamos a de-


mostrar que el ángulo formado por las rectas AR y SD es igual a ∠AT S , y por
lo tanto la intersección de dichas rectas se halla sobre Γ.
S

A A

R E R
F F
I

B D C B D C

Comenzamos observando que los triángulos AF E y SBC son semejantes, ya


que son isósceles y ∠F AE = ∠BAC = ∠BSC . Además, como la altura del lado
desigual es AT (la bisectriz interna) en un caso y ST en el otro, la rotación
necesaria para orientar igualmente ambos triángulos es igual a ∠AT S . Por lo
tanto, basta con demostrar que los puntos R y D son puntos equivalentes en la
semejanza, es decir, RE
FR
= BD
DC
.
Sea I el incentro de ABC . Claramente ∠IBD = ∠B/2 y ∠ICD = ∠C/2.
Dado que ∠AF E = 90◦ − ∠A/2 (por estar en el triángulo isósceles AF E ) y
∠BF D = 90◦ −∠B/2, obtenemos ∠DF R = 90◦ −∠C/2 y ∠DER = 90◦ −∠B/2.
Concluimos que
FR DR sec(90◦ − ∠C/2) ID sec(∠B/2) BD
= ◦
= = ,
RE DR sec(90 − ∠B/2) ID sec(∠C/2) DC

tal y como queríamos demostrar.

Problema 4
Sea P un punto en el plano. Demuestra que es posible trazar tres semirrectas
con origen en P con la siguiente propiedad: para toda circunferencia de radio r
que contiene a P en su interior, si P1 , P2 y P3 son los puntos de corte de las
semirrectas con la circunferencia, entonces

|P P1 | + |P P2 | + |P P3 | ≤ 3r.

Solución 1: Trazamos tres semirrectas con origen en P de manera que el


ángulo formado por dos cualesquiera de las tres sea de 120◦ . Imaginemos el círculo
dividido en seis regiones mediante tres rectas que pasen por su centro y sean
paralelas a las tres semirrectas. Observemos que, por simetría respecto a una de
las rectas o por giros de ángulo 120◦ , basta con analizar lo que ocurre cuando el
punto P se encuentra en una cualquiera de las seis regiones. Usamos las notaciones
de la gura.
Sin pérdida de generalidad, podemos suponer que el radio de la circunferen-
cia es r = 1. Los tres puntos de corte de las semirrectas con la circunferencia
son, respectivamente, P1 , P2 y P3 . Las paralelas con origen en O (centro de la
circunferencia) la cortan, respectivamente, en A, B y C . La recta P P3 y la tan-
gente a la circunferencia en C se cortan en el punto T . S 0 es el punto de corte
de P P3 con la paralela a OA por C . Es obvio que ∠S 0 CT = 30◦ , con lo que
SP3 = SS 0 + S 0 P3 = OC + S 0 P3 ≤ OC + S 0 T = OC + 21 CS 0 = 1 + 12 OS . De mane-
ra análoga, se demuestra que RP2 ≤ 1 + 12 OR. También es obvio que RP = P S .
Además, como 12 OS = 12 OR + 21 RS y P Z = OR + 12 RS , tenemos:

P P1 + P P2 + P P3 = P P1 + RP2 + P R + SP3 − SP =
1 1
P P1 + RP2 + SP3 ≤ P P1 + 1 + OR + 1 + OS =
2 2
1
2 + P P1 + OR + RS ≤ 2 + P P1 + P Z ≤ 3.
2

Solución 2: Comenzamos con la siguiente lema:

Lema En un triángulo equilátero ABC , la suma de las distancias de cualquier


punto interior P a los tres lados es independiente de P . Si Q no se encuentra en
el interior de ABC , entonces la suma de distancias de Q a los tres lados es mayor
o igual a aquella de P .
Demostración: Sean X , Y y Z las proyecciones de P en AB , BC y CA,
respectivamente. Entonces tenemos que
2 · área ABP 2 · área BCP 2 · área CAP 2 · área ABC
PX +PY +PZ = + + = ,
AB BC CA AB
que no depende de P . Si Q no está en el interior de ABC usamos el mismo cálculo
con área ABQ + área BCQ + área CAQ ≥ área ABC .

P1
O1
P

P3
O

P2
O3
O2

Trazamos tres semirrectas desde P que formen ángulos de 120◦ . Ahora sea `1
la recta perpendicular a P P1 que pasa por P1 , y denimos `2 y `3 análogamente.
Estas tres rectas forman ángulos de 60◦ entre sí, por lo que forman un triángulo
equilátero.
Sea O el centro de la circunferencia. Sea O1 la proyección de O sobre `1 ,
y denimos análogamente O2 y O3 . Como P está en el interior del triángulo
equilátero, tenemos que
P P1 + P P2 + P P3 ≤ OO1 + OO2 + OO3 ≤ OP1 + OP2 + OP3 = 3r.

Solución 3: Consideramos tres semirrectas arbitrarias que pasen por P y


que formen entre sí un ángulo de 120◦ . Tomemos ahora una circunferencia que
contenga en su interior al punto P , y jemos unos ejes coordenados de manera
que la circunferencia tiene radio 1 y una de las semirrectas tiene la dirección del
eje de abscisas.
De esta manera P = (a, b), con a2 + b2 < 1. Para hallar las coordenadas de P1 ,
consideramos el valor de λ1 > 0 tal que (a, b) + λ1 (1, 0) tiene módulo 1. Se tiene
además que |P P1 | = λ1 , y es inmediato comprobar que

4 − 4b2
λ1 = −a + .
2
De forma similar, para hallar las coordenadas de P2 , consideramos el valor
de λ2 > 0 tal que (a, b) + λ2 (cos 120◦ , sin 120◦ ) tiene módulo 1. Resolviendo la
ecuación cuadrática se comprueba que
√ p √
a − b 3 + 4 − 3a2 − b2 − 2 3ab
λ2 = .
2
De la misma manera, |P P2 | = λ2 . Análogamente,
√ p √
a + b 3 + 4 − 3a2 − b2 + 2 3ab
λ3 = .
2
La desigualdad del enunciado equivale entonces a
√ √ √
q q
4 − 4b2 + 4 − 3a − b − 2 3ab + 4 − 3a2 − b2 + 2 3ab ≤ 6.
2 2

Aplicando la desigualdad entre las medias aritmética y cuadrática, obtenemos


que
√ √ √
q q
4 − 4b2 + 4 − 3a2 − b2 − 2 3ab + 4 − 3a2 − b2 + 2 3ab
√ p
≤ 3 · 12 − 6(a2 + b2 ) ≤ 6,
como queríamos ver.

Problema 5
En un grupo de 2022 estudiantes, algunos son amigos entre sí, y la amistad es
siempre recíproca. Sabemos que cualquier subconjunto de esos estudiantes tiene
la siguiente propiedad: siempre existe un estudiante del subconjunto que es amigo
de, a lo sumo, 100 estudiantes del mismo.
(a) Determina el menor entero positivo N que nos asegura que se cumple la
siguiente propiedad: es posible dividir a los estudiantes en N grupos (no
necesariamente del mismo tamaño), de manera que dos estudiantes que
están en el mismo grupo nunca son amigos entre sí.
(b) Numeramos a los estudiantes del 1 al 2022. Sea ci el número de amigos del
estudiante i. Determina el máximo valor que puede tomar la suma

c1 + c2 + . . . + c2022 .

Solución: Para la primera parte, seguimos la siguiente estrategia: tomamos un


estudiante con a lo sumo 100 amigos (que existe por hipótesis). Diremos que es el
estudiante 1. Sacamos a ese estudiante y en el subconjunto resultante, existirá un
estudiante con a lo sumo 100 amigos en dicho subconjunto; este será el estudiante
2, y así sucesivamente.
Vamos a probar entonces que la respuesta es N = 101. Empezamos a asignar
grupo por el nal. Al estudiante 2022 le asignamos un grupo cualquiera. En
general, al estudiante i le asignaremos un grupo que no haya sido usado en ninguno
de los estudiantes ya asignados de los que sea amigo. Por construcción, sabemos
que es amigo de como máximo 100 estudiantes con un número mayor, así que con
tener 101 grupos es suciente para completar esta asignación.
Para ver que N = 101 es la mejor opción, consideremos la siguiente congura-
ción: tomamos 100 estudiantes de los 2022 y hacemos que esos 100 sean amigos
de todos (tanto entre ellos como con los 1922 restantes); entre los demás no hay
más relaciones de amistad. Si ahora tomamos esos 100 estudiantes y uno de los
otros tenemos que hay todas las relaciones de amistad posibles, con lo que 100
grupos, o cualquier cantidad inferior, no sería suciente.
Para el segundo apartado seguimos esta misma estrategia. Tomamos un estu-
diante con a lo sumo 100 amigos y lo eliminamos. Cada vez que hacemos este
proceso, si quedaban por lo menos 101, estamos eliminando a lo sumo 100 rela-
ciones de amistad. Cuando quedan i estudiantes, con i ≤ 100, estamos sacando
como máximo i − 1 relaciones de amistad. En total, no sacamos más de
99  
X 100
100 · 1922 + i = 192200 +
i=1
2

relaciones de amistad. Teniendo en cuenta que, por el enunciado del apartado (b),
cada relación de amistad ha de contarse dos veces, la suma pedida será menor
o igual que 384400 + 9900 = 394300. Está claro que podemos conseguir la cota.
Para ello, al igual que antes, tomamos 100 de los 2022 y hacemos que esos 100
sean amigos de todos (tanto entre ellos como con los 1922 restantes). En este
caso, el número de amistades es el proporcionado por la cota.

Problema 6
Halla todas las ternas de enteros positivos (x, y, z) , con z > 1, que satisfacen
simultáneamente que
x divide a y + 1, y divide a z − 1, z divide a x2 + 1.

Solución 1: Las soluciones son (1, 1, 2), (2, 1, 5) y (2n + 1, 2n, 2n2 + 2n + 1)
con n ≥ 1.
Si x = 1 la única solución es (1, 1, 2) (z divide a 2, por lo que z = 2, e y divide
a 1). Si x = 2 la única solución es (2, 1, 5) (z divide a 5, así que z = 5, e y divide
a 4 y es impar). Supongamos ahora que x ≥ 3.
Sean y+1 = rx, z −1 = sy y x2 +1 = tz . Entonces sustituyendo sucesivamente,
obtenemos que
x2 + 1 = tz = sty + t = rstx − st + t. (1)
De esta igualdad se tiene que st − t ≡ −1 (mod x). Esto implica que st − t ≥
x − 1, y a su vez st ≥ x. Si r ≥ 2, entonces

x2 + 1 = rstx − st + t ≥ (2x − 1)st + 1 ≥ (2x − 1)x + 1;

que, reordenado, deja x(x − 1) ≤ 0, lo que no es cierto para x ≥ 3. Por lo tanto,


concluimos que r = 1 en las soluciones restantes. Tenemos entonces
x2 + 1 = st(x − 1) + t (2)
Si tomamos módulo x − 1 a ambos lados, tenemos t ≡ 2 (mod x − 1). Si t = 2,
entonces (2) es equivalente a s = x+1
2
. Entonces x es un número impar (de la
forma 2n + 1), y = rx − 1 = 2n, y z = sy + 1 = 2n2 + 2n + 1. Si t 6= 2, entonces
t ≥ x + 1. De (2) obtenemos x2 + 1 ≥ (x + 1)(x − 1) + x + 1, o x ≤ 1, con
lo que llegamos a una contradicción. No hay más soluciones. Comprobamos que
todas las posibles soluciones que hemos obtenido satisfacen las condiciones del
enunciado.

Solución 2: Denotemos por a, b, c los tres cocientes, de manera que y = ax−1,


z = abx − b + 1 y
x2 − abcx + bc − c + 1 = 0.
Se trata de una ecuación cuadrática en x, que ha de tener por discriminante un
cuadrado perfecto, esto es,
(abc)2 − 4(bc − c + 1) = d2 ,

con d ≥ 0 un número entero. Observemos que c(b − 1) + 1 > 0, de manera que


d < abc, o alternativamente d ≤ abc − 1. Esta condición nos dice que

4(bc − c + 1) = (abc)2 − d2 ≥ (abc)2 − (abc − 1)2 = 2abc − 1,

y la desigualdad tiene que ser estricta dado que el lado izquierdo siempre es par
y el lado derecho siempre es impar. De aquí se deduce lo siguiente:
Si c > 1, en particular sucede que el lado derecho es positivo, por lo que
necesariamente tiene que pasar que a = 1.
Si c = 1 también puede pasar que a = 2 (pero nunca a > 2).
Vamos a analizar cada caso por separado.
(a) Si c > 1 y a = 1, entonces
d2 = (bc)2 − 4bc + 4c − 4 = (bc − 2)2 + 4(c − 2) ≥ (bc − 2)2 ,

con igualdad si y solo si c = 2. En este caso, x = y+1, 2z = x2 +1, con lo que


x es par. La segunda ecuación señala que x − 1 divide a (x2 − 1)/2, lo cual
siempre es cierto. Se obtiene pues como solución (2n + 1, 2n, 2n2 + 2n + 1),
con n ≥ 1. En caso contrario, si c 6= 2, como bc−2 < d < bc, necesariamente
pasa que d = bc − 1. Ahora bien, operando se llega a 4(c − 2) = 2bc − 3, lo
cual no es posible por razones de paridad.
(b) Si c = 1 y a = 2, entonces z = x2 + 1, y + 1 = 2x. Luego 2x − 1 divide a x2 .
Pero si 2x − 1 divide a x2 , entonces también divide a 2x2 − x(2x − 1) = x.
A su vez, esto implica que 2x − 1 también divide a 2x − (2x − 1) = 1. Por
tanto, x = 1, y = 1, z = 2.
(c) Si c = 1 y a = 1, entonces z = x2 + 1, y + 1 = x. Luego x − 1 divide a x2 .
Pero si x − 1 divide a x2 , entonces también divide a x2 − x(x − 1) = x. A
su vez, esto implica que x − 1 también divide a x − (x − 1) = 1. Por tanto,
x = 2, y = 1, z = 5.
En denitiva, las únicas soluciones son:
(1, 1, 2), (2, 1, 5) y (2n + 1, 2n, 2n2 + 2n + 1), con n ≥ 1.
OLIMPIADA MATEMÁTICA
PROBLEMAS Y SOLUCIONES 2021

REAL SOCIEDAD MATEMÁTICA ESPAÑOLA





73 8 3 9 4 0 4 14 2 4 3

444 35363
 334

3 23
45 6 47


5 2 6 2 7 2 8 2 9 3 0 3 13


4 84 9 5 0 5 1 5 2 5 3 5 4 5 5 5 6 5


23 2 4 2



23 4 92 0

2 12 2
5 678 9
1 0 1 1 1 2 13 1 4 1 5 1 6 1 7 1 8 1


71



K>/DW/DdDd/
d/s/^ϮϬϭϴ LVII Olimpiada Matemática Española

Concurso Final Nacional
Z>^K/DdDd/^WHK>

7 y 8 de mayo de 2021
PROBLEMAS Y SOLUCIONES

Problema 1
Los vértices, A, B y C, de un triángulo equilátero de lado 1 están en la superfi-
cie de una esfera de radio 1 y centro O. Sea D la proyección ortogonal de A sobre
el plano, α, determinado por B, C y O. Llamamos N a uno de los cortes con la
esfera de la recta perpendicular a α por O. Halla la medida del ángulo DN \ O.
(Nota: la proyección ortogonal de A sobre el plano α es el punto de corte con
α de la recta que pasa por A y es perpendicular a α.)

Solución 1:
Es obvio que A, B, C y O son vértices de un tetraedro regular de arista igual
a 1, puesto que la distancia entre dos cualesquiera de ellos es 1. Como D es la
proyección ortogonal de A sobre la cara opuesta del tetraedro, D es el centro de la
cara BCO. Así pues, la distancia de D a O (distancia del centro de un triángulo
equilátero de lado 1 a uno de sus vértices) es

2 3 1
d(D, O) = =√ .
3 2 3

Como el triángulo DN O es rectángulo en O, el cateto OD mide 1/ 3 y el
cateto ON mide 1, el ángulo buscado es arc tg √13 = 30o .

Solución 2:
Elegimos un sistema de coordenadas (x, y, z) de modo que sea O ≡ (0, 0, 0), α
el plano z = 0 y la recta y = z = 0 (es decir, el eje x) que sea la mediatriz de
BC por O, estando  √BC en el  √ x > 0, z = 0. Tenemos así que, al ser
 semiplano
3 −1
BC = 1, son B ≡ 2 , 2 , 0 y C ≡ 23 , 21 , 0 . Por simetría respecto del plano
y = 0, tenemos que A ≡ (u, 0, v) con u2 + v 2 = 1 para que A esté en la esfera, y
 √ 2
u − 23 + 14 + v 2 = 1 para que AB = AC = 1. Además, por ser D la proyección
de A sobre α se tiene que D ≡ (u, 0, 0) y OD = |u|. En consecuencia,
√ !2
3 1 √ 3 1 1
u2 = 1 − v 2 = u − + = u2 − 3u + + , OD = u = √ .
2 4 4 4 3
Luego ODN es un triángulo rectángulo en O, con ON = 1 y OD = √1 ,
3
concluyéndose que el ángulo buscado es arc tg √13 = 30o .
Problema 2
Dado un número entero positivo n, definimos λ(n) como el número de solu-
ciones enteras positivas de la ecuación x2 − y 2 = n. Diremos que el número n
es “olímpico” si λ(n) = 2021. ¿Cuál es el menor entero positivo que es olímpico?
¿Y cuál es el menor entero positivo impar que es olímpico?

Solución:
Distinguiremos 4 casos, según n sea impar o par y según n sea cuadrado per-
fecto o no.
(a) Sea n = pa11 · · · par r un número impar que no es cuadrado perfecto.
Si x2 − y 2 = (x + y)(x − y) = n, con x, y > 0, entonces existen enteros positivos
a, b, con a > b y teniendo ambos la misma paridad, de manera que x + y = a y
x − y = b (con lo que, x = (a + b)/2, y = (a − b)/2). Las formas de escribir n
como un producto de dos números diferentes de la misma paridad son, en este
caso, la mitad del número de divisores, por lo que tendremos que buscar números
con 4042 divisores:
4042 = (a1 + 1) · · · (ar + 1).
La descomposición de 4042 como producto de primos es 4042 = 2·43·47. Por tanto,
las opciones de números naturales con 4042 divisores son de la forma: pq 42 r46 ;
pq 2020 ; p42 q 93 ; p46 q 85 ; o p4041 , en donde p, q, r son primos impares diferentes. Es
inmediato comprobar que la opción que da el número más bajo es 346 · 542 · 7.
(b) Consideremos ahora el caso en el que n es impar y cuadrado perfecto. En
este caso, el número de divisores es impar, y como excluimos el caso de que los dos
números de la factorización sean iguales, tenemos que buscar cuadrados perfectos
con 4043 divisores:
4043 = (a1 + 1) · · · (ar + 1).
La descomposición de 4043 como producto de primos es 4043 = 13 · 311, luego
las únicas opciones son p12 q 310 y p4042 . El número más bajo es 3310 · 512 , que es
mayor que el que hemos encontrado antes.
(c) Supongamos ahora que n = 2k pa11 · · · par r es par, pero no un cuadrado per-
fecto. Nuevamente, tenemos que hacer la descomposición n = ab, siendo a y b de
la misma paridad; esto hace que a y b tengan que ser pares, o sea, que k ≥ 2 y
no sirven los divisores impares. Por tanto, las opciones para el exponente de 2 en
cada divisor son 1, 2, 3, . . . , k − 1, pero nunca 0 ni k. Así pues,
4042 = (k − 1)(a1 + 1) · · · (ar + 1),
y las opciones son 23p42q46; 244pq46; 248pq42; 287p46; 248p85; 295p42; 244q93; 23p2020;
22022p; y 24043 .También es válida la opción 4m, donde m es cualquiera de los
números considerados en el apartado (a). El número más bajo es 248 · 342 · 5, que
es menor que el encontrado en el apartado (a).
(d) Por último, tenemos el caso en el que n es par y cuadrado perfecto, esto es
4043 = (k − 1)(a1 + 1) · · · (ar + 1) y las opciones son 214p310, 2312p12 y
24044.También es válida la opción 4m, donde m es cualquiera de los números
considerados en el apartado (b). Los tres números son mayores que el encontrado
en el apartado (c).
Por consiguiente, el menor número olímpico es 248 · 342 · 5 y el menor número
impar olímpico es 346 · 542 · 7.
Problema 3
Tenemos 2021 colores y 2021 fichas de cada color. Colocamos las 20212 fichas
en fila. Se dice que una ficha, F , es “mala” si a cada lado de F quedan un número
impar de las 2020 × 2021 fichas que no comparten color con F .

(a) Determina cuál es el mínimo número posible de fichas malas.

(b) Si se impone la condición de que cada ficha ha de compartir color con al


menos una ficha adyacente, ¿cuál es el mínimo número posible de fichas
malas?

Solución 1:
(a) Como 2020 × 2021 es par, para decidir si una ficha es mala es suficiente
comprobar que el número de fichas a su izquierda con las que no comparte color
es impar. Sea A el conjunto de fichas que ocupan una posición impar en la fila, es
decir, las fichas que tienen un número par de fichas a su izquierda. Es claro que
2
|A| = 20212 +1 .
Sea B el conjunto de fichas que, entre las de su color, ocupan una posición
impar. Es decir, hay un número par de fichas de su color a su izquierda. Hay 1011
fichas de cada color en B, así que |B| = 1011 × 2021.
Por construcción, las fichas que están en B pero no en A son malas (tienen un
número impar de fichas a su izquierda, de las cuales un número par comparten
2
color con ella). Por lo tanto, hay al menos |B|−|A| = 2021×1011− 20212 +1 = 1010
fichas malas.
Este número se puede conseguir de la siguiente forma: colocamos 2020 fichas
de color 1, luego 2020 fichas de color 2, etc., hasta tener 2020 fichas de cada color;
luego colocamos las fichas restantes por orden de color. Las únicas fichas malas
en esta configuración son las que son última de cada color par. En la imagen se
ve esta configuración para 5 colores y 5 fichas de cada color.

malas

(b) Sean X = A ∩ B, Y = A ∩ B c y Z = Ac ∩ B, donde A y B son los


conjuntos descritos en el apartado anterior. Las fichas malas son precisamente
Y ∪ Z (contando las fichas a la izquierda que no comparten color con la ficha
dada, como antes), por lo que tenemos que minimizar el tamaño de este conjunto.
Obsérvese que si la ficha F es adyacente a una ficha F 0 ∈ Z del mismo color,
entonces F ∈ Y . Como cada ficha de Z es adyacente a al menos una ficha de
Y , y cada ficha de Y es adyacente a como mucho dos fichas de Z, tenemos que
|Z| ≤ 2|Y |.
Además, tenemos que |Z| − |Y | = (|X| + |Z|) − (|X| + |Y |) = |B| − |A| = 1010.
Y concluimos que

|Y | + |Z| = 3(|Z| − |Y |) − 2(|Z| − 2|Y |) ≥ 3 × 1010 − 2 × 0 = 3030.

(La conclusión del último párrafo también se puede obtener haciendo:


|Z| = 1010 + |Y | ≥ 1010 + 21 |Z|, luego |Z| ≥ 2020 y |Z| + |Y | ≥ 23 |Z| ≥ 3030.)
Una forma de tener exactamente 3030 fichas malas es colocar primero 2018
fichas de color 1, luego 2018 de color 2, etc., y después colocar las tres fichas
restantes de color 1, luego las tres de color 2, etc. Las únicas fichas malas son las
tres últimas de cada color par.

Solución 2: Diremos que una ficha que no es mala, es “buena”. Numeramos los
colores de 1 a 2021. Decimos que una ficha es par (impar) si ocupa una posición
par (impar) en la fila, y decimos que es color-par (color-impar) si de entre las
fichas de su color, ocupa un lugar par (impar). Si una ficha es par y color-par,
eso quiere decir que tiene antes de ella en la fila un número impar de fichas de su
color, y también un número impar de fichas en total. Luego tiene antes que ella
un número par de fichas de otros colores, y al ser 2020 · 2021 par el número total
de fichas de otros colores, también tiene un número par de fichas de otros colores
detrás de ella. Luego una ficha par y color-par es buena. De forma análoga, una
ficha impar y color-impar es buena, pero son malas todas las fichas pares que
son color-impares, y todas las fichas impares que son color-pares. Nótese que el
número de fichas color-pares, pares, impares y color-impares son respectivamente
20212 − 1 20212 + 1
2021 · 1010 < < < 2021 · 1011.
2 2
Supongamos que hay exactamente u fichas que sean malas por ser impares pero
2
color-pares. Esto quiere decir que hay exactamente 20212 +1 − u fichas que son
buenas por ser impares y color-impares, luego el número de fichas que son malas
por ser pares y color-impares es exactamente
20212 + 1
 
2021 · 1010 − − u = 1010 + u.
2
Las restantes fichas serán pares y color-pares y por lo tanto buenas, y el número
total de fichas malas será igual a 1010 + 2u.
(a) Podemos hacer u = 0, y por lo tanto que haya exactamente 1010 fichas
malas, todas ellas pares y color-impares, si colocamos primero 2020 fichas del
color 1, luego 2020 fichas del color 2, y así sucesivamente hasta el color 2021,
y a continuación las 2021 fichas restantes, una de cada color. Nótese que las
primeras 2020 · 2021 fichas, todas son bien pares y color-pares, bien impares y
color-impares, luego buenas en cualquier caso. Las 2021 fichas últimas son cada
una de ellas color-impares, pero de ellas 1011 son impares y por lo tanto buenas,
y 1010 son pares y por lo tanto malas. El mínimo, alcanzable de esta forma, es
entonces igual a 1010.
(b) Nótese que cada ficha mala por ser par y color-impar tiene ahora a su lado
una ficha del mismo color, que por lo tanto será impar y color-par, luego u > 0.
Asignemos cada ficha par y color-impar a la ficha contigua de su mismo color
que es impar y color-par. Si una ficha par y color-impar es contigua a dos de su
mismo color, la asignamos a una cualquiera de las dos. Nótese que como cada
ficha impar y color-par tiene a lo sumo dos fichas vecinas, hay a lo sumo 2u fichas
que pueden ser asignadas a u fichas impares y color-pares. Pero sabemos que
hay exactamente 1010 + u fichas asignadas, luego 1010 + u ≤ 2u, y por lo tanto
u ≥ 1010. Concluimos que hay como mínimo 3030 fichas malas, alcanzándose ese
mínimo si la asignación es completa, es decir si cada ficha mala por ser impar y
color-par está rodeada de dos fichas malas de su mismo color, y cada ficha mala
por ser par y color-impar es contigua sólo a una ficha mala de su mismo color.
Esto puede conseguirse por ejemplo colocando primero 2018 fichas del color 1,
luego 2018 fichas del color 2, y así hasta el color 2021, colocando después las
restantes 3 · 2021 fichas, 3 de cada color, en 2021 ternas formada cada una por las
3 fichas restantes de cada color. Las primeras 2018 · 2021 fichas son claramente
buenas, y de las 2021 ternas al final de la fila, 1011 ternas tienen su primera ficha
impar y color-impar, y por lo tanto las 3 fichas de la terna son buenas, y 1010
ternas tienen su primera ficha par y color-impar, y por lo tanto las 3 fichas de la
terna son malas. El mínimo, alcanzable de esta forma, es entonces igual a 3030.
Problema 4
Sean a, b, c, d números reales tales que

a+b+c+d=0 y a2 + b2 + c2 + d2 = 12.

Halla el valor mínimo y el valor máximo que puede tomar el producto abcd, y
determina para qué valores de a, b, c, d se consiguen ese mínimo y ese máximo.

Solución 1:
De las condiciones del enunciado tenemos que no todos los números tienen el
mismo signo. El producto abcd tomará un valor positivo cuando dos números sean
positivos y dos negativos, así que buscaremos el máximo suponiendo que a, b > 0
y c, d < 0. Notemos que

2(ab + cd) ≤ a2 + b2 + c2 + d2 = 12, (1)

con lo que ab + cd ≤ 6. Utilizando la desigualdad entre las medias aritmética y


geométrica,
 2
ab + cd
(ab) · (cd) ≤ ≤ 9. (2)
2

La igualdad en (2) se alcanza cuando ab = cd = 3, con lo que (1) obliga


√a
2 2
que sea (a√− b) = (c − d) = 0; es decir, a = b y c = d. Así pues, a = b = 3,
c = d = − 3. El valor máximo de la expresión es por tanto 9.
Para hallar el valor mínimo supondremos que a, b, c > 0 y que d < 0. (Si tres de
los números son negativos y el otro positivo, considerando sus opuestos tenemos
que el valor de abcd permanece invariante.) Por tanto, d = −(a + b + c) y

a2 + b2 + c2 + d2 = 2(a2 + b2 + c2 + ab + bc + ca) = 12.

Esto quiere decir que

a2 + b2 + c2 + ab + bc + ca
(a+b+c)2 = a2 +b2 +c2 +ab+bc+ca+ab+bc+ca ≤ 6+ ≤ 9,
2
donde se ha usado que, por la desigualdad de Cauchy, ab + bc + ca ≤ a2 + b2 + c2 .
Por tanto, a + b + c ≤ 3. El problema es equivalente a encontrar el máximo valor
posible de abc(a + b + c). Usando nuevamente la desigualdad entre las medias
aritmética y geométrica,

(a + b + c)3 (a + b + c)4
abc ≤ y por tanto abc(a + b + c) ≤ ≤ 3.
27 27
En consecuencia, el valor mínimo es −3 y se alcanza en los casos (3, −1, −1, −1)
y (1, 1, 1, −3) (o permutaciones de estos).
Solución 2:
Aplicando la desigualdad de las medias cuadrática y geométrica a |a|, |b|, |c|, |d|,
tenemos que
r !4
a2 + b 2 + c 2 + d 2 √ 4
|a| · |b| · |c| · |d| ≤ = 3 = 9,
4

con igualdad si y sólo |a| = |b| = |c| = |d| = 3. Con la condición a + b + c + d =
0, esto sólo es posible si exactamente dos de entre a, b, c, d son positivos y los
otros dos negativos. Como esta cota máxima es independiente de la condición
a + b + c + d = 0, tenemos entonces que el máximo √ valor posible de abcd√es 9, y
se obtiene si y sólo si dos de entre a, b, c, d son 3 y los otros dos son − 3.
Tomando a = 3, b = c = d = −1, se obtiene abcd = −3, con lo que el valor
mínimo del producto ha de ser negativo, y se debe conseguir claramente cuando
un número impar de los a, b, c, d es negativo, y los demás son positivos. Como
invertir simultáneamente los signos de a, b, c, d deja inalteradas las condiciones
y el producto, podemos asumir sin pérdida de generalidad que a > 0 > b, c, d
para el mínimo de abcd. Ahora bien, por la desigualdad entre medias aritmética
y geométrica aplicadas a |b|, |c|, |d| tenemos que
3 3
a3
 
|b| + |c| + |d| −b − c − d
|bcd| = |b| · |c| · |d| ≤ = = ,
3 3 27

con igualdad si y sólo si b = c = d, mientras que por la desigualdad entre medias


aritmética y cuadrática,
r r
b2 + c 2 + d 2 12 − a2
a = |b| + |c| + |d| ≤ 3 =3 , a2 ≤ 36 − 3a2 ,
3 3
por lo que a ≤ 3, con igualdad si y sólo si b = c = d = −1. Luego cuando
a > 0 > b, c, d, tenemos que

a4
abcd = −a|bcd| ≥ − ≥ −3,
27
y el mínimo valor posible de abcd es −3 y, restaurando la generalidad, se obtiene
si y sólo si (a, b, c, d) es una permutación de (±3, ∓1, ∓1, ∓1).
Problema 5
Disponemos de 2n bombillas colocadas en dos filas (A y B) y numeradas de 1
a n en cada fila. Algunas (o ninguna) de las bombillas están encendidas y el resto
apagadas; decimos que eso es un “estado”. Dos estados son distintos si hay una
bombilla que está encendida en uno de ellos y apagada en el otro. Diremos que
un estado es “bueno” si hay la misma cantidad de bombillas encendidas en la fila
A que en la B.
Demuestra que el número total de estados buenos, EB, dividido por el número
total de estados, ET, es

EB 3 · 5 · 7 · · · (2n − 1)
= .
ET 2n n!

Solución 1:
Es obvio que ET = 22n , puesto que cada una de las 2n bombillas puede estar
apagada o encendida.
 2 El número  de “estados buenos” con k bombillas encendidas
n n
en cada fila es , ya que hay formas de elegir las k bombillas encendidas
k k
de la fila A y otras tantas de elegir las k bombillas encendidas de la fila B. En
consecuencia,
n  2
X n
EB = .
k
k=0
 
2n
Es conocido que esa suma da como resultado EB = . En cualquier caso,
n
basta con observar que un estado es “bueno” si hay exactamente k (0 ≤ k ≤ n)
bombillas encendidas en la fila A y exactamente n−k bombillas apagadas en la fila
B. Cada “estado bueno” se obtiene (de una única manera) eligiendo n bombillas
en total y haciendo que estén  encendidas
 las elegidas de la fila A y las no elegidas
2n
de la fila B. Así pues, EB = .
n
Ahora, tenemos:
 
2n Qn  Qn 
EB n (2n)! k=1 2k k=1 (2k − 1)
= 2n = 2n = =
ET 2 2 n! n! 22n n! n!
Qn 
2n n! k=1 (2k − 1) 3 · 5 · 7 · · · (2n − 1)
= 2n
= .
2 n! n! 2n n!
Solución 2:
Si empezamos “desde el final”, notemos que

(2n)!
= (2n) · (2n − 2) · · · 4 · 2 = 2n · n · (n − 1) · · · 2 · 1 = 2n · n!,
3 · 5 · 7 · · · (2n − 1)

y como claramente ET = 22n porque cada una de las 2n bombillas puede estar,
independientemente de las demás, encendida o apagada, el problema se reduce a
demostrar que
 
2n 3 · 5 · 7 · · · (2n − 1) 2n (2n)! (2n)! 2n
EB = 2 · =2 · n = = .
n
2 · n! (2 · n!)2 (n!)2 n
 
2n
La demostración de que EB = puede hacerse de distintas maneras. Una
n
de ellas la dada en la Solución 1. Otras maneras son:
2n n n n
Considerando el binomio  (1  + x) = (1 + x) · (1 + x) . El coeficiente de x
2n
es por una parte igual a , y por otra igual a la suma de los productos de
n  
 
n n
los coeficientes respectivos y de xm y xn−m en cada (1 + x)n .
m n−m
También puede verse como el número de caminos en una cuadícula de n × n,
desde (0, 0) hasta (n, n), en la que cada desplazamiento lleva desde (x, y) bien a
(x + 1, y) bien a (x, y + 1). El número total de caminos, cada uno de los cuales
ha de tener
 2n desplazamientos de los que n son en horizontal y n en vertical,
2n
es . Ahora bien, si consideramos la diagonal “descencente” formada por los
n
puntos (x, y) tales que x + y = n, cada camino ha de pasar por exactamente uno
de ellos. Los que pasan por (m, n − m) tienen, en sus primeros n desplazamientos,
m horizontales y n − m verticales, y en sus siguientes   2 n − m
   n desplazamientos,
n n 2n
horizontales y m verticales, para un total de = caminos
m n−m n
pasando por ese punto. Sumando las contribuciones de cada elemento de esta
diagonal se halla la relación pedida.
Problema 6
Sea ABC un triángulo con AB 6= AC, sea I su incentro, γ su circunferencia
inscrita y D el punto medio de BC. La tangente a γ por D diferente de BC toca
a γ en E. Demuestra que AE y DI son paralelas.

Solución 1:
(Ver la figura 1) Sea P el punto de tangencia de γ con BC. Sean Q y R los
puntos simétricos de P con respecto a D e I, respectivamente. Tenemos que:

• ER es paralela a DI. Por un lado, DI es perpendicular a P E (es de hecho


la mediatriz de P E). Por otro lado, por ser R el punto diametralmente
opuesto a P en γ el ángulo ∠P ER es recto.

• QR es paralela a DI. Esto es así porque D e I son los puntos medios de


P Q y P R.

De ahí se deduce que Q, R y E están alineados, en una recta paralela a DI. Para
concluir el problema basta demostrar que A, Q y R están alineados.
Trazando la paralela a BC por R obtenemos un triángulo semejante a ABC,
que es el resultante de tomar una homotecia de ABC con respecto a A. Tanto
R como Q son los puntos de tangencia de la circunferencia exinscrita del lado
opuesto de A, por lo que la homotecia envía Q a R. Concluimos que A, Q y R
están alineados.
Nota: (Ver la figura 2) A continuaciónn se incluye la demostración de que
los puntos de tangencia de las circunferencias inscrita y exinscrita con BC son
simétricos con respecto al punto medio de BC. Usando la notación de la figura,
tenemos que demostrar que BP = CX. Por un lado tenemos que BP + CP =
BX +CX. Por otro, tenemos CP −BP = CN −BM = CN +AN −BM −AM =
AC − AB = (AY − AB) − (AZ − AC) = BY − CZ = BX − CX. De este sistema
de ecuaciones se halla BP = CX.
Solución 2:
Sea P el punto de tangencia de γ con BC, y sean S y T los puntos medios
de EP y AP , respectivamente. Claramente ST es paralela a AE. Como DI es la
mediatriz de EP , tenemos que S está en DI. Por lo tanto, el problema se reduce
a demostrar que T está en DI.
Sea L el punto en el que la bisectriz AI corta a BC. Aplicando el teorema
de Menelao al triángulo AP L, tenemos que demostrar que TATP PDL D LI
IA
= 1. Por
AT
definición de T , T P = 1.
Si a, b y c son las longitudes de los lados de ABC, por el teorema de la bisectriz
tenemos BL CL
= cb . Como BL + LC = a, tenemos CL = b+c ab
. Aplicando ahora el
LI CL a
teorema de la bisectriz a ALC, tenemos IA = CA = b+c .
Tenemos también CD = a2 y CP = a+b−c 2
, con lo que
a+b−c
PD CP − CD 2
− a2 b+c
= = ab
= .
DL CL − CD b+c
− a2 a
AT P D LI
Sustituyendo, obtenemos T P DL IA
= 1, como queríamos demostrar
Solución 3:
Sea ω la circunferencia con centro D que pasa por E. Por simetría respecto de
la recta DI, ω también pasa por el punto F de tangencia de γ con BC. El segundo
punto, G, donde ω corta a BC es claramente el simétrico de F con respecto al
punto medio D de BC, que como es conocido es el punto donde la circunferencia
A−exinscrita es tangente al lado BC. Es también conocido que existe una ho-
motecia con centro A que convierte γ en esta circunferencia A−exinscrita, y por
esta homotecia, los puntos A y G están alineados con el punto F 0 diametralmente
opuesto a F en γ. Ahora bien, como F G es diámetro de ω y F F 0 es diámetro
de γ estando E en ambas circunferencias, se tiene que ∠F EF 0 = ∠GEF = 90o ,
luego F 0 , E, G están alineados, y por lo tanto la recta AE es perpendicular a F E.
Como E, F son los puntos de corte de γ y ω, son simétricos respecto a la recta
DI que une sus centros. Luego DI es perpendicular a F E, y por lo tanto paralela
a AE, como queríamos demostrar.

Solución 4:
Sea un sistema de coordenadas cartesianas tal que el eje horizontal sea la recta
BC, y el eje vertical pase por I. Se tiene entonces que D ≡ (δ, 0) para un cierto
δ 6= 0 y que como D es el punto medio de BC, B ≡ (−∆ + δ, 0) y C ≡ (∆ + δ, 0),
donde ∆ = a2 . Además, I ≡ (0, r), donde r es el inradio.
El origen O que es el punto de tangencia de BC con γ y E son claramente
simétricos respecto a la recta DI ≡ y = r − rxδ
. Luego E está en la recta y = δx r
,
2 2 2
y sobre la circunferencia γ, que es x + (y − r) = r . Se tiene entonces por
sustitución, y descartando la solución x = 0 que produce O, que

2r2 δ 2r2 δ
 
E≡ , .
r2 + δ 2 r2 + δ 2
De forma análoga podemos hallar el punto de tangencia de γ con el lado AB,
sin más que sustituir la coordenada horizontal δ de D por la coordenada horizontal
−∆ + δ de B, obteniéndose que este punto es
 2
2r(δ − ∆)2

2r (δ − ∆)
E≡ , .
r2 + (δ − ∆) r2 + (δ − ∆)2
y por lo tanto la recta AB, que pasa por este punto y por B, tiene ecuación

2r(δ − ∆)
y= (x − δ + ∆).
r2 (δ − ∆)2
Como la coordenada horizontal de C es igual a la de B pero con signo opuesto
para ∆, invirtiendo el signo de ∆ en la anterior ecuación, obtenemos la de la recta
AC:

2r(δ − ∆)
y= (x − δ − ∆).
r2− (δ + ∆)2
Resolviendo el sistema formado por ambas ecuaciones, obtenemos
2r2 δ 2r(δ 2 − ∆2 )
 
A≡ , .
r2 + δ 2 − ∆2 r2 + δ 2 − ∆2
La pendiente de la recta AE viene entonces dada por la diferencia entre sus
coordenadas y,

2r3 ∆2
(r2 + δ 2 )(r2 + δ 2 − ∆2 )
dividida entre la diferencia de sus coordenadas x,

2r2 δ∆2
,
(r2 + δ 2 )(r2 + δ 2 − ∆2 )
que es claramente igual a − rδ , y por lo tanto igual a la pendiente de DI, y hemos
terminado.
Nota: En esta solución puede haber en algún momento denominadores que
se anulen, casos que por lo tanto han de ser tratados aparte. En concreto, los
denominadores que podrían a priori anularse son r2 − (δ − ∆)2 , r2 − (δ + ∆)2 y
r2 + δ 2 − ∆2 . El que r2 = (δ − ∆)2 no es sin embargo ningún problema. En ese
caso, la recta AB sería vertical, y la coordenada x de A sería

2(δ − ∆)2 δ
,
(δ − ∆)2 ) + δ 2 − ∆2
idéntica a la de B, quedando determinada la coordenada y de A por la recta AC.
De forma análoga, si r2 = (δ − ∆)2 , la recta AC sería vertical, siendo la
coordenada x de A igual a δ + ∆, y quedando determinada su coordenada y por
la recta AB. Finalmente, como ∆ = a2 y δ = | a2 − c+a−b
2
| = |b−c|
2
, tenemos que
r2 = ∆2 − δ 2 es equivalente a

a2 − (b − c)2 a2 − b2 − c2 + 2bc A B C
r2 = = = bc sin2 = r2 cot cot ,
4 4 2 2 2
donde hemos usado el conocido resultado r = 4R sin A2 sin 2b sin C2 . Ahora bien, a
su vez esto sería equivalente a cos B+C
2
= 0, absurdo pues B + C < 90o . Obsérvese
que la condición para que ese tercer denominador se anule es B + C = 180◦ . Esto
es intuitivamente consistente con que el denominador al hallar las coordenadas
de A se anule: el que este denominador sea nulo es equivalente a que el punto A
esté “en el infinito”, que a su vez es equivalente a que las rectas AB y AC sean
paralelas, es decir B + C = 180◦ .
En definitiva, los denominadores que pueden realmente anularse no suponen
un problema en la solución propuesta, sino que simplemente indican que dos de
las rectas definidas en la solución son verticales.
LVI Olimpiada matemática Española (Concurso Final)

Enunciados y Soluciones

1. Decimos que un polinomio p(x), con coeficientes reales, es almeriense si


tiene la forma
p(x) = x3 + ax2 + bx + a
y sus tres raı́ces son números reales positivos en progresión aritmética. Halla
todos los polinomios almerienses tales que p(7/4) = 0.

Solución. Llamemos α ≤ β ≤ γ a las raı́ces del polinomio. De la condición de


estar en progresión aritmética tenemos que existe un número real no negativo
δ de manera que α = β − δ y γ = β + δ. Por su parte, utilizando las fórmulas
de Cardano–Viète, resulta que

αβγ = −a = α + β + γ,

o lo que es lo mismo
β(β 2 − δ 2 ) = 3β.
Como las raı́ces son distintas de cero, se llega finalmente a que

(β − δ)(β + δ) = β 2 − δ 2 = 3.

Tenemos que analizar los tres casos posibles:

(a) Si β − δ = 7/4, entonces β + δ = 12/7. De aquı́ se tiene que β = 97/56 y


δ = −1/56, lo cual contradice la hipótesis de que δ ≥ 0 (alternativamente,
no es posible que β + δ < β − δ).

(b) Si β = 7/4, entonces δ 2 = 1/4. Por lo tanto, las raı́ces del polinomio son
(3/2, 7/4, 2) y se obtiene que
21 2 73 21
p(x) = (x − 3/2)(x − 7/4)(x − 2) = x3 − x + x− .
4 8 4
1
(c) Si β + δ = 7/4, entonces β − δ = 12/7. De aquı́ se tiene que β = 97/56
y δ = 1/56. Por lo tanto, las raı́ces del polinomio son (12/7, 97/56, 7/4)
y se obtiene que
291 2 14113 291
p(x) = (x − 12/7)(x − 97/56)(x − 7/4) = x3 − x + x− .
56 1568 56

2. Consideramos la sucesión de números enteros {f (n)}∞


n=1 definida por:

ˆ f (1) = 1.

ˆ Si n es par, f (n) = f (n/2).

ˆ Si n > 1 es impar y f (n − 1) es impar, entonces f (n) = f (n − 1) − 1.

ˆ Si n > 1 es impar y f (n − 1) es par, entonces f (n) = f (n − 1) + 1

a) Calcula f (22020 − 1).


b) Demuestra que {f (n)}∞ n=1 no es periódica, es decir, no existen enteros pos-
itivos t y n0 tales que f (n + t) = f (n) para cualquier n ≥ n0 .

Solución. En primer lugar, hacemos notar que la sucesión está bien definida:
para cada n ∈ N, n > 1, el valor de f (n) está perfectamente determinado a
partir de los valores de f (r) con r < n.
Consideramos la sucesión g(n) dada por g(n) = 0 si la expresión binaria de
n tiene un número par de unos; y g(n) = 1 si la expresión binaria de n tiene
un número impar de unos. Es obvio que g cumple todas las condiciones que
definen a f , luego f (n) = g(n) para todo n ∈ N.
De esta forma, puesto que 22020 − 1 se escribe en binario con 2020 unos,

f (22020 − 1) = 0

Finalmente, veamos que la sucesión no es periódica. Supongamos que lo fuera


a partir de un valor n0 y con periodo t. Tomamos el entero, r, tal que 2r ≤ t <
2r+1 . Observamos que la expresión binaria de 2r + t se obtiene sustituyendo el
primer 1 (por la izquierda) de la expresión de t por dos dı́gitos 10, con lo que
f (2r + t) = f (t). Tomamos un entero k > r + 1 tal que 2k > n0 . Ahora, se
tiene:
Si f (t) = 1, entonces f (2k + t) = 0 y por tanto, f (2k + t) 6= f (2k ) = 1.

2
Si f (t) = 0, entonces f (2k + 2r + t) = 1 + f (2r + t) = 1 + f (t) = 1 y por tanto,
f (2k + 2r + t) 6= f (2k + 2r ) = 0.

3. A cada punto del conjunto A = {(x, y, z) ∈ Z3 }, formado por los puntos


del espacio tridimensional cuyas coordenadas son enteras, le asignamos un
color de entre p colores posibles. Demuestra que forzosamente existe algún pa-
ralelepı́pedo recto (poliedro de seis caras en el que cada cara es un rectángulo)
cuyos vértices pertenecen a A y son todos del mismo color.
 
p+1
Solución. Ponemos n = p . Para cada j ∈ {0, 1, . . . , n}, pensamos
2
en el conjunto A0j = {(i, j, 0) ∈ A; 0 ≤ i ≤ p}. Por el Principio del Palomar,
podemos asegurar que, para cada j hay dos puntos de A0j del mismo color.
Puede que haya más de dos o que ocurra para más de un color; no importa,
de entre
 los p+ 1 marcamos dos que comparten color. Teniendo en cuenta que
p+1
hay p + 1 posibles valores de j, de nuevo por el Palomar, existirán al
2 
p+1
menos r = + 1 de esos A0j (digamos A0j1 , . . . , A0jr ) en los que el color
2
repetido en todos ellos es el mismo (color c). Como el número de maneras de
elegir los dos lugares (valores de i) de cada uno de esos A0jt , con t = 1, . . . , r,
 
p+1
es , usando de nuevo el Principio del Palomar, podemos asegurar que
2
existen Ajα y Ajβ , con α, β ∈ {1, . . . , r}, en los que el color c aparece dos veces
en cada uno y en los mismos lugares. O sea, queda probado que en el plano
z = 0, y con valores de la coordenada x ∈ {0, 1, . . . , p} y de la coordenada
y ∈ {0, 1, . . . , n}, existen cuatros puntos

(i1 , jα , 0), (i2 , jα , 0) ∈ Ajα (i1 , jβ , 0), (i2 , jβ , 0) ∈ Ajβ


que son vértices de un rectángulo (siempre de Lados Paralelos a los Bordes de
la Cuadrı́cula: LPBC) y los cuatro de un mismo color.
El número, m, de formas de elegir, en una cuadrı́cula de (p + 1) × (n + 1)
puntos, cuatro puntos que sean vertı́ces de un rectángulo LPBC es lo mismo
que el número de maneras
 de elegir
  dos valores de la abscisa y dos valores de
p+1 n+1
la ordenada, o sea m = .
2 2
Para cada k ∈ {0, 1, . . . , p × m} consideramos la cuadrı́cula Ak = {(i, j, k) ∈
A; 0 ≤ i ≤ p, 0 ≤ j ≤ n}. Tenemos p m + 1 cuadrı́culas y en cada una de
ellas hay cuatros puntos que comparten color y son vértices de un rectángulo

3
LBPC. Por el Principio del Palomar, hay al menos p + 1 de esas cuadrı́culas
en las que los cuatro puntos de cada una de ellas ocupan los mismos lugares
en lo referido a sus dos primeras coordenadas. Finalmente, de nuevo por el
Palomar, hay (al menos) dos de esas p + 1 cuadrı́culas tales que el color común
de los cuatro vértices de una es el mismo que el color común de los cuatro
vértices de la otra. Ası́ tenemos ocho puntos de A que comparten color y son
vértices de un paralelepı́pedo recto.

4. Ana y Benito juegan a un juego que consta de 2020 rondas. Inicialmente,


en la mesa hay 2020 cartas, numeradas de 1 a 2020, y Ana tiene una carta
adicional con el nœmero 0. En la ronda k-ésima, el jugador que no tiene la
carta k − 1 decide si toma la carta k o si se la entrega al otro jugador. El
número de cada carta indica su valor en puntos. Al terminar el juego, gana
quien tiene más puntos. Determina qué jugador tiene estrategia ganadora, o
si ambos jugadores pueden forzar el empate, y describe la estrategia a seguir.

Solución. Ambos jugadores pueden forzar el empate. Dividimos el juego en


505 etapas, cada una con cuatro rondas consecutivas de la forma

{k, k + 1, k + 2, k + 3}.

Vamos a demostrar que cada jugador puede conseguir al menos la mitad de


los puntos de cada etapa, independientemente de qué jugador tenga la carta
k − 1. No importa qué ocurra en la k-ésima ronda. A partir de ahı́:

ˆ El jugador que recibe la carta k (sin pérdida de generalidad, podemos


suponer que es Ana) puede asegurase al menos el empate en la etapa de
4 turnos. En efecto, si Benito le entrega también la carta k + 1 y la carta
k + 2, entonces Ana ya ha recibido 3k + 3 puntos y gana la etapa de 4
turnos. Si Benito entrega a Ana la carta k + 1 y se queda la k + 2, Ana
toma la carta k + 3 y gana la etapa de 4 turnos. Si Benito se queda la
carta k + 1, Ana entrega la carta k + 2 a Benito y se queda con la carta
k + 3, con lo que empata la etapa de 4 turnos. En resumen, quien recibe
la carta k siempre puede conseguir al menos el empate.

ˆ El jugador que no recibe la carta k (sin pérdida de generalidad, Benito)


se queda con la carta k + 1. Si Ana le entrega la carta k + 2, Benito ya
tiene 2k + 3 puntos y se garantiza el empate en la etapa de 4 turnos. Si
Ana se queda la carta k + 2, Benito se queda con la carta k + 3, con lo
que acaba con 2k + 4 puntos y gana la etapa de 4 turnos. En resumen,
quien no recibe la carta k siempre puede conseguir al menos el empate.

4
5. En un triángulo acutángulo ABC, sea M el punto medio del lado√AB y
P el pie de la altura sobre el lado BC. Prueba que si AC + BC = 2AB,
entonces la circunferencia circunscrita del triángulo BM P es tangente al lado
AC.

Solución. Sea S el punto de AC, al mismo lado de A que C, tal que AS =


√ 2
2AB/2. Este punto cumple que AS 2 = AB 2
= AB · AM , que es la potencia
de A con respecto a la circunferencia circunscrita de BM P ; luego si esta
circunferencia circunscrita pasa por S entonces es tangente a AB. Vamos a
demostrar que esto es cierto, probando que ∠M SB = ∠M P B. Sea N el

P
A N S C


punto medio de AC. Como AS = 2AB 2
= AC+BC
2
= AN + M N , tenemos
N S = N M . De ahı́ obtenemos ∠N M S = ∠N SM . √
Obsérvese que los triángulos ASB y AM S son semejantes, porque AB
AS
= 2=
AS
AM
. De aquı́ se deduce que ∠AM S = ∠ASB. Entonces ∠ABC = ∠AM N =
∠AM S − ∠N M S = ∠ASB − ∠N SM = ∠M SB.
Por otro lado, por ser M el punto medio de la hipotenusa del triángulo
rectángulo AP B, tenemos M P = M B. Esto quiere decir que ∠M P B =
∠M BP = ∠ABC = ∠M SB, como querı́amos demostrar.

6. Sea S un subconjunto finito de los números enteros. Definimos d2 (S) y


d3 (S) de la siguiente manera:

ˆ d2 (S) es el número de elementos a ∈ S para los que existen x, y ∈ Z tales


que x2 − y 2 = a.

ˆ d3 (S) es el número de elementos a ∈ S para los que existen x, y ∈ Z tales


que x3 − y 3 = a.

5
(a) Sea m un número entero y sea S = {m, m + 1, . . . , m + 2019}. Prueba
que
13
d2 (S) > · d3 (S).
7
(b) Sea n un entero positivo y sea Sn = {1, 2, . . . , n}. Prueba que existe un
número N de manera que si n > N ,

d2 (Sn ) > 4 · d3 (Sn ).

Solución. En primer lugar, observamos que un número se puede escribir como


diferencia de cuadrados si y solo si no es de la forma 4k + 2. Para ver esto,
escribimos x2 − y 2 = α · β, donde α y β tienen la misma paridad y simplemente
ponemos x = α+β 2
, y = β−α2
. Por ejemplo, si n = 4k sirve α = 2, β = 2k; si
n = 4k + 1 o n = 4k + 3, sirve α = 1, β = n. Recı́procamente, una diferencia
de cuadrados nunca podrá dar resto 2 al dividir entre 4 (simplemente notando
que un cuadrado es 0 o 1 módulo 4).
Analicemos ahora cómo pueden ser las diferencias de dos cubos. Observamos
que la congruencia módulo 7 del cubo de un entero solo puede ser 0, 1 o 6; en
consecuencia, la diferencia de dos cubos, módulo 7, solo puede ser 0, 1, 2, 5 o
6. Haciendo lo mismo módulo 9, vemos que la congruencia de un cubo módulo
9 solo puede ser 0, 1 u 8; en consecuencia, la diferencia de dos cubos módulo
9, solo puede ser 0, 1, 2, 7 u 8. Por tanto, hay a lo sumo 5 opciones módulo 7
y 5 opciones módulo 9, y, por el teorema chino de los restos, tenemos tan solo
25 opciones módulo 63.
En un intervalo de longitud 2020 (múltiplo de 4), exactamente 3/4 partes de
los números serán no congruentes con 2 módulo 4 (esto es, d2 (S) = 1515). Por
su parte, tenemos que 2020 = 32 · 63 + 4, de manera que

d3 (S) ≤ 32 · 25 + 4 = 804.

Por tanto, tenemos que d2 (S)/d3 (S) ≥ 1515/804 > 13/7, como querı́amos
demostrar.
Pasamos ahora a resolver la segunda parte. El mismo razonamiento usado
anteriormente muestra que
3n 1 3n 1
− ≤ d2 (Sn ) ≤ + .
4 2 4 4
Será suficiente entonces con ver que asintóticamente d3 (n) < 3n
16
, esto es, que
para valores de n suficientemente grandes se cumple esa última desigualdad.

6
Supongamos que x > y > 0. Una primera observación es que si x3 − y 3 ≤ n,
entonces

n ≥ x3 − y 3 = (x − y)(x2 + xy + y 2 ) ≥ x2 + xy + y 2 > 3y 2 ,
p
con lo que y < n/3. Por su parte,

x3 ≤ n + y 3 ≤ n + (n/3)3/2 .
p
Esto es, x ≤ 3 n + (n/3)3/2 . Como el cociente n/(n/3)3/2 tiende a 0 cuando
n tiende a infinito, tenemos que para cualquier δ > 0 existirá un número N
suficientemente grande de manera que cuando n > N ,
q p
x ≤ 3 n + (n/3)3/2 < (1 + δ) n/3.

Como sabemos además que y < x, necesariamente el número de parejas está


2
acotado por (1+δ)2 n/3 . Por tanto, podemos obtener a lo sumo (1 + δ)2 n/6
números. Observamos que no es necesario considerar el caso en el que x, y < 0,
dado que los números obtenidos serán los mismos.
En el caso en que y < 0 < x, tenemos que x3 +(−y)3 ≤ n, y han de ser x, −y <
n1/3 , de manera que podemos obtener a lo sumo n2/3 números. Seleccionando
un δ suficientemente pequeño (por ejemplo δ = 0.01), concluimos que
3n
d3 (Sn ) ≤ (1 + 0.01)2 n/6 + n2/3 <
16
si n es suficientemente grande.

7
LV Olimpiada matemática Española (Concurso Final)

Enunciados y Soluciones
1. Un conjunto de números enteros T es orensano si existen enteros a < b < c
tales que a y c pertenecen a T y b no pertenece a T . Hallar el número de
subconjuntos T de {1, 2, . . . , 2019} que son orensanos.

Solución. El número de subconjuntos de {1, 2, · · · , 2019} es 22019 como es


bien conocido. Contemos ahora el número de estos conjuntos que NO tienen
la propiedad pedida. Claramente, el conjunto vacı́o, y los subconjuntos de
{1, 2, . . . , 2019} con un único elemento no tienen la propiedad pedida, ya que
es imposible elegir a, c distintos en ellos. Hay en total
   
2019 2019
+ = 1 + 2019 = 2020
0 1
de estos conjuntos. Cualquier otro subconjunto que no tenga la propiedad
pedida ha de estar formado por elementos consecutivos de {1, 2, . . . , 2019}.
En efecto, si m, M son respectivamente el mı́nimo y el máximo de T , para que
T NO tenga la propiedad enunciada, todos los elementos m, m+1, m+2, . . . , M
han de estar en T , pues si alguno k no lo estuviera, podemos tomar a = m,
b = k yc = M , cumpliéndose la propiedad. El número de tales subconjuntos
es 2019
2
. En efecto, dado un tal subconjunto,  su mı́nimo m y su máximo
2019
M lo determinan biunı́vocamente, y hay 2 formas distintas de elegir dos
números del conjunto {1, 2, . . . , 2019}, de forma que el menor será m, y el
mayor M .
Finalmente, se tiene que el número total de subconjuntos T de {1, 2, · · · , 2019}
con la propiedad pedida es
 
2019 2019
2 − 2020 − = 22019 − 2039191.
2

2. Determinar si existe un conjunto finito S formado por números primos


positivos de manera que para cada entero n ≥ 2, el número 22 + 32 + · · · + n2
sea múltiplo de algún elemento de S.

1
Solución. Es bien conocido que
n(n + 1)(2n + 1) 2n3 + 3n2 + n
12 + 22 + · · · + n2 = = ,
6 6
con lo que restando 1 a ambos lados, resulta
2n3 + 3n2 + n − 6 (n − 1) (2n2 + 5n + 6)
22 + 32 + · · · + n2 = = .
6 6
Esta expresión sugiere tomar p = (n − 1)/6 o n = 6p + 1 con lo que se obtiene
22 + 32 + · · · + n2 = p 72p2 + 54p + 13 .


Ahora elegimos n = 6p + 1, donde p da resto 1 al dividir entre cualquier primo


de S distinto de 139, y da resto −1 al dividir entre 139. Nótese que el resto al
dividir entre cualquier primo de S distinto de 139 es 72 + 54 + 13 = 139, que
es primo y por lo tanto no divisible por ningún primo de S distinto de 139,
y el resto al dividir entre 139 es −72 + 54 − 13 = −31, claramente tampoco
divisible por 139. Luego esta expresión es coprima con todos los primos de S,
y la respuesta es que no existe tal conjunto S.

3. Los números reales a, b y c verifican que el polinomio p(x) = x4 +ax3 +bx2 +


ax + c tiene exactamente tres raı́ces reales distintas; estas rı́ces son iguales a
tan (y), tan (2y) y tan (3y), para algún número real y. Hallar todos los posibles
valores de y, 0 ≤ y < π.

Solución. Sean r, r, s, t los tres ceros reales y distintos del polynomio p(x).
Mediante las fórmula de Cardano-Viète, identificando los coeficientes cúbico y
lineal, se obtiene
2r + s + t = r2 s + r2 t + 2rst ⇔ 2r(1 − st) = (r2 − 1)(s + t)
De la última igualdad se deduce que si r2 −1 = 0 entonces 1−st = 0 y viceversa.
A continuación, bajo la hipóteis r2 = st = 1 y considerando 0 ≤ y < π,
analizaremos los siguientes casos:
 
π 3π
• Si r = tan y entonces y ∈ , pero para estos valores s = tan 2y
4 4
no está definido.
 
π 3π 5π 7π
• Si r = tan 2y entonces y ∈ , , , . Poniendo s = tan y y
8 8 8 8
s = tan 3y se tiene que para todos los valores anteriores es rs = tan y ·
tan 3y = 1, lo cual es cierto si y + 3y = 4y es un múltiplo impar de π/2,
y esto ocurre para todos los valores anteriores.

2
• Si r = tan 3y entonces se debe cumplir rs = tan y · tan 2y = 1 y la tan 3y
no está definida.
Por tanto solo queda por analizar los casos en que r2 − 1 6= 0 y 1 − st 6=
0. Dividiendo los dos lados de la igualdad 2r(1 − st) = (r2 − 1)(s + t) por
(r2 − 1)(1 − st) resulta
2r s+t
+ = 0.
1 − r2 1 − st
Estas expresiones por separado serı́an las correspondients a las fórmulas de la
tangente del ángulo doble y de la tangente de la suma. Ahora, utilizando la
última expresión, distinguiremos los siguientes casos:
• Si r = tan y, s = tan 2y, t = tan 3y entonces tan 2y + tan 5y = 0 que
admite como soluciones los múltiplos de π2 inferiores a 7π
2
con lo que
 
π 2π 3π 4π 5π 6π
y∈ , , , , , .
7 7 7 7 7 7

• Si r = tan 2y, s = tan y, t = tan 3y entonces tan 4y + tan 4y = 0 que


admite como soluciones los múltiplos de π2 inferiores a 4π, con lo que
 
π 3π 5π 7π
y∈ , , , ,
8 8 8 8
y hemos rechazado y = π2 ya que entonces tan y no serı́a un número real,
y también π4 y 3π
4
ya que entonces tan(2y) tampoco serı́a un número real.

• Si r = tan 3y, s = tan y, t = tan 2y entonces tan 6y + tan 3y = 0 que


admite como soluciones los múltiplos de π2 inferiores a 9π
2
, con lo que
 
π 2π π 4π 5π 2π 7π 8π
y∈ , , , , , , , .
9 9 3 9 9 3 9 9

Estos son todos los valores, y hemos terminado.

4. Calcular todos los pares de enteros (x, y) tales que


34 23 x2 + y 2 = x3 y 3 .


Solución. Nótese en primer lugar que si xy = 0, entonces x2 + y 2 = 0, de


donde resulta la solución x = y = 0. Nótese también que si xy < 0, entonces

3
x2 + y 2 < 0, absurdo, luego x, y tienen ambos el mismo signo. Como cambiar
simultáneamente de signo a x y a y no altera la ecuación, podemos asumir a
partir de este momento y sin pérdida de generalidad que x, y son ambos enteros
positivos. Sean x = 3m a e y = 3n b con m, n enteros no negativos y a, b enteros
coprimos y no divisibles por 3. Supongamos que m ≥ n. Entonces la ecuación
dad se transforma en

8((3m−n a)2 + b2 ) = 33m+n−4 a3 b3 .

Como todo cuadrado perfecto da resto 0 o 1 al dividir entre 3, como es bien


conocido, el miembro de la izquierda no es divisible por 3 y para que se cumpla
la ecuación debe ser el exponente de 3 en el término de la derecha igual a cero.
Es decir, 3m + n − 4 = 0 y como m ≥ n ≥ 0 esto implica que m = n = 1. La
ecuación ahora se simplifica y queda

8(a2 + b2 ) = a3 b3 .

Por la simetrı́a de la expresión podemos suponer que a ≥ b. Entonces

16a2 ≥ a3 b3 ⇔ 16 ≥ ab3

y tenemos dos posibilidades (1) b = 2 de donde resulta a = 2, (2) b = 1 pero


en este caso los únicos valores posibles de a son 1, 2, 4, 8 y no satisfacen la
ecuación a3 − 8a2 − 8 = 0. Se deduce que (a, b) = (2, 2) y (x, y) = (6, 6).
Finalmente, se tiene que las únicas soluciones posibles son:

(x, y) = (−6, −6), (x, y) = (0, 0), (x, y) = (6, 6).

5. Se consideran todos los pares (x, y) de números reales tales que 0 ≤ x ≤


y ≤ 1. Sea M (x, y) el máximo valor del conjunto

A = {xy, xy − x − y + 1, x + y − 2xy} .

Hallar el mı́nimo valor que puede tomar M (x, y) para todos estos pares (x, y).

Solución 1. Haciendo el cambio de variable xy = p, y x + y = s y escribiendo


los tres elementos del conjunto A en términos de s y p, tenemos

a = xy = p, b = xy − x − y + 1 = (1 − x)(1 − y) = s − 1 + p, c = s − 2p

verificándose que a + b + c = 1. Observemos que s2 − 4p = (x − y)2 ≥ 0.


Ahora consideremos los siguientes casos:

4
1 1 4
• Si 0 ≤ x ≤ y ≤ ⇒ p = xy ≤ ⇔ b = (1 − x)(1 − y) ≥ .
3 9 9
 2   
4 2 4 1 4
• Si c = s−2p ≤ ⇒ 0 ≤ s −4p = 2p + −4p = 4 p − p− ,
9 9 9 9
1 4 1 4
de donde se deduce que p ≤ ó p ≥ . Si a = p ≤ ⇒ b ≥ , c =
9 9 9 9
4 4 1 4
s − 2p ≤ y si a = p ≥ ⇒ b ≤ , c ≤ . Por lo tanto, en cualquier
9 9 9 9
caso
4
máx A ≥
9
4
Para hallar el mı́nimo , las desigualdades deberán ser igualdades y se tiene:
9
1 4 2 1
• p = , s − 2p = ⇒ s = ⇒ x = y = ,
9 9 3 3
4 4 4 2
• p = , s − 2p = ⇒ s = ⇒ x = y = .
9 9 3 3

Solución 2. En primer lugar, se observa que xy + (xy − x − y + 1) + (x +


y − 2xy) = 1. Ası́, si uno de los tres elementos de A vale 19 o menos, entonces
los otros dos suman 89 o más, luego si el menor valor de A vale 19 o menos, el
mayor valor de A vale 49 o más.
Supongamos que xy ≥ x + y − 2xy. Entonces, por la desigualdad entre medias
aritmética y geométrica se tiene que
√ √ 2
3xy ≥ x + y ≥ 2 xy, xy ≥ ,
3
4
y el mayor valor de A es al menos xy ≥ 9
en este caso.
Supongamos que xy − x − y + 1 ≥ x + y − 2xy. Nuevamente por la desigualdad
entre medias aritmética y geométrica, se tiene que
√ √ √
3xy + 1 ≥ 2(x + y) ≥ 4 xy ⇔ (3 xy − 1) ( xy − 1) ≥ 0.

La última desigualdad se verifica cuando xy ≥ 1, con lo que el mayor valor

de A serı́a al menos xy ≥ 1, o bien xy ≤ 13 , para xy ≤ 19 , y por la observación
inicial el mayor valor de A serı́a 49 o más.
En cualquier otro caso, x + y − 2xy es el mayor valor de A, y supongamos que
es inferior a 49 . Es decir, x + y − 2xy < 94 . Aplicando la desigualdad entre las
medias aritmética y geométrica, se obtiene
 2  2
4 √ √ 1 1 1
2xy + > x + y ≥ 2 xy ⇔ xy − > =
9 2 36 6

5
√ √
Si xy − 12 > 16 , entonces xy > 23 y el mayor valor de A es al menos xy > 94 ,
√ √
contradicción. Si 21 − xy > 16 , entonces xy < 13 , con lo que xy < 91 , y por
la observación inicial el mayor valor de A es mayor que 49 , contradicción.
Luego el mayor valor de A nunca puede ser menor que 49 , y ése valor es 
en efecto
el mı́nimo que toma el máximode A, pues se puede obtener A = 94 , 19 , 94
tomando x = y = 23 , o A = 19 , 49 , 94 tomando x = y = 13 . Viendo las
condiciones de igualdad en los dos primeros casos analizados, se tiene además
que éstos son todos los posibles pares de valores (x, y) para los que el mayor
valor de A toma dicho valor mı́nimo.

6. En el triángulo escaleno ABC, la bisectriz del ángulo A corta al lado BC en


el punto D. Las rectas que pasan por D y son tangentes a las circunferencias
circunscritas de los triángulos ABD y ACD cortan a las rectas AC y AB en
los puntos E y F , respectivamente. Si BE y CF se cortan en G, demostrar
que los ángulos ∠EDG y ∠ADF son iguales.

Solución. Se verifican las igualdades de ángulos ∠ADE = ∠B y ∠ADF =


∠C, por ser ángulos semiinscritos en las circunferencias (ABD) y (ADC),
respectivamente. Por tanto el cuadrilátero AF DE es inscriptible y de eso se

F E
G

B D C

deduce que

∠AF E = ∠ADE = ∠B y ∠AEF = ∠ADF = ∠C,

y por tanto, EF es paralela a BC. Además DE = DF por ser cuerdas


correspondientes al ángulo ∠A/2, ası́ que el triángulo F DE es isósceles.

6
Sea M el punto medio de EF , L = AD ∩ EF , y H = DG ∩ EF . Aplicando el
teorema de Thales, se obtiene que
LE DC
=
LF BD
Por otro lado, de 4F GH ∼ 4CGD resulta
HF GH
= ,
DC GD
de 4EGH ∼ 4BGD, se obtiene
HE GH
=
BD GD
y de aquı́ que
HF DC
= ,
HE DB
luego H y L son simétricos respecto de M , como vermos mas tarde, y ∠LDM =
∠HDM = α siendo
∠C − ∠B
α = 90◦ − ∠ADC = si C > B,
2
y
∠B − ∠C
α = 90◦ − ∠ADB = si B > C.
2
Finalmente, se tiene que

∠GDE = ∠ADE − α = ∠ADF si B > C,

y
∠GDE = ∠ADE + α = ∠ADF si B < C.
Ahora falta probar que L y H son simétricos respecto a M . En efecto, si
denotamos LF + LE = HF + HE = s, entonces
LE HF
= ,
LF HE
de donde
s s
= ⇒ LF = HE y LE = HF
LF HE
con lo que HM = M L como se querı́a demostrar.

7
Agradecimientos

Agradecemos a los proponentes de los problemas sus contribuciones para for-


mar la Lista Corta de la que se han seleccionado los seis Problemas de la
competición y al profesor Juan Manuel Conde y sus colaboradores por la se-
lección de los mismos.

8
LIV Olimpiada matemática Española (Concurso Final)

Enunciados y Soluciones

1. Determina todos los enteros positivos x, tales que 2x + 1 sea un cuadrado


perfecto, pero entre los números 2x + 2, 2x + 3, · · · , 3x + 2, no haya ningún
cuadrado perfecto.

Solución. Sea n un número entero tal que 2x+1 = n2 y n2 ≤ 3x+2 < (n+1)2 .
De la primera ecuación se obtiene x = (n2 − 1)/2 y sustituyendo este valor en
la doble desigualdad, resulta

3n2 + 1
n2 ≤ < n2 + 2n + 1 ⇔ 2n2 ≤ 3n2 + 1 < 2n2 + 4n + 2
2
En la última expresión la primera desigualdad se cumple para todo n entero,
la segunda puede escribirse como
√ √
n2 − 4n − 1 < 0 ⇔ 2 − 5 < n < 2 + 5,

y se verifica para todos los enteros n ∈ {0, 1, 2, 3, 4}. De entre estos valores,
sólo para n = 3, se obtiene que x = (32 − 1)/2 = 4 es entero positivo, y esta
es la única solución del problema, ya que en este caso 2x + 1 = 9 y entre los
números 10, 11, 12, 13 y 14 no hay ningún cuadrado perfecto.

2. Se colocan 2n + 1 fichas, blancas y negras, en una fila (n ≥ 1). Se dice que


una ficha está equilibrada si el número de fichas blancas a su izquierda, más el
número de fichas negras a su derecha es n. Determina, razonadamente, si el
número de fichas que están equilibradas es par o impar.

Solución. Numeramos las posiciones en la fila desde 1 hasta 2n + 1, de


izquierda a derecha. Definimos la valoración de una cierta posición k =
1, 2, · · · , 2n + 1 como el número de fichas blancas a su izquierda más el número
de fichas negras a su derecha, con lo que una ficha está equilibrada si y sólo si
su valoración es igual a n.

1
Supongamos que las fichas en posiciones k y k + 1 tienen distinto color. El
número de fichas blancas a su izquierda en las posiciones 1, 2, · · · , k − 1 y el
número de fichas negras a su derecha en las posiciones k + 2, k + 3, · · · , 2n + 1
son los mismos para ambas. La suma de estas dos cantidades es la valoración
común de ambas si la ficha en posición k es negra y la ficha en posición k + 1
es blanca; pero en caso contrario, la ficha negra en k + 1 tiene una ficha
blanca adicional a su izquierda, y la ficha blanca en posición k tiene una ficha
negra adicional a su derecha, con lo que la valoración de estas dos fichas es en
cualquiera de los dos casos la misma.

Tenemos entonces que, sean cuales sean sus colores, si intercambiamos las
fichas en posiciones k y k + 1, la paridad del número de fichas equilibradas no
varı́a. En efecto, la valoración de las fichas en las posiciones 1, 2, · · · , k − 1
y en las posiciones k + 2, k + 3, · · · , 2n + 1 no varı́an con el cambio, si las
fichas en posiciones k y k + 1 tienen el mismo color simplemente intercambian
sus valoraciones, y si tienen distinto color, entonces ambas tienen la misma
valoración antes del cambio, y ambas tienen la misma valoración después del
cambio, luego el número de fichas equilibradas permanece constante, puede
aumentar en 2, o reducirse en 2.

Como toda permutación se puede descomponer en intercambios sucesivos de


elementos contiguos, la paridad del número de fichas equilibradas no cambia
cuando situamos todas las fichas negras en las primeras posiciones de la fila,
y todas las blancas en las últimas posiciones de la fila.

Sea a el número de fichas negras y b el de fichas blancas, con la condición de


que a + b = 2n + 1. Representamos la fila de fichas blancas y negras mediante
un camino en el interior de un rectángulo a × b, que empieza por la esquina
inferior izquierda del rectángulo. La fila se recorre de izquierda a derecha. Si
la ficha es negra se marca un paso unidad hacia arriba, y si es blanca, un paso
unidad hacia la derecha. Se considera el segmento L que une dos lados parale-
los del rectángulo, pasa por su centro O, y forma ángulos de 45◦ con los lados
del mismo. El rectángulo queda ası́ dividido por L en dos polı́gonos simétricos
respecto del punto O. Y las fichas equilibradas corresponden precisamente a
los pasos del camino que cruzan L. Si consideramos la fila con todas las fichas
negras al principio de la fila a la izquierda de las fichas blancas (lo cual no
altera la paridad del número de fichas equilibradas), entonces el correspondi-
ente camino es formado por el lado vertical del rectángulo y el lado superior
del mismo. Ası́ L corta una vez al camino y por tanto el número de fichas
equilibradas es impar.

2
3. Sea O el circuncentro del triángulo acutángulo ABC y sea M un punto arbi-
trario del lado AB. La circunferencia circunscrita al triángulo AM O interseca
por segunda vez a la recta AC en el punto K y la circunferencia circunscrita
al triángulo BOM interseca por segunda vez a la recta BC en el punto N .
Prueba que
1
Área (M N K) ≥ Área (ABC)
4
y determina el caso en que se alcanza la igualdad.

Solución. Utilizaremos las notaciones habituales en Geometrı́a del triángulo


(R, radio de la circunferencia circunscrita al 4ABC, O su incentro; a longitud
MA
del lado BC del triángulo ABC, etc.). Sea = k > 0, entonces
MB
MA + MB c c
k+1= = ⇒ MB =
MB MB k+1

K
N

B
A M

En lo que sigue utilizaremos el parámetro k para fijar la posición del punto


M en el segmento AB. Si α = ∠M N B = ∠M OB y β = ∠M OA = ∠M KA
entonces α+β = 2 C. Sean r1 y r2 los radios de las circunferencias circunscritas
a los triángulos OM A y OM B, respectivamente. Aplicando el teorema de los
senos generalizado en los dos triángulos OM B y OM A, habida cuenta que
∠OM A + ∠OM B = 180◦ , resulta
OA OB
= 2 r1 y = 2 r2
sin ∠OM A sin ∠OM B
De lo anterior se deudce que r1 = r2 ya que OA = OB = R y los ángulos
∠OM A y ∠OM B son suplementarios.

3
Aplicando el teorema de los senos a los triángulos M AK y M N B, se obtiene
MK MN
= 2 r1 y = 2 r2
sin A sin B
De lo anterior, resulta
MK MN MK MN
= ⇒ = (1)
sin A sin B a b
Por otra parte, se verifica que ∠KM A = 180◦ − (β + A) y ∠N M B = 180◦ −
(α + B). Entonces,

∠KM N = 180◦ − (∠N M B + ∠KM A) = C (2)

De (1) y (2) se deduce que los triángulos M N K y ABC son semejantes y que
M N es el lado homólogo de b en la semejanza. Entonces, para el cociente entre
las áreas, se tiene
[M N K] MN2
=
[ABC] b2
En los triángulos M OB y M N B se tiene
MB OM MN OM · sin B
= ◦
= ⇒ MN =
sin α sin(90 − C) sin B cos C
Además, aplicando el Teorema del Coseno al triángulo M OB, resulta

OM 2 = M B 2 + R2 − 2 M B · R · cos(90◦ − C) = M B 2 + R2 − 2 M B · R · sin C

Teniendo en cuenta que c = 2R sin C, entonces


sin2 B c2 c2
 
2 2
MN = +R −
cos2 C (k + 1)2 k+1
2
c2 k
 
sin B 2
= R −
cos2 C (k + 1)2
sin2 B R2 (k + 1)2 − 4R2 sin2 C
 
=
cos2 C (k + 1)2
Llevando esto al cociente entre las áreas de los triángulos M N K y ABC y
teniendo en cuenta que b = 2R sin B, se obtiene
[M N K] MN2 1 sin2 B 2 2
2 (k + 1) − 4k sin C
= = · · R ·
[ABC] b2 b2 cos2 C (k + 1)2
(k − 1)2 + 4k cos2 C
=
4(k + 1)2 cos2 C

4
Dado que cos2 C < 1, entonces se verifica que
[M N K] cos2 C [(k − 1)2 + 4k] 1
> 2 2
=
[ABC] 4(k + 1) cos C 4
El cociente entre las áreas es igual a 1/4 para k = 1, es decir, cuando M es el
punto medio del segmento AB.

4. Los puntos de una superficie esférica de radio 4, se pintan con cuatro


colores distintos. Prueba que existen dos√puntos sobre la superficie
√ que tienen
el mismo color y que están a distancia 4 3 o bien a distancia 2 6.
NOTA: La distancia entre dos puntos es la distancia euclı́dea; es decir la lon-
gitud del segmento rectilı́neo que los une.

Solución. Dado que se pintan los puntos de la esfera con cuatro colores
distintos, para resolver el problema será suficiente con encontrar cinco puntos
distintos que cumplan las condiciones del enuciado y aplicar el Principio del
Palomar.

Esto puede hacerse, por ejemplo, observando que si inscribimos un triángulo


equilátero en una circunferencia √
de diámetro máximo de la esfera, entonces el
lado de dicho triángulo mide 4 3, y que tomando dos de tales triángulos,
contenidos en planos perpendiculares √y que tengan un vértice común, los
vértices no comunes distan entre sı́ 2 6. En efecto, en un sistema de co-
ordenadas cartesianas con origen en el centro de la esfera, la superficie de ésta
viene dada por los puntos cuyas coordenadas (x, y, z) satisfacen la ecuación
x2 + y 2 + z 2 = 16.√ Sobre esta superficie,
√ consideremos los siguientes puntos
A(4, 0, 0), B(−2, 2 3, 0), C(B(−2, −2 3, 0) que están sobre la circunferencia
Γ1 = {(x, y, z) | x2 + y 2 = 16, z = 0}

y son los vértices de un triángulo
√ equilátero √de lado 4 3. Igualmente, los pun-
tos A(4, 0, 0), D(−2, 0, 2 3), E(B(−2, 0, −2 3) son los vértices de un triángulo
equilátero inscrito en la circunferencia
Γ2 = {(x, y, z) | x2 + z 2 = 16, y = 0}
Nótese que los planos de las circuneferencias Γ1 y Γ2 son perpendiculares, que
los triángulos equiláteros tienen el vértice común A, y que
√ √
AB = AC = AD = AE = BC = DE = 4 3 y BD = BE = CD = CE = 2 6

Luego cada uno √ de estos cinco puntos A, B, C, D, E está a distancia 4 3 o
a distancia 2 6 de cada uno de los cuatro restantes, y por el principio del
palomar, hay dos del mismo color, como querı́amos demostrar.

5
5. Sean a y b dos números positivos primos entre sı́. Se dice que un entero
positivo n es débil si no puede ser escrito en la forma n = ax + by, para algunos
enteros x e y no negativos. Prueba que si n es débil y n < ab 6
, entonces existe
un entero k ≥ 2, tal que kn es débil.

Solución. Trivialmente se observa que la suma de enteros positivos no débiles


es no débil. Esto motiva considerar para cada entero positivo n los enteros 2n
y 3n. Si ambos no son débiles, entonces kn no es débil para cada k ≥ 2, ya que
kn puede ser escrito en la forma 2nr + 3ns para algunos enteros no negativos
r y s.

Sea n un entero positivo débil y supongamos que 2n y 3n son no débiles, con


2n = ax1 + by1 y 3n = ax2 + by2 . Entonces

0 = 2(3n) − 3(2n) = a(2x2 − 3x1 ) + b(2y2 − 3y1 ) (3)

y
n = 3n − 2n = a(x2 − x1 ) + b(y2 − y1 ) (4)
De (4) por ser n débil, resulta que x2 < x1 o y2 < y1 . Sin pérdida de gen-
eralidad, podemos suponer que x2 < x1 . Entonces 2x2 − 3x1 < 0, y por (3),
2y2 − 3y1 > 0. Como a y b son primos entre sı́, (3) también implica que
a | (2y2 − 3y1 ), y ası́ a ≤ 2y2 − 3y1 ≤ 2y2 . Concluimos que
ab ab
3n = ax2 + by2 ≥ by2 ≥ ⇒n≥
2 6
Esto prueba que si n es débil y n < ab
6
, entonces existe un entero k ≥ 2, tal
que kn es débil. (La desigualdad en el enunciado debe de ser estricta, como
muestra el ejemplo n = 1, a = 2, b = 3).

6. Sea R+ el conjunto de los números reales positivos. Halla todas las funciones
f : R+ → R+ , tales que f (x + f (y)) = yf (xy + 1), para todo x, y > 0.

Solución. Supongamos que exista un número real y tal que y − 1 y f (y) − 1


son ambos no nulos y del mismo signo. Entonces podemos tomar el número
real positivo
f (y) − 1
x=
y−1
de donde se obtienef (y) = x(y − 1) + 1 y x + f (y) = xy + 1. Por tanto,
f (x + f (y)) = f (xy + 1) y utilizando el enunciado, resulta
f (x + f (y))
y= =1
f (xy + 1)

6
Esto es una contradicción, y por tanto, y − 1 y f (y) − 1 no puden ser ambos no
nulos ni tener el mismo signo. Es decir, f (y) ≥ 1 para todo y < 1, y f (y) ≤ 1
para todo y > 1.

Para todo y > 1, tomemos el número real x = (y − 1)/y > 0. Entonces, se


tiene que  
y−1
f + f (y) = y f (y)
y
Ahora distinguimos dos casos:
y−1
• Si yf (y) > 1, entonces + f (y) ≤ 1, luego yf (y) ≤ 1, contradicción.
y
y−1
• Si yf (y) < 1, entonces + f (y) ≥ 1, luego yf (y) ≥ 1, contradicción.
y
1
Por tanto, yf (y) = 1 para todo y > 1 y f (y) = .
y
Sea ahora y ≤ 1, y tomemos x = 1. Se tiene entonces que tanto x + f (y) =
1 + f (y) como xy + 1 = y + 1 son números reales mayores que 1, con lo que
1 y
= ⇔ xy + 1 = xy + yf (y)
x + f (y) xy + 1
1
y concluı́mos que f (y) = también para todo y ≤ 1. Luego la única función
y
1
que puede satisfacer las condiciones del enunciado es f (y) = . Vemos además
y
que  
xy + 1 y
f (x + f (y)) = f = = yf (xy + 1),
y xy + 1
1
luego en efecto f (y) = es solución, y no puede haber otras.
y

7
LIII Olimpiada matemática Española (Concurso Final)

Enunciados y Soluciones

1. Determina el número de valores distintos de la expresión


n2 − 2
,
n2 − n + 2
donde n ∈ {1, 2, . . . , 100}.

Solución. Sumando y restando 2 − n al numerador se obtiene

n2 − 2 n2 − 2 − n + 2 + n − 2 n−4
an = 2
= 2
=1+ 2
n −n+2 n −n+2 n −n+2
Ahora vamos a ver si hay dos términos iguales, es decir, cuando es ap = aq
para p 6= q. Esto es equivalente a encontrar los enteros p 6= q para los que
p−4 q−4
= 2 ⇔ (p − q)(pq − 4p − 4q + 2) = 0
p2 −p+2 q −q+2
pq − 4p − 4q + 2 + 14 = 14 ⇔ (p − 4)(q − 4) = 14
De lo anterior se deduce que (p−4) | 14 y p−4 ∈ {±1, ±2, ±7, ±14}. Los valores
negativos no son posibles porque ambos p, q ≥ 1 con lo que p−4 ∈ {1, 2, 7, 14}.
Como (p − 4)(q − 4) = 14 entonces q − 4 ∈ {14, 7, 2, 1} de donde resultan
22
los pares (p, q) = (5, 18) y (p, q) = (6, 11) para los que a5 = 23 = a18 y
17
a6 = 16 = a11 . Finalmente, dado que todos los an son números racionals,
entonces entre los 100 primeros términos de la sucesión hay 98 que son distintos.

2. Un trazador de puntos medios es un instrumento que dibuja el punto medio


exacto de dos puntos previamente señalados. Partiendo de dos puntos a distan-
cia 1 y utilizando sólo el trazador de puntos medios, debes obtener dos puntos
1 1
a una distancia estrictamente comprendida entre y , trazando el
2017 2016
menor número posible de puntos. ¿Cuál es el mı́nimo número de veces que

1
necesitas utilizar el trazador de puntos medios, y qué estrategia seguirı́as para
lograr tu objetivo?

Solución. Sin pérdida de generalidad podemos trabajar con la recta real y


considerar que uno de los puntos iniciales es el 0 y el otro el 1. Es fácil com-
probar que después de k aplicaciones del trazador, todos los puntos hallados
n
son de la forma k con 0 ≤ n ≤ 2k siendo la fracción no necesariamente
2
irreducible. Esto lo probaremos por inducción. En efecto, para k = 1 se tienen
los puntos
0 1 1 2
0 = 1, = 1, 1 = 1
2 2 2 2
Supongamos que en el paso k los puntos son

0 1 r s 2k
0= , , · · · , , · · · , , · · · , 1 =
2k 2k 2k 2k 2k
Entonces, aplicando el trazador de nuevo, en el paso k + 1 los puntos obtenidos
son
0 1 1 r s  r+s 2k+1
0 = k+1 , k+1 , · · · , + = , · · · , 1 =
2 2 2 2k 2k 2k+1 2k+1

Ahora nos centramos en las distancias entre puntos. Utilizando el trazador


una vez, la distancia entre dos cualesquiera de los puntos obtenidos
0 1 1 2
0= , = 1, 1=
21 2 2 21
1
es un múltiplo de y aplicando el trazador k veces resulta que la distancia
21
1
entre dos cualesquiera puntos es un múltiplo de k . Es inmediato comprobar
2
64 66
que entre 17 y 17 se cumplen las desigualdades
2 2
64 1 65 1 66
17
< < 17 < < 17
2 2017 2 2016 2
1 1
Luego cualquier distancia estrictamente incluida entre 2017 y 2016
ha de tener
necesariamente en el denominador un exponente k ≥ 17.

Ahora vamos a ver que el número mı́nimo de veces que hay que utilizar el
trazador de puntos medios es k = 17 y describiremos una estrategia para
conseguirlo. En efecto, usando el trazador una vez se obtiene el punto 12 = 211 ,
luego lo utilizamos

2
de nuevo entre este punto y 1 para obtener el punto 34 = 232 , y a partir de aquı́
calculamos el punto medio entre el último punto hallado hasta ese momento,
1
y el punto , obteniendo sucesivamente los puntos
2
5 5 9 9 17 17 33 33
= 3, = 4, = 5, = 6,
8 2 16 2 32 2 64 2
65 65
y tras un total de 7 aplicaciones del trazador, se obtiene el punto = 7.
128 2
Aplicamos ahora 10 veces el trazador, para obtener el punto medio entre el
último punto hallado hasta ese momento y el punto 0, obteniéndose sucesiva-
mente los puntos
65 65 65 65
8
, 9 , 10 , · · · , 17
2 2 2 2
65 65
Este último punto dista 17 tanto de 0 como del anterior punto hallado 16 .
2 2
Luego en 17 pasos hemos conseguido nuestro objetivo, y hemos terminado.

1 1 1
3. Sean p un primo impar y Sq = + + ... + ,
2·3·4 5·6·7 q(q + 1)(q + 2)
3p − 5 1 m
donde q = . Escribimos − 2Sq en la forma , donde m y n son
2 p n
enteros. Demuestra que m ≡ n (mod p); es decir, que m y n dan el mismo
resto al ser divididos por p.

Solución. Se tiene que


2 (k + 2) − k 1 1
= = −
k(k + 1)(k + 2) k(k + 1)(k + 2) k(k + 1) (k + 1)(k + 2)
   
1 1 1 1 1 1 1 3
= − − − = + + −
k k+1 k+1 k+2 k k+1 k+2 k+1
con lo que
   
1 1 1 1 1 1 1 1 1
2 Sq = + + + ··· + + + −3 + + ··· +
2 3 4 q q+1 q+2 3 6 q+1

Antes de considerar el caso general veamos que ocurre con los primeros casos
particulares. Para p = 3 es q = 2 y se cumple el enunciado trivialmente. En
el caso en que p = q = 5, se tiene
   
2 2 1 1 1 1 1 1 1 1
2 S5 = + = + + + + + −3 +
2·3·4 5·6·7 2 3 4 5 6 7 3 6

3
y
   
n−m m 1 1 1 1 1 1 1 1 1
=1− = 1 + 2 S5 − = + + + = + + +
n n 5 3 4 6 7 3 7 4 6
2·5 2·5
= +
4·6 3·7
Como en los denominadores no aparece el primo 5 entonces n−m
n
es una fracción
cuyo numerador es múltiplo de 5 mientras que su denominador no lo es, lo que
permite concluir que n − m es múltiplo de 5 o n ≡ m (mod 5).
p−1
Notese que se han agrupado las p − 1 = 4 fracciones en 2
= 2 paréntesis
cuyos denominadores suman 2p = 10.

Generalizando lo anterior, resulta


n−m m 1
=1− = 1 + 2 Sq −
n n p
   
1 1 1 1 1 1 1 1 1 1
= 1+ + + + ··· + + + −3 + + ··· + −
2 3 4 q q+1 q+2 3 6 q+1 p
   
1 1 2 1 2 1
=1+ + + ··· + − 1 + + ··· + −
2 3 3p − 1 2 p−1 p
2 1 1 2
= + ··· + + + ··· +
p+1 p−1 p+1 3p − 1
| {z }
p−1 sumandos
!  
1 1 1 1
= p+1 + 3p−1 + ··· + +
2 2
p − 1 p+1
| {z }
p−1
2
paréntesis cuyos denominadores suman 2p
2p 2p
= + ··· +
( p+1
2
)( 3p−1
2
) (p − 1)(p + 1)
| {z }
p−1
2
términos
Al igual que en el caso particular, todos los factores que aparecen en los deno-
minadores son primos con p mientras que el numerador no lo es, lo que permite
concluir que n − m es múltiplo de p. Es dcecir, n ≡ m (mod p).

4. Se dispone de una fila de 2018 casillas, numeradas consecutivamente de 0


a 2017. Inicialmente, hay una ficha colocada en la casilla 0. Dos jugadores

4
A y B juegan alternativamente, empezando A, de la siguiente manera: En su
turno, cada jugador puede, o bien hacer avanzar la ficha 53 casillas, o bien
hacer retroceder la ficha 2 casillas, sin que en ningún caso se sobrepasen las
casillas 0 o 2017. Gana el jugador que coloque la ficha en la casilla 2017.
¿Cuál de ellos dispone de una estrategia ganadora, y cómo tendrı́a que jugar
para asegurarse ganar?

Solución. Vamos a probar que el jugador A tiene estrategia ganadora. Co-


mienza de la única forma posible: llevando la ficha hasta la casilla 53. A partir
de ahı́, durante 38 turnos dobles BA, el jugador A hará lo contrario de B: si B
avanza 53, A retrocede 2, y viceversa. De este modo, la ficha queda en la casilla
53 + 38 × 51 = 1991 y es turno de B. Los siguientes movimientos son forzados:
7 turnos dobles BA de restar. La ficha queda en la casilla 1991 − 14 × 2 = 1963
y es turno de B. Ahora:

1. Si B avanza 53, dejará la ficha en la casilla 2016 y tras 13 turnos dobles


AB, forzados, la ficha queda en la casilla 2016 − 26 × 2 = 1964 y A gana
sumando 53.

2. Si B resta 2, dejará la ficha en la casilla 1961. Entonces, A avanza 53


para dejarla en 2014. Tras 12 turnos dobles forzados BA, la ficha queda
en 2014 − 24 × 2 = 1966. Después, B está obligado a restar 2 hasta 1964
y, en su turno, A gana sumando 53.

5. Determina el máximo valor posible de la expresión


√ √ √
27abc + a a2 + 2bc + b b2 + 2ca + c c2 + 2ab,
1
siendo a, b, c, números reales positivos tales que a + b + c = √ .
3
1
Solución. En primer lugar se observa que cuando a = b = c = 3

3
el valor
2
que toma la expresión es √ , lo cual sugiere conjeturar que
3 3
√ √ √ 2
27abc + a a2 + 2bc + b b2 + 2ca + c c2 + 2ab ≤ √
3 3
2
Para probar la conjetura, se puede
2 2
√ √aplicar √ √ de√Cauchy, (~u ·~v ) ≤
√ la desigualdad
||~u|| ||~v || a los vectores ~u = ( a, b, c) y ~v = ( bc, ca, ab), obteniéndose
√ √ √ 2
9abc = abc + abc + abc ≤ (a + b + c) (bc + ca + ab)

5
Multiplicando por 3 ambos miembros de la desigualdad anterior y teniendo en
cuenta la restricción, resulta

27 abc ≤ 3 (a + b + c) (bc + ca + ab) = 3 (bc + ca + ab)
√ 1 p 2 2
Por otro lado, dado que a a2 + 2bc = √ 3a (a + 2bc), aplicando la desi-
3
gualdad entre las medias aritmética y geométrica a los números 3a2 y a2 + 2bc
se obtiene
√ 1 p 2 2 1 3a2 + (a2 + 2bc) 1
a a2 + 2bc = √ 3a (a + 2bc) ≤ √ = √ (2a2 + bc)
3 3 2 3
Análogamnete, se tiene que
√ 1
b b2 + 2ca ≤ √ (2b2 + ca)
3
y
√ 1
c c2 + 2ab ≤ √ (2c2 + ab)
3
Combinando las desigualdades anteriores, y teniendo en cuenta otra vez la
restricción, se obtiene
√ √ √
27abc + a a2 + 2bc + b b2 + 2ca + c c2 + 2ab
√ 1
3 (bc + ca + ab) + √ (2a2 + bc + 2b2 + ca + 2c2 + ab)

3
2 2 2
= √ (a2 + b2 + c2 + 2ab + 2bc + 2ca) = √ (a + b + c)2 = √
3 3 3 3
2
Esto prueba la conjetura y el máximo de la expresión es √ .
3 3

6. En el triángulo ABC los puntos medios respectivos de los lados BC, CA


y AB son D, F, E. Sean: M el punto donde la bisectriz interior del ángulo
∠ADB corta al lado AB, y N el punto donde la bisectriz interior del ángulo
∠ADC corta al lado AC. Sean ademas O el punto de intersección de las
rectas AD y M N , P el punto de intersección de AB y F O, y R el punto de
intersección de AC y EO. Demuestra que P R = AD.

Solución. Aplicando el teorema la bisectriz a los triángulos ADB y ADC se


obtiene
MB BD NC DC
= y =
AM AD AN AD
6
A

E F
M N
O R
P

B D C

MB NC
Como BD = DC, = y entonces M N es paralelo a BC, de donde
AM AN
AB BC MB BD
= . De aquı́, junto con = , resulta
AM MN AM AD
BD + AD BD MB M B + AM AB BC
=1+ =1+ = = = (1)
AD AD AM AM AM MN
Como EF es la paralela media de ABC, EF y BC son paralelas, BD = EF
y BC = 2 · EF . Sustituyendo esto en (1) obtenemos

BD + AD BC BD + AD 2 · EF 2 BD + AD
= ⇔ = ⇔ =
AD MN AD MN MN AD · EF
que lo podemos escribir como
2 1 1
= + (2)
MN AD EF
Como M N y EF son paralelas a BC, son paraleleas entre sı́, y entonces
AE AF
= (3)
EM AN
Aplicando dos veces el Teorema de Menelao al triángulo AM N , una con la
transversal ER y otra con la transversal P F , se obtiene
AR AE M O AP AF M O
= · y = · ,
NR EM ON MP F N ON

7
AP AR
ası́ que teniendo en cuenta (3) obtenemos = lo cual es equivalente a
MP NR
que P R es paralelo a M N . Pero M N es paralelo a EF y el cuadrilátero EP RF
es un trapecio. Ya que M N es paralela a las bases por el punto de intersección
de las diagonales, M N es la media armónica de las bases del trapecio, es decir
1 1 2 1 1
+ = y por (2), =
P R EF MN PR AD
de donde se concluye que P R = AD.

8
Problema 1.

Se tienen dos progresiones de números reales, una aritmética (an )n∈N y otra geométrica (gn )n∈N no
constante. Se cumple que a1 = g1 = 0, a2 = g2 y a10 = g3 . Decidir, razonadamente, si para cada entero
positivo p, existe un entero positivo m, tal que gp = am .

Solución.
Sean d y r = 1 la diferencia y la razón, respectivamente, de las progresiones aritmética (an )n∈N y
geométrica (gn )n∈N . En primer lugar tenemos g1 r = g2 = a2 = a1 + d = g1 + d de donde d = g1 (r − 1).
En segundo lugar g1 r2 = g3 = a10 = a1 + 9d = g1 + 9g1 (r − 1). De aquı́ sale r2 − 9r + 8 = 0 puesto
que g1 = 0. Las soluciones son r = 1 (que debemos descartar ya que la progresión geométrica no es
constante) y r = 8 que es la razón buscada. De aquı́ también resulta d = 7g1 .
Sea p un entero positivo cualquiera. Debemos encontrar un m tal que gp = am , es decir gp = g1 8p−1 =
am = a1 + (m − 1)d = g1 + (m − 1)7g1 que es equivalente a 8p−1 + 6 = 7m. Puesto que las potencias de
8 módulo 7 siempre son 1, resulta que 8p−1 + 6 es siempre múltiplo de 7 y siempre podremos encontrar
8p−1 + 6
m= .
7

Problema 2.

Sea p un número primo positivo dado. Demostrar que existe un entero α tal que α(α − 1) + 3 es divisible
por p si y sólo si existe un entero β tal que β(β − 1) + 25 es divisible por p.

Solución.

Sean f (x) = x(x − 1) + 3 = x2 − x + 3, g(x) = x(x − 1) + 25 = x2 − x + 25.


Caso p = 2. No podemos encontrar ni un tal α ni un tal β porque para cualesquiera α y β enteros, f (α)
y g(β) son impares, es decir, no múltiplos de p simultáneamente, y por lo tanto el enunciado se cumple.
Caso p = 3. Ahora f (1) = 3, g(2) = 27 y el enunciado también se cumple.
Caso p ≥ 5. Decir que p divide a f (α) es lo mismo que decir que f (α) ≡ 0 mod p. En adelante
seguiremos con esta notación de congruencias sobreentendiendo el módulo p. El enunciado es equivalente
a ver que las congruencias f (x) = x2 − x + 3 ≡ 0 y g(x) = x2 − x + 25 ≡ 0 tengan o no tengan solución
simultáneamente.
Puesto que 2 no es congruente con p, se puede dividir por 2 módulo p. Tenemos
 √
1 2 11 1 ± −11
x −x+3 ≡ x−
2
+ ≡ 0 ⇐⇒ x ≡ .
2 4 2
Análogamente √
 1 2 99 1 ± 3 −11
x − x + 25 ≡ x −
2
+ ≡ 0 ⇐⇒ x ≡ .
2 4 2
En consecuencia las congruencias f (x) ≡ 0 y g(x) ≡ 0 tienen o no solución (a la vez) según que −11 sea
cuadrado perfecto módulo p o no lo sea.
Observación. Recordemos que esto se cumplirá según que
p−1 p−1
(−11) 2 ≡ 1 o (−11) 2 ≡ −1.

Problema 3.

En la circunferencia circunscrita al triángulo ABC, sea A1 el punto diametralmente opuesto al vértice


A. Sea A el punto en el que la recta AA1 corta al lado BC. La perpendicular a la recta AA trazada
por A corta a los lados AB y AC (o a sus prolongaciones) en M y N , respectivamente. Demostrar que
los puntos A, M , A1 y N están en una circunferencia cuyo centro se encuentra en la altura desde A en
el triángulo ABC.

Primera solución.
Sea O el circuncentro de ABC, y D el pie de la altura desde A. Es conocido que AO y la altura desde
A son rectas isogonales en cualquier triángulo. En nuestro caso, los son en los dos triángulos ABC y
AM N , por la manera como se construye el triángulo AM N .
En ABC, AA1 es diámetro de la circunferencia circunscrita Γ y la recta AD es altura. En AM N , AO
es altura, ası́ que el centro de la circunferencia circunscrita Γ estará en la recta AD (isogonal de AO en
AM N ).
Para terminar el problema hay que probar que A1 pertenece a Γ . En primer lugar, es fácil demostrar que
los triángulos ABC y AM N son semejantes (AM  N = 90◦ − α = Ĉ). Escribiendo la proporcionalidad
entre sus lados, obtenemos,
AB AN
= ,
AC AM
y esto quiere decir que las rectas M C y BN son antiparalelas y el cuadrilátero BM CN es cı́clico. Sea
Γ la circunferencia que pasa por los vértices de dicho cuadrilátero.
El eje radical de Γ y Γ es BC; el de Γ y Γ es M N . La intersección de las rectas BC y M N , es decir,
A , es el centro radical de las tres circunferencias. El eje radical de Γ y Γ debe, necesariamente, pasar
por A y A , luego no puede ser otro que AA y por lo tanto A1 estará en la circunferencia Γ .
Segunda solución.
Sean Γ y Γ las circunferencias circunscritas a ABC y AM N , respectivamente y AA1 un diámetro de Γ
(por construcción). El triángulo ABA1 es rectángulo en B y su ángulo en A1 es Ĉ por ver, desde A1 la
cuerda AB de Γ. En consecuencia, los triángulos
ABA1 y ADC son semejantes
y los ángulos marcados en A son iguales, α = 90◦ − Ĉ.
El segmento M A1 se ve desde A y desde B bajo ángulo recto (el primero por construcción y el segundo
por inscrito que ve un diámetro). Esto nos dice que los cuatro puntos B, M, A , A1 son concı́clicos. Y
esto nos lleva a que γ = δ. De forma similar, el segmento A1 N se ve desde A y C bajo ángulo recto y
los puntos A1 , A , C, N son concı́clicos. De ahı́ que γ  = δ = γ.

Se tiene que BA ◦
1 C = 180 − Â por ser inscrito en Γ y ver la cuerda BC desde el arco contrario al vértice
A. Tenemos que M  
A1 N = BA  ◦
1 C + γ − γ = 180 − Â. Esto nos indica que A1 tiene que estar sobre la

circunferencia Γ en el arco opuesto al del vértice A.
Como que el triángulo AA M es rectángulo por construcción, resulta que AM  A = Ĉ. Los puntos M y
P de Γ ven sobre esta circunferencia la cuerda AN . Luego AP  N = Ĉ. En el triángulo AP N los ángulos
◦ 
en A y en P suman 90 , luego AN P = 90 y AP es un diámetro de Γ .

Problema 4.

Sean m ≥ 1 un entero positivo, a y b enteros positivos distintos mayores estrictamente que m2 y menores
estrictamente que m2 + m. Hallar todos los enteros d, que dividen al producto ab y cumplen m2 < d <
m2 + m.

Solución.
Sea d un entero positivo que divida a ab y tal que d ∈ (m2 , m2 + m). Entonces
 d dividea (a − d)(b − d) =
ab − da − db + d2 . Como que |a − d| < m y |b − d| < m, deducimos que (a − d)(b − d) < m2 < d lo que
implica que (a − d)(b − d) = 0. Ası́ d = a o d = b.
Problema 5.

De entre todas las permutaciones (a1 , a2 , . . . , an ) del conjunto {1, 2, ., n}, (n ≥ 1 entero), se consideran
las que cumplen que 2(a1 + a2 + · · · + am ) es divisible por m, para cada m = 1, 2, . . . , n. Calcular el
número total de estas permutaciones.

Solución.

Sea Pn el conjunto de permutaciones de {1, 2, . . . , n} que cumplen las condiciones del enunciado. El
problema consiste en calcular |Pn |. Observemos que, para cualquier n, las condiciones se cumplen
siempre para m = 1, para m = 2 y para m = n, de manera que P1 , P2 y P3 son, en cada caso, el
conjunto de todas las permutaciones y |P1 | = 1, |P2 | = 2 y |P3 | = 6.
Supongamos que (a1 , . . . , an ) ∈ Pn . Tomando m = n − 1, debe cumplirse que
(n − 1)|2(a1 + · · · + an−1 ) = 2(a1 + · · · + an ) − 2an = n(n + 1) − 2an . Mirando esta relación en
forma de congruencias, tenemos 2 − 2an ≡ 0 mod (n − 1), o bien que 2(an − 1) es múltiplo de n − 1, que
es equivalente a que an − 1 sea múltiplo de n−1
2 . Dada la acotación obvia an − 1 ≤ n − 1, resulta que los
únicos valores que puede tomar an − 1 son 0, n−12 o n − 1. Entonces an solamente puede ser 1, 2 o n.
n+1

Si fuese an = n+1 2 , entonces n deberı́a ser impar. La propiedad de Pn para m = n − 2 nos dice,
con un cálculo parecido al hecho antes, que (n − 2)|2(a1 + · · · + an−2 ) = n(n + 1) − 2an−1 − 2an =
(n−1)(n+1)−2an−1 . Mirando esta relación en forma de congruencias módulo (n−2) queda 3−2an−1 ≡ 0
mod (n − 2), de manera que 2an−1 − 3 tiene que ser múltiplo de n − 2 y esto solo sucede si es n − 1.
Pero esto conduce a que an−1 = n+12 = an , que es absurdo.
En conclusión, an solamente puede tomar los valores 1 y n.
Estudiemos estos dos casos.
Caso an = n. Entonces (a1 , . . . , an−1 ) es una permutación de {1, 2, . . . , n − 1}. Se comprueba fácilmente
que es de Pn−1 . Entonces habrá tantas permutaciones de Pn con an = n como permutaciones en Pn−1 .
Caso an = 1. Ahora a1 , a2 , . . . , an−1 > 1 y (a1 − 1, a2 − 1, . . . , an−1 − 1) es una permutación de Pn−1 .
La correspondencia (a1 , a2 , . . . , an−1 , 1)  (a1 − 1, a2 − 1, . . . , an−1 − 1) es biyectiva. Habrá tantas
permutaciones de Pn con an = 1 como permutaciones en Pn−1 .
En definitiva, |Pn | = 2|Pn−1 | si n > 3, de donde, |Pn | = 3 · 2n−2 .
Observación: La demostración anterior nos da el algoritmo recurrente para obtener todas las permuta-
ciones que cumplen la condición. Por ejemplo, conocemos que las permutaciones de P3 son todas.
Añadiendo un 4 al final de cada una obtenemos la mitad de las de P4 . La otra mitad sale de sumar 1 a
cada elemento de cada permutación y añadir un 1 al final.
Problema 6.

Sea n ≥ 2 un número entero. Determinar el menor número real positivo γ de modo que para cua-
lesquiera números reales positivos x1 , x2 , . . . , xn y cualesquiera números reales y1 , y2 , . . . , yn con 0 ≤
y1 , y2 , . . . , yn ≤ 12 que cumplan x1 + x2 + . . . + xn = y1 + y2 + . . . + yn = 1, se tiene que
x1 x2 . . . xn ≤ γ (x1 y1 + x2 y2 + . . . + xn yn ),

Solución.
M
Sean M = x1 x2 . . . xn y Xi = para 1 ≤ i ≤ n. Consideremos la función
xi
M
ϕ : (0, +∞) → R definida por ϕ(t) = que es convexa como se prueba fácilmente. Como los números
t
no negativos yi , (1 ≤ i ≤ n), son tales que y1 + y2 + . . . + yn = 1, entonces aplicando la desigualdad de
Jensen a la función ϕ se tiene  n 
  n
ϕ yi xi ≤ yi ϕ(xi ),
i=1 i=1
es decir,
 −1

n 
n
M  n
M yi xi ≤ yi = yi Xi . (1)
i=1 i=1
xi i=1

Ahora se trata de encontrar la menor cota superior del término de la derecha de (1). Sin pérdida de
generalidad, podemos suponer que x1 ≤ x2 , ≤ . . . ≤ xn e y1 ≥ y2 ≥ . . . ≥ yn . Entonces se tiene que
X1 ≥ X2 ≥ . . . ≥ Xn como se comprueba inmediatamente. Aplicando la desigualdad del reordenamiento,

n
sabemos que entre todas las sumas de la forma yi Xi la que alcanza el valor máximo es la que se obtiene
i=1
cuando y1 ≥ y2 ≥ . . . ≥ yn y X1 ≥ X2 ≥ . . . ≥ Xn .

Ahora observamos que



n
yi Xi = y1 X1 + (y2 X2 + . . . + yn Xn ) ≤ y1 X1 + (y2 + . . . + yn )X2 = y1 X1 + (1 − y1 )X2 .
i=1

Al ser 0 ≤ y1 ≤ 1/2, se tiene que


 1 
n
1
yi Xi ≤ (X1 + X2 ) = (x1 + x2 )x3 . . . xn ≤
i=1
2 2
 n−1  n−1
1 (x1 + x2 ) + x3 + . . . + xn 1 1
≤ =
2 n−1 2 n−1
donde se ha utilizado la desigualdad entre las medias aritmética y geométrica y la condición x1 + x2 +
. . . + xn = 1. De lo anterior y (1), resulta
 n  n    n 
  1 1
n−1 
M≤ yi xi yi Xi ≤ yi xi
i=1 i=1
2 n−1 i=1
y
 n−1
1 1
γ≤ .
2 n−1
1 1 1
Si tomamos x1 = x2 = , x3 = x4 = . . . xn = e y1 = y2 = , y3 = y4 = . . . = yn = 0,
2(n − 1) n−1 2
entonces
 n
1 1
M = x1 x2 . . . xn = =
4 n−1
 n−1  n−1 
n
1 1 1 1
= (y1 x1 + y2 x2 ) = yi xi
2 n−1 2 n−1 i=1
 n  n−1  n−1 
n
1 1 1 1 1 1
M = x1 x2 . . . xn = = (y1 x1 + y2 x2 ) = yi xi
4 n−1 2 n−1 2 n−1 i=1

y se concluye que
 n−1
1 1
γ= .
2 n−1
LI Olimpiada matemática Española (Concurso Final)

Enunciados y Soluciones
1. Sobre la gráfica de una función polinómica con coeficientes enteros, se eligen dos
puntos con coordenadas enteras. Probar que si la distancia entre ellos es un número
entero, entonces el segmento que los une es paralelo al eje de abscisas.
n
X
Solución. Sea el polinomio f (x) = ai xi , ai ∈ Z y sean A(c, f (c)) y B(d, f (d))
i=0
dos puntos con coordenadas enteras. Entonces
n
X
f (c) − f (d) = ai (ci − di )
i=1

Todos los sumandos de esta suma son divisibles por c − d, ası́ que
n
X
f (c) − f (d) = ai (ci − di ) = k(c − d),
i=1

donde k es un entero. Como la distancia entre los puntos A y B, que denotamos


por d(A, B) es un entero, entonces d2 (A, B) es un cuadrado perfecto. Pero

d2 (A, B) = (c − d)2 + k 2 (c − d)2 = (c − d)2 (1 + k 2 )

luego la única posibilidad para que esta expresión sea un cuadrado perfecto es que
k = 0, en cuyo caso
f (c) − f (d) = 0
y efectivamente el segmento AB es paralelo al eje de abcisas.

2. En el triángulo ABC, sea A0 el punto simétrico de A respecto del circuncentro


O de ABC.
a) Probar que la suma de los cuadrados de los segmentos de tangentes trazadas
desde A y A0 a la circunferencia inscrita en ABC es igual a 4R2 − 4Rr − 2r2 .
b) Sea I el incentro del 4ABC. Probar que la circunferencia de centro A0 y radio
A0 I corta
√ a la circunferencia circunscrita de ABC en un punto L tal que
AL = AB.AC.

1
Solución. a) Si t y t0 son las longitudes de los segmentos de tangentes, se tiene

t2 = AI 2 − r2 y t02 = A0 I 2 − r2 ;

por lo tanto,

t
M
I O

M'
B C
L t'
A'

t2 + t02 = AI 2 + A0 I 2 − 2r2
Aplicando el teorema de Apolonio al triángulo AIA0 con la mediana IO resulta

AI 2 + A0 I 2 = 2IO2 + 2R2 = 2(R2 − 2Rr) + 2R2 = 4R2 − 4Rr

Sustituyendo en la expresión anterior resulta la relación pedida.


b) Teniendo en cuenta a) y que AL2 = A0 A2 −A0 L2 = 4R2 −A0 I 2 , ya que el triángulo
AA0 L es claramente rectángulo por ser AA0 un diámetro, resulta

abc bc(p − a)
AL2 = 4R2 − (4R2 − 4Rr − AI 2 ) = 4Rr + AI 2 = + = bc,
p p
donde las dos últimas expresiones se obtienen a partir de las relaciones entre ángulos
y lados en el triángulo, teniendo en cuenta que t = p − a siendo p el semiperı́metro.

3. En la pizarra está escrito un número entero. Dos jugadores A y B juegan


alternadamente, empezando por A. Cada jugador en su turno reemplaza el número
existente por el que resulte de realizar una de estas dos operaciones: restar 1 o dividir
entre 2, siempre que se obtenga un entero positivo. El jugador que llegue al número
1 gana. Determinar razonadamente el menor número par que le exige a A jugar al
menos 2015 veces para ganar (no se contabilizan los turnos de B).

2
Solución. Si el valor de N inicial es par, veamos que gana A: ya sea restando 1
o bien dividiendo entre 2 (en el caso N = 4k + 2, N2 = 2k + 1 impar), A siempre
le dejará a B un impar, obligándolo a restar 1 por no poder dividir entre 2, con lo
cual al jugar A volverá a encontrarse con un par, menor que el anterior. Ası́, A se
encontrará finalmente con un 2, y ganará.
Hay que destacar que cuando A tiene dos opciones válidas para dejarle a B un impar,
siempre preferirá dividir entre 2, para acercarse más rápidamente al objetivo.
A continuación usaremos el siguiente resultado:
Sea y un número par con el cual se encuentra A en su turno. Entonces, dos turnos
antes A se encontraba con un número mayor o igual que 2y + 4.
En efecto, distinguiremos dos casos: (1) Caso y = 4k + 2. Proviene de una jugada
obligada de B desde 4k + 3. Antes, A podı́a estar en 8k + 6, o bien 4k + 4 (esto
es viable, ya que desde 4k + 4 A no puede dividir porque 2k + 2 es par). Es decir,
B estaba antes en 8k + 7 o en 4k + 5. Si B estaba en 8k + 7, A pudo estar antes
en 16k + 14 o en 8k + 8, mientras que si B estaba en 4k + 5 A sólo pudo estar en
8k + 10, no en 4k + 6, porque en ese caso habrı́a preferido dividir en vez de restar.
Resumiendo, si y = 4k + 2, hace dos turnos A podı́a encontrarse en 16k + 14, 8k + 10
o 8k + 8, siendo la menor opción justamente 2y + 4.
(2) Caso y = 4k. Razonando de forma similar, se deduce que hace dos turnos A se
encontraba en 8k + 4 (2y + 4) o
La solución al problema se obtendrá aplicando repetidamente el resultado anterior.
Definimos la sucesión a0 = 2, an+1 = 2an + 4, cuya fórmula explı́cita es

an = 6 · 2n − 4

Aplicando reiteradamente el resultado, vemos que para todo n se tiene que cualquier
N desde el cual se llegue al 2 en 2n turnos, debe cumplir N ≥ an . Es decir, contando
con que A pasa del 2 al 1 en un turno, hemos probado que para todo n, los números
N que le exigen a A al menos 2n + 1 jugadas cumplen N ≥ an .
Dado que para n = 1007 se tiene 2n + 1 = 2015, se cumple que N es mayor o igual
que el término a1007 de la sucesión considerada, es decir, el número 6 · 21007 − 4.
Queda por probar que se alcanza la igualdad. Para ello, basta ver que para todo
n ≥ 1, si A se encuentra con an , tras dos turnos estará en an−1 . En efecto,

n A B A B
6| · 2{z − 4} → 6 · 2n − 5 → 6 · 2n − 6 → 3 · 2n − 3 → 3 · 2n − 4 = |6 · 2n−1
{z − 4}
an an−1

4. Todas las caras de un poliedro son triángulos. A cada uno de los vértices de este
poliedro se le asigna de forma independiente uno de entre tres colores: verde, blanco

3
o negro. Decimos que una cara es extremeña si sus tres vértices son de distintos
colores, uno verde, uno blanco y uno negro. Es cierto que, independientemente de
cómo coloreemos los vértices, el número de caras extremeñas de este poliedro es
siempre par?

Solución. Sea C el número de caras del poliedro. Cada cara tiene 3 lados, cada uno
de los cuáles pertenece exactamente a dos caras. Luego el número total de aristas
del poliedro es 3C/2, que ha de ser entero. Luego el número C de caras del poliedro
es par.
A una arista cuyos vértices extremos son del mismo color la llamaremos monocroma.
Si sumamos las aristas monocromas de todas las caras, como cada una de ellas está
exactamente en dos caras, tendremos un número par. A este número no contribuyen
las caras extremeñas, pues no contienen aristas monocromas, y las no extremeñas
lo hacen con un número impar: 3, si los tres vértices son del mismo color, o 1 en
otro caso. Por tanto, el número de caras no extremeñas tiene que ser par. Como el
número total de caras es par, también será par el número de caras extremeñas.

5. Sean p y n enteros positivos, tales que p es primo, n ≥ p, y 1+np es un cuadrado


perfecto. Probar que n + 1 es suma de p cuadrados perfectos no nulos.

Solución. Sea 1 + np = k 2 , con k entero positivo. Entonces np = k 2 − 1 =


(k − 1)(k + 1). Ahora consideramos dos casos:
1. Si el primo p divide a k − 1, entonces k − 1 = p` y k = p` + 1, con ` entero
positivo. Por tanto

1 + np = k 2 = (p` + 1)2 = p2 `2 + 2p` + 1 ⇔ np = p2 `2 + 2p` ⇔ n = p`2 + 2`

Entonces, n + 1 = p`2 + 2` + 1 = (p − 1)`2 + (` + 1)2 como querı́amos probar.


2. Si el primo p divide a k + 1, entonces k + 1 = p` y k = p` − 1, con ` > 1 entero
(` = 1 se corresponde con el caso n = p − 2, que no es posible). Por tanto

1 + np = k 2 = (p` − 1)2 = p2 `2 − 2p` + 1 ⇔ np = p2 `2 − 2p` ⇔ n = p`2 − 2`

Entonces, n + 1 = p`2 − 2` + 1 = (p − 1)`2 + (` − 1)2 es suma de p cuadrados


perfectos.

6. Sean M y N puntos del lado BC del triángulo ABC tales que BM = CN ,


estando M en el interior del segmento BN . Sean P, Q puntos que están respectiva-
mente en los segmentos AN, AM tales que ∠P M C = ∠M AB y ∠QN B = ∠N AC.
¿Es cierto que ∠QBC = ∠P CB?

Solución. La idea clave de la solución es considerar las circunferencias circunscritas


de los triángulos BN Q (en verde en la figura) y P M C (en rojo). Si AM corta a

4
la circunferencia (BN Q) en X, y AN corta a la circunferencia (P M C) en Y , es
evidente que los cuadriláteros BQN X y M P CY son cı́licos. Puesto que ∠QBC =

A
α β
P
Q
B α
β
M C
N

∠QBN y ∠P CB = ∠P CM, los ángulos del enunciado del problema serán iguales
si lo son ∠QBC y ∠P CM. Pero ∠QBN = ∠QXN = ∠M XN y por otra parte
∠P CM = ∠P Y M = ∠N Y M. Entonces el problema estará resuelto en sentido
afirmativo si demostramos que ∠M XN = ∠N Y M lo cual es tanto como decir que
los cuatro puntos M, N, Y, X están en una circunferencia, para lo cual podemos
intentar demostrar que
AM AY
AM · AX = AN · AY ⇔ =
AN AX
Para ello razonaremos de la siguiente manera:
Los triángulos ABM y ACN tienen la misma área, ya que sus bases son iguales por
hipótesis y la altura desde A es la misma; entonces
AM · AB · sin α = AN · AC · sin β, (1)
donde α = ∠M AB y β = ∠N AC. Por otra parte, dos de los ángulos del triángulo
ABX son α y ∠BXQ = ∠QN B = β (en la circunferencia (BNQ)).
Análogamente, dos de los ángulos del triángulo ACY son β y α. Por lo tanto esos dos
triángulos son semejantes, y escribiendo la proporcionalidad entre lados homólogos,
se tiene
AY CY
= (2)
AX AB

5
Por último, aplicando el teorema del seno en el triángulo ACY , tenemos
AC CY sin β CY
= ⇔ =
sin α sin β sin α AC

Utilizando (1) y teniendo en cuenta (2), resulta

AM AC · sin β AC CY AY
= = · =
AN AB · sin α AB AC AX
y esto demuestra la igualdad de ángulos indicada en el enunciado

6
L Olimpiada matemática Española (Concurso Final)

Enunciados y Soluciones
1. Es posible disponer sobre una circunferencia los números 0, 1, 2, . . . , 9 de tal
manera que la suma de tres números sucesivos cualesquiera sea, como mucho a) 13,
b) 14, c) 15?

Solución. Cortamos la circunferencia a la izquierda del 0. Descartado el 0, nos


quedan 9 números, que podemos agrupar en tres trı́os cuya suma total es

1 + 2 + . . . + 9 = 45

Por lo tanto, no puede suceder que las tres sumen menos de 15. Luego la respuesta
a los apartados a) y b) es NO, mientras que al apartado c) es SÍ, colocando por
ejemplo los números en el siguiente orden:

0, 9, 5, 1, 8, 4, 3, 2, 7, 6
| {z } | {z } | {z }

1 1 1
2. Dados los números racionales r, q y n, tales que + = , probar
r r + qn q + rn r + q
n−3
que es un número racional.
n+1
Solución. Racionalizando, tenemos
r s p
n−3 (n − 3)(n + 1) (n − 1)2 − 4
= =
n+1 (n + 1)2 n+1

Por tanto, el enunciado quedará probado si vemos que (n − 1)2 − 4 es el cuadrado de


un número racional. Para ello, utilizamos la condición y la escribimos en la forma
equivalente

(r + qn)(r + q) + (q + rn)(r + q) = (r + qn)(q + rn) ⇔ (r + q)2 = rq(n − 1)2

(r + q)2
de donde resulta (n − 1)2 = . Entonces, se tiene que
rq
2
(r + q)2 (r − q)2 (r − q)2 (n − 1)2

2 (r − q)(n − 1)
(n − 1) − 4 = −4= = =
rq rq (r + q)2 (r + q)

(r + q)2
(En la penúltima igualdad se ha utilizado que rq = ). Por tanto, (n−1)2 −4
(n − 1)2
es el cuadrado de un número racional y hemos terminado.

1
3. Sean B y C dos puntos fijos de una circunferencia de centro O, que no sean dia-
metralmente opuestos. Sea A un punto variable sobre la circunferencia, distinto de
B y C, y que no pertenece a la mediatriz de BC. Sean H, el ortocentro del triángulo
ABC; y M y N los puntos medios de los segmentos BC y AH, respectivamente.
La recta AM corta de nuevo a la circunferencia en D, y, finalmente, N M y OD
se cortan en un punto P . Determinar el lugar geométrico del punto P cuando A
recorre la circunferencia.

Solución. Empezaremos considerando el caso en que 4ABC es acutángulo. En


primer lugar, denotaremos por A0 el punto diametralmente opuesto a A con lo que los
triángulos ACA0 y ABA0 son rectángulos. Los segmentos HB y CA0 son paralelos
por ser perpendiculares a AC. Igualmente, HC y BA0 también son paralelos por ser
perpendiculares a AB. Figura-1:
Figura-1: Caso en que ABCCaso
sea en que ABC sea acutángulo.
acutángulo.

A A A A

N N N N

H H H O H O
O O

C C B C B C M B B
M M M
P P

A' A' D D
Figura-1a: Figura-1a: Figura-1b Figura-1b

Triángulo ABC acutángulo


Análogamente seAnálogamente se razonaría en el:
razonaría en el:

Entonces, CHBA0 es un paralelogramo Figura-2:


Figura-2: Caso en que ABCCaso
sea en
tanto, M es el unto medio de HA0 .
que ABC sea obtusángulo.
y, obtusángulo.
por
Los triángulos AA 0 0
H H y OA
H M son semejantes H conHrazón de semejanza conocida. Es
N N
decir, tenemos que
N N
A A
A A

OM M A0 1 AH
C
= 0
= ⇒C OM B= C = AN =BN H B
C AH HA
M 2M B 2 M M
O O O
O
Luego OM HN es otro paralelogramo (figura izquierda).
P P
D D

Sea D la intersección de AM con A'


la circunferencia
A'
y sea P el punto de corte de
AD con N M . Puesto que 4AOD es isósceles, entonces ∠AOD = ∠ODA. Como
OM y AN son paralelos, pues Figura-2a:
Figura-2a: ambos son perpendiculares
Figura-2b al lado BC, además de
Figura-2b

iguales, entonces AOM N es también un paralelogramo. Y, de aquı́, tenemos que


∠OAM = ∠AM N = ∠P M D por ser opuesto por el vértice. Sintetizando, tenemos
que ∠P M D = ∠OAM = ∠OAD7 = ∠ODA 7 = ∠P DM con lo que 4P DM es
isósceles y, por tanto, P M = P D.

Finalmente, tenemos que

OP + P M = OP + P D = OD = r = constante

Es decir, con A variable, el punto P se mueve sobre una elipse incompleta con
focos en O y M , y eje mayor el radio de la circunferencia. En esta elipse hay que
descartar los cuatro vértices. En efecto, si el punto P estuviese sobre el eje mayor
de la elipse, también estarı́a D y por tanto A, lo cual está excluido del enunciado ya
que en este caso AD y N M son coincidentes. Si el punto P fuese uno de los vértices
del eje menor de la elipse, tendrı́amos OP = P M = r/2. Como OD = r, entonces

2
Figura-1: Caso enFigura-1:
que ABCCaso
sea acutángulo.
en que ABC sea acutángulo.

A A A
A

N N N
N

H H O H
O O H O
P D = r/2. Supongamos que P está del lado de B, entonces la paralela a BM por el
punto medio de OB es el eje menor BdeCla elipse, con lo que elB punto medio de OB es
C CM B CM B
precisamente P y D coincide conMB, lo que implicará P
queMA coincide con C, lo cual
P
está escluido del enunciado ya que ABC serı́a degenerado. El resto de puntos de la
A'
elipse se pueden obtener cuando A es A'distinto de BDy C y noD está en la mediatriz de
Figura-1a: Figura-1b
BC. Figura-1a: Figura-1b

Análogamente se razonaría en el:


A continuación apareceAnálogamente
la Figura-2:
figura para el caso en que 4ABC sea obtusángulo.
se razonaría en el:
Caso en que ABC sea obtusángulo.
Figura-2: Caso en que ABC sea obtusángulo.

H H H
H
N N N
N A
A A A

C C C B
C B B M B
M M M
O O O
O
P P
D D

A' A'

Figura-2a: Figura-2a: Figura-2b Figura-2b


Triángulo ABC obtusángulo

7
7
4. Sea {xn }n≥1 la sucesión de enteros positivos definida por x1 = 2 y xn+1 =
2x3n + xn para todo n ≥ 1. Determinar la mayor potencia de 5 que divide al número
x22014 + 1.

Solución. Para n = 1 se tiene que x1 = 2 y x21 + 1 = 22 + 1 = 5 que es múltiplo de


5 pero no de 52 . Para n = 2 es x2 = 2 · 23 + 2 = 18 y x22 + 1 = 182 + 1 = 325 que
es múltiplo de 52 pero no de 53 . Esto nos sugiere conjeturar que x2n + 1 es múltiplo
de 5n pero no de 5n+1 . La demostraremos por inducción. Los casos n = 1 y n = 2
ya están probados. Supongamos que n ≥ 3 y que x2n + 1 = k 5n con (k, 5) = 1.
Entonces,

x2n+1 + 1 = (2x3n + xn )2 + 1 = x2n (2x2n + 1)2 + 1 = (k5n − 1)(2k5n − 1)2 + 1


= (k5n − 1)(4k 2 52n − 4k5n + 1) + 1 = 4k 3 53n − 8k 2 52n + k5n+1
 
= k5n+1 |4k 2 52n−1{z− 8k5n−1} +1

que es múltiplo de 5n+1 pero no es múltiplo de 5n+2 ya que para n ≥ 3 es 2n − 1 ≥ 1


y n − 1 ≥ 1 respectivamente y el término entre paréntesis no es múltiplo de 5. Esto
prueba la conjetura y, por tanto, la mayor potencia de 5 que divide a x22014 + 1 es
52014 .

5. El conjunto M está formado por números enteros de la forma a2 + 13b2 , con a


y b enteros distinos de cero.

• Demostrar que el producto de dos elementos cualesquiera de M es un elemento


de M .

• Determinar, razonadamente, si existen infinitos pares de enteros (x, y) tales


que x + y no pertenece a M , pero x13 + y 13 sı́ pertenece a M .

3
Solución. (1) Sean 2 2 2 2
√ √ a + 13b y c + 13d dos elementos de M . Poniendo m = a, n =
13 b, p = c, q = 13 d en la identitdad algebraica

(m2 + n2 ) (p2 + q 2 ) = (mq − np)2 + (mp + nq)2 (Euler)

se obtiene
√ √
(a2 +13b2 ) (c2 +13d2 ) = ( 13ad− 13bc)2 +(ac+13bd)2 = (ac+13bd)2 +13(ad−bc)2 ∈ M

(2) Puesto que los cuadrados de los enteros son congruentes con 0 o con 1 módulo
4, entonces los elementos de M no son congruentes con 3 módulo 4. Para que la
suma x + y de dos elementos de M no pertenezca a M, es suficiente que x + y sea
congruente con 3 módulo 4. Ahora consideramos las sucesiones {xk }k≥1 e {yk }k≥1
definidas, respectivamente por
 
xk = (213 + 1) (4k)2 + 13(4k + 1)2 , yk = 2xk

Entonces,
 
xk + yk = 3xk = 3 (213 + 1) (4k)2 + 13(4k + 1)2 ≡ 3 (mod 4)

y por tanto xk + yk ∈
/ M para todo entero positivo k.

Calculamos ahora
 13
x13
k + yk
13
= x 13
k + 2 13 13
x k = (213
+ 1)x13
k = (213
+ 1) 14
(4k) 2
+ 13(4k + 1) 2
∈M

al ser el producto de un cuadrado por un elemento de M.

6. Se tienen 60 puntos en el interior del cı́rculo unidad. Demostrar que existe un


punto V de la frontera del cı́rculo, tal que la suma de las distancias de V a los 60
puntos es menor o igual que 80.

Solución. Consideremos un triángulo equilátero P QR inscrito en la circunferencia


frontera del cı́rculo unidad dado. Observamos que si para cualquier punto X del
cı́rculo se cumple que
|XP | + |XQ| + |XR| ≤ 4,
entonces sumando las correspondientes desigualdades para los 60 puntos Xk , (1 ≤
k ≤ 60) se obtiene
60
X 60
X 60
X 60
X
|Xk P | + |Xk Q| + |Xk R| = (|Xk P | + |Xk Q| + |Xk R|) ≤ 60 × 4 = 240
k=1 k=1 k=1 k=1

En consecuencia, una de las sumas del miembro izquierdo no excede de 240/3 = 80


y, por tanto, alguno de los puntos P, Q o R verifica el enunciado. Ahora veremos
que
|XP | + |XQ| + |XR| ≤ 4

4
En efecto, debido a la simetrı́a del
recinto, será suficiente probar la de- P
siguladad cuando el punto X esté en
el sector P OQ, siendo O el centro
X y
del cı́rculo. De hecho, el máximo
valor de |XP | + |XQ| + |XR| se al-
canza cuando el punto X está so- z x
bre el arco P Q. En este caso, apli- a a
cando el Teorema de Ptolomeo al
cuadrilátero cı́clico RQXP se tiene O
que ax = ay + az de donde resulta
que x = y + z y como x ≤ 2 entonces Q a R
|XP | + |XQ| + |XR| ≤ 2 + 2 = 4 y
hemos terminado.

5
SOLUCIONES DE LOS PROBLEMAS DE LA OME 49ª

1. Sean a, b y n enteros positivos tales que a  b y ab  1  n 2 . Prueba que


a  b  4 n  3.
Indica justificadamente cuándo se alcanza la igualdad.

SOLUCIÓN:

Supongamos que el resultado a demostrar fuera falso.


Entonces (a  b) 2  (a  b) 2  4ab  4n  3  4(n 2  1)  (2n  1) 2 . Pero como a  b es
entero y a  b  2n  1, entonces a  b  2n y por la desigualdad entre las medias
2
2 ab 2
aritmética y geométrica n  1  ab     n , que es una contradicción. Luego el
 2 
resultado a demostrar es cierto.

El caso de la igualdad requiere que (a  b) 2  4n  3  4(n 2  1)  (2n  1) 2 , debido a la


identidad (a  b) 2  (a  b) 2  4ab. Por tanto a  b  2n  1. La igualdad a  b  4n  3
se alcanza cuando el radicando sea necesariamente un cuadrado perfecto impar; es decir
4n  3  (2u  1) 2 para algún entero no negativo u, con lo que
n  u 2  u  1 y a  b  2u  1.
Además, b  n  u  u 2  1, luego a  b  2u  1  u 2  2u  2. Se comprueba fácilmente
que en efecto ab  u 4  2u 3  3u 2  2u  2  (u 2  u  1) 2  1  n 2  1.
Luego hay igualdad si y sólo si
a  u 2  2u  2, b  u 2  1 y n  u 2  u  1, con a  b  2u  1,
para todo entero no negativo u.

2. Determina todos los números enteros positivos n, para los cuales


Sn  xn  y n  z n
es constante, cualesquiera que sean x, y, z reales tales que, xyz  1 y x  y  z  0.

SOLUCIÓN:

Supongamos z conocido. Entonces x, y son dos números reales con suma  z y


1 1
producto ; es decir son las raíces de la ecuación  2   z   0, o bien:
z z
4
 z  z2 
x, y  z.
2
1
Así una posible solución es por ejemplo z  3 4 y x  y   3 . Sin embargo,
2
podemos hacer z arbitrariamente grande en valor absoluto, siendo entonces uno de los
dos números x ó y también arbitrariamente grande en valor absoluto y de signo
contrario a z. Y el otro arbitrariamente pequeño en valor absoluto y de signo negativo.
Se deduce inmediatamente que n no puede ser par cuando S n es constante, ya que por
una parte toma un valor finito y bien determinado cuando z  3 4 y por otra puede
tomar valores arbitrariamente grandes y siempre positivos cuando z crece
arbitrariamente.
Para finalizar el problema utilizaremos el siguiente resultado.
Lema: Para todo n  3 impar, fijado z , se puede escribir x n  y n como un polinomio
en z de grado n (más algún término posiblemente de exponente negativo) y con
coeficientes iguales a  1, para los términos de grado n, iguales a 0, para los términos
de grado n  1, n  2 y n para los términos de grado n  3.
Demostración:
Para n  3 tenemos que x 3  y 3  ( x  y ) 3  3 xy ( x  y )   z 3  3.
5
Para n  5 tenemos x 5  y 5  ( x  y ) 5  5 xy ( x 3  y 3 )  10 x 2 y 2 ( x  y )   z 5  5 z 2  .
z
Si el resultado es cierto para un entero impar n, entonces para n  2 tenemos
n 1
 n  2  u u n  2 2 u
2

x n 2
y n2
  
 (x  y) n2

u
 x y x  
 y n  2  2u 
u 1  
n 1
n2 n 2
 n  2  x n  2  2 u  y n  2  2u
 z n 2  n
( x  y )     .
z u 2  u  zu

Por hipótesis de inducción la suma para u  2 no contiene ningún término de grado


superior a n  4, y también por hipótesis de inducción, el término de mayor grado del
n2
segundo sumando del miembro de la derecha es  ( z n )  (n  2) z n1.
z
De este modo queda probado el lema por inducción completa.

Del razonamiento inicial se deduce que si S n es constante, no puede ser n par y por el
lema cuando z   para algún n  5 impar dado, se tiene que
x n  y n  z n  nz n 3  ,
con signo positivo además por ser n  3 par en este caso. Luego sólo podrían ser constantes
S1 y S 3 : S1  0 es constante por hipótesis y S 3  3 como consecuencia de un resultado
hallado durante la demostración del lema.
Por tanto, se concluye que S n es constante con las condiciones del enunciado, si y sólo si
n =1 ó n =3.

3. Sean k y n enteros, con n  k  3. Se consideran n  1 puntos en el plano, no


alineados entre sí tres a tres. A cada segmento que une entre sí dos de esos puntos se le
asigna un color de entre k colores dados.
Se dice que un ángulo es bicolor si tiene por vértice uno de los n+1 puntos, y por lados,
dos de los segmentos anteriores que sean de distinto color.
Demuestra que existe una coloración tal que el número de ángulos bicolores es
estrictamente mayor que
2
n k 
n    .
k  2
OBSERVACIÓN: Se denota por  t  la parte entera del número real t ; es decir el mayor entero n  t.

SOLUCIÓN:

Utilizaremos en la solución el siguiente resultado o lema:

Para todo entero k3, se pueden colorear, usando k colores, todos los lados y
diagonales de un polígono regular de k lados, de forma que los k1 lados y diagonales
que confluyen en cada vértice estén coloreados de distinto color. Se puede además
elegir la coloración de forma que, para cada dos vértices consecutivos del polígono, el
color ausente para cada uno de ambos (es decir, aquel color del que no está pintado
ningún lado o diagonal que concurre en cada uno de dichos vértices) sea distinto.

Prueba: Cada lado o diagonal del polígono regular de k lados es una cuerda de su
circunferencia circunscrita, y su mediatriz es eje de simetría del polígono. Como es
conocido que el polígono tiene k ejes de simetría, podemos, a cada eje de simetría,
asociarle un color, y pintar de dicho color todos los lados y diagonales que son
perpendiculares al mismo (aquellos para los que el eje de simetría del polígono es la
mediatriz).Luego dos lados o diagonales tienen el mismo color si y sólo si son paralelos,
y como los k1 lados o diagonales que confluyen en un vértice uniéndolo a los k1
restantes no pueden ser paralelos dos a dos, estos k1 lados o diagonales tienen distinto
color, como queríamos demostrar. Además, dados cuatro vértices consecutivos que
numeramos ordenadamente P1, P2, P3, P4, el color del que está pintada la diagonal P1P3
corresponde a todos los lados y diagonales perpendiculares al eje de simetría que pasa
por P2, luego está ausente del vértice P2, y de forma análoga el color del que está
pintada la diagonal P2P4 está ausente del vértice P3, y estos dos vértices consecutivos
tienen colores ausentes distintos.
Para k  3 se cumple trivialmente, siendo suficiente pintar cada lado del triángulo
equilátero, de uno de los 3 colores disponibles.

Dados n y k cumpliendo las restricciones del enunciado, construimos una coloración en


la que el número de ángulos bicolores sea mayor que la cota propuesta. Para ello,
elegimos primero un punto O, al que llamamos “pivote”, y distribuimos los n puntos
n
restantes en k subconjuntos A1, A2,..., Ak, colocando primero   puntos en cada
k 
subconjunto, y distribuyendo los demás a voluntad. Numeramos ahora P1, P2,..., Pk los
vértices de un polígono regular de k lados, y asociamos a cada subconjunto Ai, el vértice
Pi. Coloreamos además los lados y diagonales del polígono regular de k lados de forma
que en cada vértice concurran k1 lados y diagonales de k1 colores distintos, de
acuerdo al Lema.

Para cada punto Q de los n1 dados y distinto del pivote, pintamos todos los segmentos
que lo unen a cada punto R de los n restantes, de la siguiente forma:
 Si Q y R están en distintos subconjuntos Ai y Aj, pintamos el segmento que los
une del mismo color que el lado o diagonal PiPj en el polígono regular.
 Si Q y R están en el mismo subconjunto Ai, o si RO es el pivote , pintamos el
segmento que los une del color que no está presente en ninguno de los lados y
diagonales que confluyen en el vértice Pi del polígono regular.
n
Claramente, con esta coloración cada punto Q de un subconjunto Ai, está unido a al menos  
k 
puntos por segmentos pintados con cada uno de los k colores. En efecto, para cada uno de los
k1 colores de los que están pintados los lados y diagonales que confluyen en el vértice Pi del
n
polígono regular, el punto Q está unido a los al menos   puntos del subconjunto Aj asociado
k 
al vértice Pj que está unido al vértice Pi por dicho color; y para el color restante, el punto Q está
n
unido a los al menos    1 puntos restantes del conjunto Ai, más al pivote O.
k 
De lo anterior, deducimos que, en la coloración propuesta y para cada uno de los n puntos
n
distintos del pivote, hay al menos   segmentos de cada uno de los k colores que confluyen
k 
2
n
en dicho vértice. Para cada par de colores distintos, hay por lo tanto   posibles pares de
k 
k 
segmentos que forman un ángulo bicolor cuyos lados son estos dos colores, y como hay  
 2
posibles formas de elegir dos colores de entre los k existentes, para cada uno de los n puntos
2
n k 
distintos del pivote hay al menos     ángulos bicolores que lo tienen por vértice, para un
k  2
total de al menos
2
n k 
n   
k   2
ángulos bicolores.

Ahora bien, por la segunda parte del Lema, los segmentos que unen al pivote con los puntos de
dos subconjuntos Ai y Aj, asociados a vértices consecutivos del polígono regular de k lados,
están pintados de los colores ausentes en cada uno de los dos vértices, que son distintos, y hay al
menos un ángulo bicolor con vértice en el pivote O, y por lo tanto, para la coloración
construida, el número de ángulos bicolores es mayor que
2
n k 
n    .
k   2

4. ¿Existen infinitos enteros positivos que no pueden representarse de la forma


a 3  b 5  c 7  d 9  e11 , donde a, b, c, d , e son enteros positivos?
Razona la respuesta.

SOLUCIÓN:

Como 5  7  9  11  3465, veamos cuántos enteros podemos obtener que sean menores o
iguales que N 3465 . Al ser a 3 , b 5 , c 7 , d 9 , e11 positivos, cada uno de ellos es menor que
N 3465 , y tenemos
a  N 1155 , b  N 693 , c  N 495 , d  N 385 y e  N 315 .

Luego al ser 1155  693  495  385  315  3043, hay menos de N 3043 tales números
que se pueden poner en la forma indicada, y hay más de N 3464 números entre los N 3465
primeros que no se pueden representar en la forma propuesta. Al crecer N arbitrariamente,
también aumenta el número de enteros positivos que no pueden representarse en la forma
indicada y por lo tanto sí existen infinitos enteros que no se pueden representar de la
manera propuesta.

5. Estudiar si existe una sucesión estrictamente creciente de enteros 0  a 0  a1  a 2  ... ,


que cumple las dos condiciones siguientes:
i) Todo número natural puede ser escrito como suma de dos términos, no
necesariamente distintos, de la sucesión.
n2
ii) Para cada entero positivo n, se verifica que a n  .
16
SOLUCIÓN:

Sea a n  la sucesión de todos los números naturales k , que en el sistema de numeración


binario sólo tiene unos en las posiciones pares o sólo en los lugares impares. De esta
forma se satisface la condición primera. Probemos a continuación que la estimación de la
condición segunda es también válida.
Consideremos todos los enteros no negativos que son menores que 2 2r , es decir los
números que no tienen más de 2r dígitos (cifras) en su representación binaria. Contamos
el número de elementos de la sucesión que hay entre esos números: hay 2 r elementos que
tienen ceros en todos los lugares pares y 2 r elementos que tienen ceros en todos los
lugares impares y 0 es el único número que se cuenta dos veces. Por tanto hay 2 r 1  1
elementos de la sucesión que son menores que 2 2 r y así a 2r 1 1  2 2 r .
Para cada número natural, se puede encontrar un entero r , tal que 2 r 1  1  n  2 r  2  1.
n n2
Por tanto 2 r  y esto implica que a 2r 1 1  2 2 r  ..
4 16

6. Sea ABCD un cuadrilátero convexo tal que:


AB  CD  2 AC y BC  DA  2 BD .
¿Qué forma tiene el cuadrilátero ABCD ?

SOLUCIÓN :

Sean M es el punto medio de AB, N el de BC, P el de CD y Q el de DA. El punto medio


de la diagonal AC es E y el de la diagonal BD es F.
Las longitudes respectivas de los lados AB, BC, CD y DA son a, b, c y d. Y e y f las de
las diagonales AC y BD, respectivamente.
Las relaciones de Euler en los cuadriláteros convexos las formulamos como
i )a 2  c 2  4  MP 2  b 2  d 2  e 2  f 2
ii ) b 2  d 2  4 NQ 2  a 2  c 2  e2  f 2
iii ) e 2  f 2  4 EF 2  a 2  b 2  c 2  d 2
Recordemos brevemente la demostración de una de ellas, por ejemplo i).

MP es una mediana en el triángulo APB, así que el teorema de la mediana en él nos da


4MP 2  2  PA2  PB 2   AB 2 ;
ahora bien, PA es mediana en ACD y PB lo es en BCD, así que volviendo a aplicar en
esos dos triángulos el teorema de la mediana y sustituyendo en la expresión anterior se
obtiene i).
Vamos a demostrar ahora que se verifica la desigualdad
1
QN   AB  CD  ,
2
y que el signo igual se verifica si y sólo si AB es paralelo a CD.
En efecto, sobre la siguiente figura,

en el triángulo EQN se verifica obviamente QN  QE  EN , valiendo el signo igual


cuando E está sobre QN; puesto que QE es paralela media en ACD (paralela a CD e
igual a su mitad) y EN también es paralela media en ABC (paralela a AB e igual a su
mitad), se tiene, inmediatamente,
1
QN   AB  CD  . (1)
2
(El signo igual vale, entonces, cuando ABCD es un trapecio de bases AB y CD).
A continuación vamos a demostrar que
Si ABCD es un cuadrilátero convexo, entonces
e 2  f 2  d 2  b 2  2ac , (2)
y el signo igual vale si y sólo si AB es paralelo a CD.
En efecto, multipliquemos (1) por 2 y elevemos al cuadrado:
2
4QN 2   AB  CD  ,
y sustituyamos aquí la expresión de QN2 dada por (ii):
b 2  d 2  4 NQ 2  a 2  c 2  e 2  f 2 ;
obtendremos
AC 2  BD 2  AD 2  BC 2  2  AB  CD ,
que es (2), y el signo igual es válido si y sólo si AB es paralelo a CD.
De una forma completamente similar obtendríamos
e 2  f 2  a 2  c 2  2bd , (3)
donde ahora el signo = es válido si y sólo si AD es paralelo a BC.
Sumando (2) y (3) llegamos entonces a que
2 2
2  e 2  f 2    a  c    b  d  , (4)
Donde el signo igual vale si AB es paralelo a CD y BC lo es a AD, es decir, si ABCD es
un paralelogramo. Puesto que elevando al cuadrado las dos relaciones del enunciado y
sumando se obtiene (4) y hemos terminado.

CONSTRUCCIÓN GRÁFICA:

Utilizaremos las notaciones siguientes.


Lados: AB = a, BC = b, CD = c, DA = d.
Diagonales del cuadrilátero: AC = d1, y BD = d2. Y para fijar ideas supongamos que
d1  d 2 .
Además, M es el punto medio de AB, N el de BC, P el de CD y Q el de DA. El punto
medio de la diagonal AC es E y el de la diagonal BD es F.
Según el enunciado, se cumplen las relaciones
a  c  2d1 y b  d  2d 2 .
Dados d1 y d2, se construyen sendos triángulos rectángulos isósceles cuyos catetos son
d1 y d2; entonces sus hipotenusas son a + c y b + d respectivamente.

El punto M (que coincidirá con A una vez terminada la construcción) será centro de una
circunferencia de radio d1; el punto N será centro de una circunferencia de radio d2. A
uno de los puntos de intersección de ambas circunferencias se le llama O. Sea P el
simétrico de M respecto de O.
En el triángulo NOP, el ángulo   NOP es el suplementario de MON .
Apliquemos el teorema del coseno en los triángulos MON y NOP:

En NOP : NP 2  d 12  d 22  2d 1 d 2 cos  y


en MNO :  2 d1 2
 d 12  d 22  2d1 d 2 cos

Sumando miembro a miembro y simplificando, obtenemos


NP 2  2d 2 2  NP  2d 2  b  d .
Trazando por P la paralela a MN y por M la paralela a NP obtenemos el punto Q.
El paralelogramo MNPQ es semejante al que buscamos; éste se obtiene trazando las
paralelas a los lados por el punto O.
Para que la construcción sea posible, debe ser
   
d1  d 2  d1 2  d 2  2  1 d1  d1 2  1  d 2  d1 .
Si d1  d 2  cos   0 y se obtendría un cuadrado.
SOLUCIONES DE LA 48ª OME

1. Determinar razonadamente si el número λn = 3n 2 + 2n + 2 es irracional para


todo entero no negativo n.

SOLUCIÓN. Supongamos que n es par. Entonces, 3n 2 + 2n es múltiplo de 4 y


3n 2 + 2n + 2 es múltiplo de 2 pero no de 4, con lo que no puede ser un cuadrado
perfecto.
Supongamos que n es impar. Cualquier cuadrado perfecto impar da resto 1 al dividir
entre 8; este resultado se demuestra trivialmente, escribiendo el cuadrado de cualquier
entero impar 2m + 1 en la forma 4m(m + 1) + 1 y observando que, bien m, bien m + 1,

ha de ser par. Se tiene entonces que si n es impar y 3n 2 + 2n + 2 fuera un cuadrado


perfecto, entonces 3n 2 + 2n + 2 daría resto 1 al dividir entre 8, o equivalentemente,
2n daría resto 1 − 2 − 3 = −4 al dividir entre 8, con lo que 2n sería múltiplo de 4 y n

par, contradicción. Luego para cualquier entero, positivo o negativo, 3n 2 + 2n + 2 es


un entero que no es un cuadrado perfecto, por tanto λ n es siempre irracional para

cualquier entero n, positivo o negativo.


Nótese también que λn es siempre real, incluso cuando n es un entero negativo, pues

3n 2 + 2n + 2 > (n + 1) 2 ≥ 0.

2. Hallar todas las funciones f : R → R de variable real con valores reales, tales
que
( x − 2) f ( y ) + f ( y + 2 f ( x)) = f ( x + y f ( x)) (1)
para todo x, y ∈ R.

SOLUCIÓN. Supongamos primeramente que f (0) = 0. Haciendo x = 0 en (1),


f ( y ) = 0 para todo y ∈ R. Esta función satisface la ecuación funcional dada (1).
Sea f (0) ≠ 0. Haciendo y = 0 en (1), se obtiene ( x − 2) f (0) + f (2 f ( x)) = f ( x) para
todo x ∈ R. Claramente esto implica que f es inyectiva porque si f ( x1 ) = f ( x 2 ),
entonces ( x1 − 2) f (0) + f (2 f ( x1 )) = f ( x1 ) = ( x 2 − 2) f (0) + f (2 f ( x 2 )) = f ( x 2 ) y por
tanto ( x1 − x 2 ) f (0) = 0 y x1 = x 2 .
Poniendo ahora x = 2 en (1), f ( y + 2 f (2)) = f (2 + yf (2)) para todo y ∈ R. Al ser
f inyectiva y + 2 f (2) = 2 + yf (2) para todo y ∈ R. En esta igualdad si y = 0,
3
f (2) = 1. Al ser inyectiva f (3) ≠ 1; por tanto con x = 3 e y = se llega a
1 − f (3)

1
⎛ 3 ⎞
f ⎜⎜ + 2 f (3) ⎟⎟ = 0. Hemos de demostrar que f tiene un cero. Sea a, tal que
⎝ 1 − f (3) ⎠
f (a) = 0. Poniendo ahora y = a en (1) resulta f (a + 2 f ( x)) = f ( x + af ( x)) para todo
x ∈ R. Así por la inyectividad de f , a + 2 f ( x) = x + af ( x) para todo x ∈ R. Como
x−a
a ≠ 2, f ( x) = . Sustituyendo esta función en (1) resulta que a = 1, lo que
2−a
proporciona dos únicas soluciones de la ecuación funcional inicial f ( x) = 0 y
f ( x) = x − 1.

3. Sean x y n enteros tales que 1 ≤ x < n. Disponemos de x + 1 cajas distintas y


n − x bolas idénticas. Llamamos f (n, x) al número de maneras que hay de
distribuir las n − x bolas en las x + 1 cajas. Sea p número primo, encontrar los
enteros n mayores que 1 para los que se verifica que el número primo p es
divisor de f (n, x) para todo x ∈ {1, 2, ..., n − 1 }.

SOLUCIÓN. Claramente f (n, x) es el número de combinaciones con repetición de


x + 1 elementos tomados de n − x en n − x. Es decir,
⎛ ( x + 1) + (n − x) − 1⎞ ⎛ n ⎞
f (n, x) = CR ( x + 1, n − x) = ⎜⎜ ⎟⎟ = ⎜⎜ ⎟⎟.
⎝ n−x ⎠ ⎝ x⎠
Vamos a probar que los n buscados son todos los de la forma p a con a entero positivo.
Sea m p la p -parte del entero positivo m, es decir si m = p a q (con q ≥ 1 entero),
m p = p a , siendo a ≥ 1 entero. Ahora probaremos el siguiente resultado previo:
Si m p = p a , entonces (m − i ) p = i p para cada i ∈ {1, 2, ..., p a − 1 }.
En efecto, si i p = p k entonces k < a y es obvio que p k (m − i ), luego i p ≤ (m − i ) p .
Recíprocamente, si (m − i ) p = p k , ha de ser k < a porque si no sería p a i. Ahora,
p k i porque p k m y p k (m − i ). Es decir (m − i ) p ≤ i p .

A continuación probaremos que si p es primo y n un entero mayor que 1. Entonces p


⎛n⎞
divide a ⎜⎜ ⎟⎟ para todo x ∈ {1, 2, ..., n − 1 } si y sólo si n = p a con a entero.
⎝ x⎠
⎛ n⎞ ⎛ n⎞
Si p ⎜⎜ ⎟⎟ para todo x ∈ {1, 2, ..., n − 1 }, p ⎜⎜ ⎟⎟ = n. Poniendo n p = p a , se tiene:
⎝ x⎠ ⎝1⎠
⎛ n ⎞ n (n − 1) ...(n − p a + 1)
⎜⎜ a ⎟⎟ =
⎝p ⎠ p a ( p a − 1)...2 ⋅ 1
⎛ n ⎞
y por el resultado previo concluimos que la p − parte de ⎜⎜ a ⎟⎟ es 1, luego n = p a .
⎝p ⎠
Recíprocamente, si n = p a , para cada x ∈ {1, 2, ..., p a − 1 },
⎛ n ⎞ p a ( p a − 1) ...( p a − x + 1)
⎜⎜ ⎟⎟ =
⎝ x⎠ x ( x − 1)...2 ⋅ 1

2
⎛n⎞ pa
y de nuevo por el resultado previo, la p − parte de ⎜⎜ ⎟⎟ es , que es múltiplo de p
⎝ x⎠ xp
por ser x < p a .

4. Hallar todos los números enteros positivos n y k, tales que


(n + 1) n = 2n k + 3n + 1.

SOLUCIÓN. Para n = 1 , la ecuación se escribe 2 = 6, claramente falsa. Luego n ≥ 2.


Por la fórmula del binomio de Newton,
(n + 1)n − 1 = n 2 + ⎛⎜⎜ ⎞⎟⎟n 2 + ⎛⎜⎜ ⎞⎟⎟n 3 + ...
n n
⎝ 2⎠ ⎝ 3⎠
es múltiplo de n2. Tenemos entonces dos casos a analizar:
• k = 1. Entonces, n2 divide a 2n1+3n = 5n, es decir, n divide a 5, con lo que n = 5,
y la ecuación se convierte en 65=26, claramente falsa.
• k ≥ 2. Entonces, n2 divide a 2nk + 3n, pero como divide a 2nk, también ha de
dividir a 3n, es decir, n divide a 3, con lo que n=3. Se comprueba fácilmente
que para n = 3, la ecuación se convierte en 43 = 2×3k + 10, luego en 3k = 27 = 33,
que es cierta si y sólo si k = 3.

5. Una sucesión (a n )n≥1 se define mediante la recurrencia


a n2−1 + 4
a1 = 1, a 2 = 5, a n = , para n ≥ 3.
an−2
Demostrar que todos los términos de la sucesión son números enteros y encontrar
una fórmula explícita para a n .

SOLUCIÓN. Observamos a partir de la definición que a k a k − 2 = a k2−1 + 4 y


a k +1 a k −1 = a k2 + 4 . Restando a la segunda ecuación de la primera, resulta
a k +1 a k −1 − a k a k −2 = a k2 − a k2−1 ⇔ a k2−1 + a k −1 a k +1 = a k2 + a k a k + 2 ,
que es equivalente a su vez a a k −1 (a k −1 + a k +1 ) = a k (a k + a k −2 ). Haciendo que 3 ≤ k ≤ n,
se obtienen
a 2 (a 2 + a 4 ) = a3 (a3 + a1 ),
a3 (a3 + a5 ) = a 4 (a 4 + a 2 ),
a 4 (a 4 + a 6 ) = a5 (a5 + a3 ),
………………………….
a n − 2 (a n − 2 + a n ) = a n −1 (a n −1 + a n −3 ),
a n −1 (a n −1 + a n +1 ) = a n (a n + a n − 2 ).

Multiplicando las igualdades anteriores y simplificando términos, resulta

3
a 2 (a n −1 + a n +1 ) = a n (a1 + a3 ) y teniendo en cuenta que a1 = 1, a 2 = 5 y a3 = 20, resulta
a n +1 = 6a n − a n −1 . De este modo es inmediato que todos los términos de la sucesión son
enteros.
Para encontrar una formula explícita de a n ensayamos con a n = t n con lo que
obtenemos a partir de la última expresión que
t n +1 − 6t n − t n −1 = 0 ⇔ t n −1 (t 2 − 6t + 1) = 0.
Se tienen tres soluciones. La primera t = 0, que no cumple con el enunciado. Las otras
dos t = 3 ± 2 2 combinadas linealmente, sí lo harán. Es decir la solución que buscamos
(
es de la forma a n = λ 3 − 2 2 ) n
( ) n
+ µ 3 + 2 2 , λ , µ ∈ R.
Para determinar las constantes λ y µ utilizamos que a1 = 1 y a 2 = 5 y tenemos
⎧⎪λ (3 − 2 2 ) + µ (3 + 2 2 ) = 1
⎨ .
⎪⎩λ (3 − 2 2 ) 2 + µ (3 + 2 2 ) 2 = 5
2+ 2 2− 2
Resolviendo este sistema λ = y µ= , con lo que
4 4

an =
2+ 2
4
(
3− 2 2 +
n
) 2− 2
4
( n
3+ 2 2 . )

6. Sea ABC un triángulo acutángulo, ω su circunferencia inscrita de centro I , Ω


su circunferencia circunscrita de centro O, y M el punto medio de la altura AH ,
donde H pertenece al lado BC. La circunferencia ω es tangente a este lado BC
en el punto D. La recta MD corta a ω en un segundo punto P, y la
perpendicular desde I a MD corta a BC en N . Las rectas NR y NS son
tangentes a la circunferencia Ω en R y S respectivamente. Probar que los puntos
R, P, D y S están en una misma circunferencia.

SOLUCIÓN. Supongamos que b = c. Entonces, el pie de la altura H coincide con el


punto de tangencia D, luego DM es perpendicular a BC y N no está definido.
Asumiremos entonces sin pérdida de generalidad que b > c. Sea U el punto de la recta
BC cuya potencia es la misma respecto de ω y Ω. Claramente, hay exactamente dos
tangentes a cada una de ambas circunferencias que pasan por U , siendo D el punto de
tangencia de una de ellas con ω; llamemos E al punto de tangencia con ω de la
segunda recta que pasa por U . La distancia de U a los cuatro puntos de tangencia es la
misma, luego existe una circunferencia de centro U que pasa por los cuatro puntos, es
decir, si demostramos que U = N , el problema quedaría resuelto. Ahora bien, el eje
radical de la circunferencia descrita con centro U y ω , es claramente la recta DE y la
perpendicular a esta recta por I es la mediatriz de la cuerda DE , luego pasa por U .
Basta entonces con demostrar que el punto W de la altura AH cuya potencia es la
misma respecto a la circunferencia de centro U por D y por E , y respecto a ω , es el
punto medio de AH , con lo que sería P = E y N = U . Ahora bien, dicha potencia es

UD 2 − UW 2 = ID 2 − IW 2

4
Pero UW 2 = UH 2 + WH 2 , IW 2 = (WH − ID) 2 + HD 2 , con lo que la anterior condición
es equivalente a
HD ⋅ UD
UD 2 − 2WH ⋅ ID = UH 2 − HD 2 = UD(UD − 2 HD ), WH = ,
ID
y el problema se reduce a demostrar que esta última expresión es la mitad de la altura.
Llamando s al semiperímetro de ABC , tenemos que BD = s − b, CD = s − c,
BH = c cos B, y al estar U definido como el punto sobre BC tal que su potencia es la
misma respecto de ω y Ω, y llamando Σ al área de ABC y usando la fórmula de
Herón para la misma, tenemos
BD ⋅ CD (s − b )(s − c ) Σ2
UD 2 = (UD − BD )(UD + CD ) , UD = = = .
CD − BD b−c s (b − c )(s − a )
Luego

WH = =
(
h a (s − b − c cos B ) h a(a + b + c ) − 2ab − a 2 − c 2 + b 2)=
2 (b − c )(s − a ) 2 (b − c )(b + c − a )

=
(
h ac − ab − c 2 + b 2 ) h
= ,
2 (b − c )(b + c − a ) 2
como queríamos demostrar.

5
XLVII Olimpiada Matemática Española
Fase nacional (Pamplona)
25 y 26 de marzo de 2011

ENUNCIADOS Y SOLUCIONES OFICIALES

1. En un polígono regular de 67 lados trazamos todos los segmentos que unen


dos vértices, incluidos los lados del polígono. Elegimos n de estos segmentos y
asignamos a cada uno de ellos un color entre 10 colores posibles. Halla el valor
mínimo de n que garantiza, que independientemente de cuáles sean los n
segmentos elegidos y de cómo se haga la asignación de colores, siempre habrá
un vértice del polígono que pertenece a 7 segmentos del mismo color.

SOLUCIÓN:
Veamos en primer lugar que con n = 2010 no es suficiente.
Diremos que un segmento es de tamaño r si une dos vértices entre los que, por el
camino más corto siguiendo los lados del polígono, hay otros r − 1 vértices. Elegimos
los 2010 segmentos de tamaño mayor que 3. Para cada r ∈ {1, 2,...,10 }, asignamos el
color r a los segmentos de tamaño 3r + 1, 3r + 2 y 3r + 3. Es obvio que cada vértice
pertenece a 6 segmentos de cada color.
Ahora vamos a probar que si n = 2011, hay algún vértice que está en 7 segmentos del
mismo color. En efecto, en los 2011 segmentos intervienen, contando repeticiones,
4022 vértices, luego, por el principio del palomar, como 4022 > 60 × 67, algún vértice
interviene en, al menos, 61 segmentos, de los cuales, de nuevo por el principio del
palomar, al menos, 7 serán del mismo color.

2. Sean a, b, c números reales positivos. Demuestra que

a b c ab + bc + ca 5
+ + + ≥ .
b+c c+a a+b a2 + b2 + c2 2

¿Cuándo se alcanza la igualdad?

SOLUCIÓN:
Nótese en primer lugar que, en virtud de la desigualdad entre medias aritmética y
geométrica, se tiene
2
1 a2 + b2 + c2 ab + bc + ca 3 a 2 + b 2 + c 2 ⎛ ab + bc + ca ⎞ 3
+ ≥ 3 ⎜ 2 ⎟ = ,
2 ab + bc + ca a +b +c
2 2 2
2 ab + bc + ca ⎜⎝ a + b + c ⎟⎠
2 2
2
dándose la igualdad si y sólo si a2+ b2+ c2= ab + bc + ca, o equivalentemente, si y sólo
si (a− b)2 + (b− c)2 + (c− a)2 = 0, es decir, si y sólo si a = b = c. Nos bastaría entonces,
para concluir el problema, con demostrar que
a b c 1 a 2 + b2 + c2
+ + ≥1+ .
b+c c+a a+b 2 ab + bc + ca
Esto puede conseguirse por fuerza bruta, viéndose en primer lugar que el miembro de la
derecha puede escribirse como
a 3 + b 3 + c 3 + abc
+1,
(a + b )(b + c )(c + a )

1
y a partir de aquí comparando las fracciones restantes, llegándose a la conclusión tras
algo de cálculo. Sin embargo, se puede obtener el resultado deseado de una forma más
elegante, ya que podemos escribir
2 2 2
a b c a2 b2 c2 ⎛ a ⎞ ⎛ b ⎞ ⎛ c ⎞
+ + = + + =⎜ ⎟ +⎜ ⎟ +⎜ ⎟
b + c c + a a + b ab + ca bc + ab ca + bc ⎝ ab + ca ⎠ ⎝ bc + ab ⎠ ⎝ ca + bc ⎠
,
( ) ( ) (
2(ab + bc + ca ) = ab + ca + bc + ab + ca + bc ,
2 2
) 2

y por la desigualdad del producto escalar, se tendría

2(ab + bc + ca )
a b c
+ + ≥
b+c c+a a+b
a b c
≥ ab + ca + bc + ab + ca + bc = a + b + c,
ab + ca bc + ab ca + bc
es decir,
a
+
b
+
c

(a + b + c )2 = a 2 + b 2 + c 2 + 1 ,
b + c c + a a + b 2(ab + bc + ca ) 2(ab + bc + ca )
con lo que se concluye el problema. Nótese que la igualdad se da en la última
desigualdad si y sólo si
ab + ca bc + ab ca + bc
ab + ca = bc + ab = ca + bc ,
a b c
es decir, si y sólo si a = b = c, que es por lo tanto también la condición necesaria y
suficiente para que se dé la igualdad en la desigualdad propuesta.

3. Sean A, B, C , D cuatro puntos en el espacio, tales que no hay ningún plano que
pasa por los cuatro a la vez. Los segmentos AB, BC , CD, DA son tangentes a una
misma esfera. Demuestra que los cuatro puntos de tangencia están en un mismo
plano.

SOLUCIÓN:
Sean R, S, T, U los puntos de tangencia respectivos de la esfera con los segmentos AB,
BC, CD, DA. Siendo O el centro de la esfera, claramente OU, OR son respectivamente
perpendiculares a DA, AB, con lo que los triángulos OAU y OAR son rectángulos
respectivamente en U, R, y al ser OU = OR el radio de la esfera, ambos triángulos son
iguales, luego AU = AR = a, y de forma análoga,
BR = BS = b, CS = CT = c, y DT = DU = d.

2
Las rectas UR y BD están en el plano del triángulo ABD, así o bien se cortan o son
paralelas. Si UR y BD son paralelas, del teorema de Thales resulta b = d, y del
recíproco de Thales resulta que también son paralelas TS y DB. Entonces, UR y TS son
paralelas y los cuatro puntos están en un plano.
Si UR y BD no son paralelas, sea E su punto de intersección. Tampoco pueden
entonces ser paralelas TS y DB, porque si lo fueran, lo serían UR y BD, en contra de lo
supuesto. Sea entonces F el punto de intersección de TS y DB. Aplicando el teorema
de Menelao en ABD cortado por UR, y en BCD cortado por TS, obtenemos
BE DU AR BE d BF DT CS BF d BE BF
1= = , 1= = , = .
ED UA RB ED b FD TC SB FD b ED FD
Entonces, E y F dividen a DB en la misma razón. Como ninguno de los dos puntos (E y
F) pueden estar entre D y B – pues BCD tendría tres puntos de intersección con ST –
entonces E y F tienen que coincidir. Las rectas UR y ST se cortan entonces en E = F,
luego están en un mismo plano y en él, en particular, están los puntos U, R, S y T.

4. Sea ABC un triángulo con ∠B = 2∠C y ∠A > 90 o . Sean D el punto de la


recta AB tal que CD es perpendicular a AC , y M el punto medio de BC .
Demuestra que ∠AMB = ∠DMC .

SOLUCIÓN:

La recta que pasa por A y es paralela a BC corta a DM y a DC en los puntos N y


F respectivamente.
Se sigue que AN : BM = DN : DM = NF : MC .
Pues BM = MC , resulta AN = NF . Como que ∠ACF = 90 o , tenemos AN = NC , de
manera que ∠NCA = ∠NAC = ∠ACB .
Así pues, ∠NCB = 2∠ACB = ∠ABC .
Como AN BC , el cuadrilátero ABCN es un trapecio isósceles con AB = NC .
Por consiguiente, ∆ABM ≡ ∆NCM y ∠AMB = ∠NMC = ∠DMC
.
D

N
A
F
α

α 90
2α α
B M C

3
5. Cada número racional se pinta de un color, usando sólo dos colores, blanco y
rojo. Se dice que una tal coloración es sanferminera cuando para cada dos
números racionales x, y, con x ≠ y, si se cumple una de las tres condiciones
siguientes:
a) xy = 1,
b) x + y = 0,
c) x + y = 1,
entonces x e y están pintados de distinto color. ¿Cuántas coloraciones
sanfermineras hay?

SOLUCIÓN:
Si una coloración es sanferminera, podemos hallar otra coloración sanferminera
intercambiando simultáneamente el color de cada racional, de rojo a blanco y de blanco
a rojo; si en la coloración inicial dos racionales tienen distinto color, también lo tendrán
en la resultante. Hallemos entonces el número de coloraciones sanfermineras tales que,
sin pérdida de generalidad, el racional 1 está pintado de rojo, y el número total de
coloraciones sanfermineras será exactamente el doble.
Supongamos entonces que tenemos una coloración sanferminera con el 1 pintado de
rojo. Dado el color del racional x>0, el color del racional −x < 0 tiene que ser distinto,
pues x + (− x) = 0; al mismo tiempo, como 1 es rojo y 1 + 0 = 1, necesariamente 0 es
blanco. Luego fijado el color de cada racional positivo en una coloración sanferminera,
quedaría fijado el color de todos los racionales. Al mismo tiempo, para cada racional
positivo x, como ( x + 1) + (− x) = 1, entonces x + 1 y − x tienen color distinto, luego
x y x + 1 tienen el mismo color. Por inducción sobre n, se comprueba entonces
fácilmente que para cada racional positivo x y para cada entero positivo n, los racionales
x y n + x tienen el mismo color. En particular, todos los enteros positivos serán
necesariamente de color rojo.
Coloreados entonces todos los enteros positivos de color rojo, procedemos ahora de la
siguiente manera para los racionales positivos q que no tienen un color asociado
u
todavía: expresamos q = m + , donde m es la parte entera de q, y u < v son enteros
v
positivos primos entre sí; esta expresión es claramente única para cada racional positivo
q, y para dos racionales positivos que difieran en un entero, los valores de u, v son
v
claramente los mismos. Si tiene ya un color asociado, entonces le asociamos a q el
u
v
color opuesto; si no tiene todavía color asociado, entonces procedemos de la misma
u
v
forma con , hasta llegar a un racional que sí tenga color asociado, pudiendo entonces
u
v
asociarle un color a , y asociándole a q el color opuesto; nótese que en cualquier caso
u
x y n + x tendrán el mismo color asociado. El proceso termina necesariamente, ya que
v u'
expresando q = = m + , se tiene que v v' = m u v'+u u ' , luego al ser primos entre sí u,
u v'
v, entonces u divide a v', y al ser primos entre sí u', v', entonces v' divide a u, luego u = v'
> u', y tras a lo sumo u pasos, llegaríamos a u' = 0, es decir, a un entero, que sí tiene

4
asociado un color. Nótese además que el proceso es único ya que, en cada paso, los
valores de m, u, v quedan unívocamente determinados por el valor del racional positivo
q, con lo que al ser todos los enteros positivos rojos, cada racional positivo podrá tener
uno y sólo un color asociado. Esta coloración única para los racionales positivos se
extiende también de forma única a todos los racionales, como ya hemos visto. Existe
entonces a lo sumo una coloración sanferminera pintando el racional 1 de rojo.

Comprobemos que en efecto esta coloración única que hemos construido satisface las
condiciones del enunciado:
1) Si x + y = 0 con x ≠ y, sin pérdida de generalidad x > 0 > y = −x, luego y tiene,
por la forma de extender la coloración a todos los racionales, color distinto al
de x.
2) Si xy = 1 con x ≠ y, o ambos son positivos, o ambos son negativos, teniendo en
el segundo caso x, y colores respectivamente opuestos a los de −x, −y, que serían
positivos, reduciéndose la comprobación al primer caso. Si x, y son ambos
u
positivos, sin pérdida de generalidad x>1> y, con y = 0 + para enteros
v
v
positivos u < v, y por construcción a y se le asigna color opuesto al de x = .
u
3) Si x + y = 1 con x ≠ y, entonces sin pérdida de generalidad x > y, y bien x = 1 es
rojo, y = 0 es blanco, bien x > 1> 0 > y, bien 1 > x > y > 0. En el segundo caso, el
racional positivo x−1 tiene por construcción el mismo color que x, luego por 1),
x tiene color opuesto a 1−x = y. En el tercer caso, existen enteros positivos u < v
v u
primos entre sí tales que x = ,y= . Por construcción y por 2), x tiene
u+v u+v
u+v u v
color opuesto al de = 1 + , luego x tiene el mismo color que . Pero
v v u
u+v v
también por construcción y por 2), y tiene color opuesto al de y = = 1+ ,
u u
v
luego opuesto al de , y opuesto al de x.
u
Luego la coloración construida es sanferminera, y es la única que puede ser
sanferminera con el racional 1 pintado de rojo. Restaurando la generalidad, hay
exactamente dos coloraciones sanfermineras.

6. Sea ( S n ), con n ≥ 0, la sucesión definida por:


(i) S n = 1 para 0 ≤ n ≤ 2011.
(ii) S n + 2012 = S n + 2011 + S n , para n ≥ 0.
Demuestra que, para todo entero no negativo a, se cumple que S 2011a − S a , es
múltiplo de 2011.

SOLUCIÓN:

Sea p = 2011 . Observemos que p es primo.


Sea An el número de formas de cubrir un tablero de [ n filas ] × ⎡⎣( p + 1) columnas ⎤⎦ con
fichas de dimensiones 1× ( p + 1) (que podemos colocar horizontal o verticalmente).

Probemos primero que An = S n .

5
Tenemos que:
• para 1 ≤ n ≤ p , An = 1 = S n .
• Ap +1 = 2 = S p+1.
• para n ≥ 0 , An + p +1 = An + p + An ; en efecto, al cubrir un tablero de
⎡⎣( n + p + 1) filas ⎤⎦ × ⎡⎣( p + 1) columnas ⎤⎦ con fichas de 1× ( p + 1) , se da uno de
los dos casos siguientes:
*o bien la última fila, de ( p + 1) casillas, está cubierta por una ficha colocada
horizontalmente: en este caso, hay An + p formas de cubrir el tablero.
*o bien las ( p + 1) últimas filas, cada una de ( p + 1) casillas, están cubiertas por
( p + 1) fichas en posición vertical: en este caso, hay An formas de cubrir el
tablero.
Deducimos que, para n > 0 , An = S n .

Hallemos ahora una expresión de An en función de n.


Escribamos la división euclidiana de n por ( p + 1) : n = q ( p + 1) + r .
Observemos que, al cubrir el tablero, se pueden juntar las fichas colocadas en posición
vertical para formar cuadrados de ( p + 1) × ( p + 1) :

p+1

p+1
n

Llamemos bloques a estos cuadrados.


Sea Ank el número de formas de cubrir el tablero con k bloques, para 0 ≤ k ≤ q .
Dada una de estas formas de cubrir el tablero, llamemos B1 ,..., Bk los bloques ordenados
de arriba hacia abajo, y sean a0 el número de filas situadas encima de B1 , a1 el número
de filas entre B1 y B2 , a2 el número de filas entre B2 y B3 ,…, ak el número de filas
debajo de Bk . Por ejemplo, en el dibujo anterior, k = 2, a0 = 5, a1 = 3, a2 = 0 . Entonces
se tiene que a0 + ... + ak = n − ( p + 1) k .
Recíprocamente, a una ( k + 1) -upla de enteros no negativos ( a0 ,..., ak ) tal que
a0 + ... + ak = n − ( p + 1) k , le podemos asociar una única forma de cubrir el tablero con k
bloques, colocando de forma alternada y da arriba hacia abajo a0 fichas horizontales, un

6
bloque, a1 fichas horizontales, un bloque, …, ak −1 fichas horizontales, un bloque, ak
fichas horizontales.
Por tanto, Ank es el número de ( k + 1) -uplas de enteros no negativos ( a0 ,..., ak ) tales
⎛ n − pk ⎞
que a0 + ... + ak = n − ( p + 1) k , luego Ank = ⎜ ⎟.
⎝ k ⎠
q
⎛ n − pk ⎞
Deducimos que S n = An = ∑ ⎜⎜ ⎟.
k =0 ⎝ k ⎟⎠

⎢ pc ⎥
⎢ ⎥
⎛ p (c − k ) ⎞ ⎣ p +1 ⎦
Estudiemos finalmente S pc =
k =0 ⎝
∑ ⎜⎜
⎟⎟ (mod p) .
k ⎠
Para ello, utilizamos el lema siguiente:
Lema: Para todo primo p y enteros a y b, se tiene
⎛ pa ⎞ ⎛ a ⎞
(i) ⎜⎜ ⎟⎟ ≡ ⎜⎜ ⎟⎟ (mod p).
⎝ pb ⎠ ⎝ b ⎠
⎛ pa ⎞
(ii) Si p no divide a b, entonces p | ⎜ ⎟ .
⎝ b ⎠
Si admitimos el lema,
⎢ pc ⎥ ⎢ 1 ⎢ pc ⎥ ⎥ ⎢ c ⎥
⎢ ⎥ ⎢ ⎢ ⎥⎥ ⎢ ⎥
⎛ p (c − k ) ⎞
⎣ p +1 ⎦
⎛ c − pk ⎞⎣⎢ p ⎣ p +1 ⎦ ⎦⎥ ⎣ p +1 ⎦
⎛ c − pk ⎞
S pc = ∑ ⎜⎜
k =0 ⎝
⎟⎟ ≡ ∑ ⎜⎜ ⎟⎟ ≡ k = ∑ ⎜⎜ ⎟⎟ ≡ S c (mod p),
k ⎠ k =0 ⎝ k ⎠ k =0 ⎝ k ⎠
que es lo que queríamos probar.

Demostración del lema:


(i)
a −1 a

⎛ pa ⎞ ∏ ( pk + 1) ...( pk + p − 1) ∏ ( pk )
⎜ ⎟= ⋅
k =0 k =1
b −1 a − b −1 b a −b
⎝ pb ⎠
∏ ( pk + 1) ... ( pk + p − 1) ∏ ( pk + 1) ...( pk + p − 1) ∏ ( pk )∏ ( pk )
k =0 k =0 k =1 k =1

( ( p − 1)!)
a
⎛a⎞ ⎛a⎞
≡ ⋅ ⎜ ⎟ ≡ ⎜ ⎟ ( mod p )
( ( p − 1)!) ( ( p − 1)!)
b a −b
⎝b ⎠ ⎝b ⎠
(ii) Si n es el cociente de b dividido por p,
⎛ ⎛ pa ⎞ ⎞ a a − n −1 a a − n −1
v p ⎜ ⎜ ⎟ ⎟ = ∑ v p ( pk ) − ∑ v p ( pk ) = ∑ v p ( pk ) − ∑ v p ( pk ) + v p ( pa ) > 0 ,
⎝ ⎝ b ⎠ ⎠ k = n +1 k =1 k =n+2 k =1
a a − n −1
puesto que ∑ v ( pk ) − ∑ v ( pk ) ≥ 0 .
k =n+2
p
k =1
p

7
Olimpiada
Matemática
Española RSME

XLVI Olimpiada Matemática Española


Fase nacional 2010 (Valladolid, 26 y 27 de Marzo)
Soluciones oficiales

• Problema 1
Una sucesión pucelana es una sucesión creciente de dieciséis números impares positivos con-
secutivos, cuya suma es un cubo perfecto. ¿Cuántas sucesiones pucelanas tienen solamente
números de tres cifras?

Solución:
Sea la sucesión n, n + 2, . . . , n + 30. Entonces la suma es 12 16(2n + 30) = 8(2n + 30). Por
tanto, es necesario que 2n + 30 sea un cubo perfecto. Ahora hay que contar el número de
tales n que son impares y verifican 101 ≤ n ≤ 969. Los cubos pares entre 232 y 1968 son
512, 1000 y 1728, que corresponden a valores de n de 241, 485 y 849. Por lo tanto hay
exactamente tres sucesiones pucelanas.

• Problema 2
Sean N0 y Z el conjunto de todos los enteros no negativos y el conjunto de todos los enteros,
respectivamente. Sea f : N0 → Z la función que a cada elemento n de N0 le asocia como
imagen el entero f (n) definido por
 n  n
f (n) = −f −3
3 3

donde x es la parte entera del número real x y {x} = x −x su parte decimal. Determina
el menor entero n tal que f (n) = 2010.
NOTA: La parte entera de un número real x, denotada por x es el mayor entero que no
supera a x. Ası́ 1,98 = 1, −2,001 = −3, 7π − 8,03 = 13.

Solución:
Se prueba fácilmente por inducción que, si n = (ak ak−1 . . . a0 )3 , entonces

k
 k

f (n) = aj − aj
j=0 j=0
j impar j par

En efecto, f (0) = 0, f (1) = −1, f (2) = −2.


k k
Supongamos que, para todo n menor que 3t, f (n) = j=0 aj − j=0 aj . Entonces, si
j impar j par

t = bk . . . b0 3 , 3t = bk . . . b0 0 3 , 3t + 1 = bk . . . b0 1 3 , 3t + 2 = bk . . . b0 2 3 . Por lo tanto,
como f (3t) = −f (t), f (3t + 1) = −f (t) − 1, f (3t + 2) = −f (t) − 2, la propiedad sigue siendo
cierta para todo entero n menor que 3t + 2.
k k
Luego, para todo n = (ak ak−1 . . . a0 )3 , f (n) = j=0 aj − j=0 aj .
j impar j par

De esta forma, se obtiene el menor n = 2020 . . . 20 3 , tal que f (n) = 2010. Este número
contiene 1005 doses; su valor en base decimal es:

32010 − 1

4

• Problema 3
Sea ABCD un cuadrilátero convexo. Sea P la intersección de AC y BD. El ángulo
 AP D = 60◦ . Sean E, F , G y H los puntos medios de los lados AB, BC, CD y DA
respectivamente. Halla el mayor número real positivo k tal que

EG + 3HF ≥ kd + (1 − k)s

siendo s el semiperı́metro del cuadrilátero ABCD y d la suma de las longitudes de sus


diagonales. ¿Cuándo se alcanza la igualdad?

Solución:

Probaremos que k = 1 + 3 y que la igualdad se da si, y sólo si, ABCD es un rectángulo.
Z D
H
A

P G Y
W
E

B F C
X
Sean W , X, Y y Z cuatro puntos exteriores a ABCD de modo que los triángulos ABW y
DCY sean equiláteros, el triángulo BCX sea isósceles en X, el triángulo AZD sea isósceles
en Z y  BXC =  AZD = 120◦ .
Los cuadriláteros W AP B, XBP C, Y CP D y ZDP A son cı́clicos. Luego, por el teorema
de Ptolomeo, se obtiene que:
√ √
WP = PA + PB , XP 3 = P B + P C , Y P = PC + PD , ZP 3 = P D + P A

Por otro lado,

 W P Y =  W P B + 60◦ +  CP Y =  W AB + 60◦ +  CDY = 180◦


Luego W , P , Y están alineados y, de forma análoga, Z, P , X están alineados. Luego:

W Y = W P + P Y = P A + P B + P C + P D = AC + BD
1 1
XZ = XP + P Z = √ (P B + P C + P D + P A) = √ (AC + BD)
3 3

Por la desigualdad, triangular:

W Y ≤ W E + EG + GY , XZ ≤ XF + F H + HZ

Luego:
√ √
3 3 1 BC AD
AC + BD ≤ AB + EG + DC , √ (AC + BD) ≤ √ + F H + √
2 2 3 2 3 2 3

Por lo tanto, sumando,


√ √
3 3
AC + BD ≤ AB 2 + EG + DC 2
√ √
3

3
3(AC + BD) ≤ BC
+ 3F H + AD
2 2
√ √
1 + 3 (AC + BD) ≤ EG + 3F H + s 3

o sea,  √  √
EG + 3F H ≥ 1 + 3 d − s 3

Luego, si k = 1 + 3, entonces EG + 3F H ≥ kd + (1 − k)s.
La igualdad se dará si, y sólo si, por un lado, W , E, G, Y están alineados y, por otro lado, X,
F , H, Z también están alineados. Como W E es perpendicular a AB y GY es perpendicular
a DC, AB y DC deben ser paralelas y, de forma análoga, BC y AD también deben ser
paralelas, luego ABCD debe ser un paralelogramo. Además, la recta EG es perpendicular
a DC, lo que implica que ABCD es un rectángulo y se comprueba fácilmente que si ABCD
es un rectángulo, entonces se da la igualdad. Luego, la igualdad se da si, y sólo si, ABCD
es un rectángulo.
Ahora, sea un número real positivo l tal que EG+3HF ≥ ld +(1 −l)s. Entonces, si ABCD
es un rectángulo,
kd + (1 − k)s ≥ ld + (1 − l)s
o sea
k(d − s) ≥ l(d − s)
Pero la desigualdad triangular
√ implica que d > s, lo que implica que k ≥ l. Luego el número
real buscado es k = 1 + 3 y la igualdad se da si, y sólo si, ABCD es un rectángulo.

• Problema 4
Sean a, b, c tres números reales positivos. Demuestra que

a + b + 3c a + 3b + c 3a + b + c 15
+ + ≥
3a + 3b + 2c 3a + 2b + 3c 2a + 3b + 3c 8
Solución:
Haciendo a = x1 , b = x2 , c = x3 y llamando s = x1 + x2 + x3 , resulta que el lado izquierdo
de la desigualdad se escribe como

s + 2x1 s + 2x2 s + 2x3


S= + +
3s − x1 3s − x2 3s − x3

Por otro lado,


3
 3

s + 2xk 7s
S+6 = +2 =
3s − xk 3s − xk
k=1 k=1

con lo que
 3
7s 7s 7s 1
S+6 = + + = 7s
3s − x1 3s − x2 3s − x3 3s − xk
k=1
3 1
3
Dado que k=1 (3s − xk ) = 8s, entonces s = 8 k=1 (3s − xk ), y

3 3
7  1 63
S+6 = (3s − xk ) ≥
8 3s − xk 8
k=1 k=1

de donde resulta que

s + 2x1 s + 2x2 s + 2x3 63 15


S= + + ≥ −6=
3s − x1 3s − x2 3s − x3 8 8

La igualdad tiene lugar cuando x1 = x2 = x3 , es decir, cuando a = b = c.

• Problema 5
Sea P un punto cualquiera de la bisectriz del ángulo A en el triángulo ABC, y sean A , B  ,
C  puntos respectivos de las rectas BC, CA, AB, tales que P A es perpendicular a BC,
P B  es perpendicular a CA y P C  es perpendicular a AB. Demuestra que P A y B  C  se
cortan sobre la mediana AM , siendo M el punto medio de BC.

Solución:
Sea E el punto de intersección de P A y B  C  . Si P se mueve sobre la bisectriz AI (I es el
incentro), la figura P B  C  E es homotética de sı́ misma con respecto al punto A. Luego E
describe una recta que pasa por A. La bisectriz AI corta a la circunferencia circunscrita a
ABC en F , que se proyecta en el punto medio Am de BC; si P = F , la recta B  C  es la
recta de Simson de F , luego el lugar geométrico de E es la mediana AAm .

• Problema 6
Sea p un número primo y A un subconjunto infinito de los números naturales. Sea fA (n) el
número de soluciones distintas de la ecuación x1 + x2 + · · · + xp = n, con x1 , x2 , . . . , xp ∈ A.
¿Existe algún número natural N tal que fA (n) sea constante para todo n > N ?

Solución:
Para demostrar el enunciado procederemos por contradicción. Supongamos que existe un
número N para el que se cumpla la propiedad anterior. Como el conjunto A es infinito,
tomemos a ∈ A mayor que N . Vamos a estudiar el valor de fA (pa) y fA (pa + 1). Por
hipótesis, se cumple que fA (pa) = fA (pa + 1).
Sea S = (s1 , s2 , . . . , sp ) solución de la ecuación

x1 + x 2 + · · · + x p = n , s 1 , s 2 , . . . , sp ∈ A

Entonces, cualquier permutación de los ı́ndices da lugar a una nueva solución de la ecuación
(posiblemente repetida si se permutan valores iguales). Diremos que una solución S =
(s1 , s2 , . . . , sp ) es asociada a una solución S  = s1 , s2 , . . . , sp si la primera se obtiene a
partir de la segunda mediante permutación de ı́ndices. Sea S = (s1 , s2 , . . . , sp ) una solución
del problema y sea Q = (q1 , q2 , . . . , qp ) una solución asociada a S con la propiedad que
q1 = q2 = · · · = qr1 = qr1 +1 = qr1 +2 = · · · qr1 +r2 , y ası́ de manera sucesiva hasta llegar a
qr1 +r2 +···+rk = qp . En otras palabras, Q se obtiene a partir de S agrupando los valores si
que son iguales. En particular, r1 + r2 + · · · + rk = p. Con esta notación, el número de
p!
soluciones asociadas a S (contando también S) es igual a .
r1 !r2 ! · · · rk !
Obsérvese que si todos los ri son estrictamente menores que p, entonces dicha expresión es
congruente con 0 módulo p, puesto que el cociente de factoriales es un número natural y en
el denominador no hay ningún término múltiplo de p.
Ya tenemos todas las herramientas que necesitábamos. Volviendo al problema original,
observar que (a, a, . . . , a) es solución de x1 + x2 + · · · + xp = pa.
 
p
Por lo tanto fA (pa) ≡ 1 (mód p): la solución (a, a, . . . , a) no se asocia a ninguna otra,
 
p
mientras que cualquier otra solución de la ecuación x1 + x2 + · · · + xp = pa tiene un número
múltiplo de p de asociadas. Por otro lado no existen soluciones de x1 +x2 +· · ·+xp = pa +1
1
con todas las xi iguales (su valor tendrı́a que ser a + ), con lo que según lo anterior
p
fA (pa + 1) ≡ 0 (mód p) y llegamos a una contradicción.
Fase Nacional de la XLV Olimpiada Matemática Española
Sant Feliu de Guixols (Girona), 27 de marzo de 2009

PRIMERA SESIÓN
SOLUCIONES

PROBLEMA 1.- Halla todas las sucesiones finitas de n números naturales


consecutivos a1 , a 2 ,..., a n , con n ≥ 3, tales que a1 + a2 + ... + an = 2009.

Primera solución:

Supongamos que N es la suma de n números naturales consecutivos


empezando por k + 1. Entonces
N = (k + 1) + (k + 2) + ... + (k + n) =
[1 + 2 + ... + k + (k + 1) + ... + (k + n) ] − [1 + 2 + ... + k ] =
(k + n)(k + n + 1) k (k + 1) n(2k + n + 1)
− = .
2 2 2

Teniendo en cuenta que 2009 = 1 × 2009 = 7 × 287 = 49 × 41 se tienen los


siguientes casos:

(1) Si n = 7 y
(2k + n + 1) = 287
resulta k = 283 con lo que
2
2009 = 284 + 285 + 286 + 287 + 288 + 289 + 290.

n
(2) Si = 7 y 2k + n + 1 = 287, resulta k = 136 con lo que
2
2009 = 137 + 138 + ... + 150.

(3) Si n = 41 y
(2k + n + 1) = 49 resulta k = 28 con lo que
2
2009 = 29 + 30 + 31 + ... + 69.

(4) Si n = 49 y
(2k + n + 1) = 41 resulta k = 16 con lo que
2
2009 = 17 + 18 + 19 + ... + 65.

(5) Los otros casos dan valores de k que no verifican el enunciado.


Segunda solución:

n (n − 1)
Claramente, es a n = a1 + n − 1, de donde 2009 = n a1 + . Entonces
2
n (n − 1) (n − 1) 2
2009 ≥ > , luego n − 1 < 4018 , y al ser
2 2
64 2 = 4096 > 4018 > 3969 = 63 2 , resulta que 4018 < 64, con lo que n < 65.

Supongamos en primer lugar que n es impar. Entonces, obviamente n divide a


2009 = 7 2 × 41 y n puede tomar los valores 7, 41 ó 49, pues cualquier otro
divisor impar de 2009 es mayor o igual que 7 × 41 = 287. Se obtiene entonces
2009 n − 1
a1 = − , con valores respectivos 284, 29 y 17, es decir, se obtienen
n 2
las tres sucesiones
284, 285,..., 290;
29, 30,..., 69;
17,18,..., 65.
No hay otras sucesiones con un número impar de términos.
n
Supongamos finalmente que n es par. Entonces divide a 2009, con lo que
2
n = 14, ya que cualquier otro valor de n ha de ser mayor o igual que
2 × 41 = 82 > 65, lo que no es posible. Entonces a1 = 137, y la única sucesión
con número par de términos es 137,138,...,150.
Fase Nacional de la XLV Olimpiada Matemática Española
Sant Feliu de Guixols (Girona), 27 de marzo de 2009

PRIMERA SESIÓN
SOLUCIONES

PROBLEMA 2 .- Sean ABC un triángulo acutángulo, I el centro del círculo inscrito


en el triángulo ABC , r su radio y R el radio del círculo circunscrito al triángulo ABC.
Se traza la altura AD = ha , con D perteneciente al lado BC. Demuestra que
DI 2 = ( 2 R − ha ) ( ha − 2r ).
Primera solución:

Sean E y M las proyecciones ortogonales de I sobre BC y AD ,


respectivamente.
r S S
Se tiene: AI = ; r= ⇒ AI = (1) donde, evidentemente,
A p A
sen p ⋅ sen
2 2
S es el área del triángulo ABC y p es su semiperímetro.
bc ⋅ senA A A
Por otra parte, S = = bc ⋅ sen ⋅ cos , así que (1) se puede escribir
2 2 2
A
bc ⋅ cos
como AI = 2 , y ya que cos 2 A = p ( p − a ) , obtenemos
p 2 bc
bc ( p − a )
AI 2 = (2).
p
S 2S
Teniendo en cuenta que bc = 2 R ⋅ ha , p = , a = , la expresión (2) se
r ha
⎛ 2S ⎞
⎜ ⎟
⎜ ha ⎟ ⎛ 2r ⎞
escribe como AI = 2 R ⋅ ha 1 −
2
= 2 R ⋅ ha ⎜⎜1 − ⎟⎟ = 2 R (ha − 2r )
⎜ S ⎟ ⎝ ha ⎠
⎜ ⎟
⎝ r ⎠
(3).Como el cuadrilátero IEDM es un rectángulo, MD = IE = r. Aplicando el
teorema de Pitágoras generalizado a ADI tenemos
DI 2 = ha2 + AI 2 − 2ha ⋅ AM ⇔ DI 2 = ha2 + AI 2 − 2ha ( ha − MD ),
y teniendo en cuenta los resultados anteriormente obtenidos resulta, finalmente,
DI 2 = ha2 + 2 R ( ha − 2r ) − 2ha ( ha − r ) = (2 R − ha ) ( ha − 2r ), c.q.d.
Segunda solución:

Sean a, b, c las longitudes de los lados BC, CA y AB respectivamente, y sea


T el punto donde la circunferencia inscrita es tangente al lado BC. Por el
teorema de Pitágoras, al ser IT // AD ⊥ DT , se tiene que
DI = r + DT = r + AT − AD y la igualdad a demostrar es equivalente a
2 2 2 2 2 2

AT 2 + 4 Rr + r 2 = 2 ( R + r ) ha .
Ahora bien, llamando S al área de ABC , es conocido que
abc r (a + b + c) 1 ah
S= = = (a + b + c) (b + c − a) (c + a − b) (a + b − c) = a .
4R 2 4 2
2abc (b + c − a ) ( c + a − b ) ( a + b − c)
De aquí se deduce que 4 Rr = , r2 = ,
a+b+c a+b+c
(b + c − a ) (c + a − b) (a + b − c)
2 R ha = bc y 2 R ha = .
2a
2ab + 2bc + 2ca − a 2 − b 2 − c 2
Obsérvese que 4 Rr + r 2 = . Ahora bien, se
4
c+a −b a+b−c
sabe que BT = , CT = .
2 2
Por el teorema de Stewart, tenemos que
BT ⋅ AC 2 + CT ⋅ AB 2
AT = 2
− BT ⋅ CT =
BC

3b 2 + 3c 2 − a 2 − 2bc (b − c) 2 (b + c)

4 2a
con lo que
b 2 + c 2 − a 2 + ab + ca (b − c) 2 (b + c)
AT 2 + 4 Rr + r 2 = = ,
2 2a
e identificando términos se comprueba que esto coincide con el valor de
2 ( R + r ) ha
y de este modo la igualdad requerida queda demostrada.

Tercera solución:

Nótese en primer lugar que


B C
2r sen sen
2 S − 2ar (b + c − a ) r 2 2 = r2 IA 2
ha − 2r = = = = ,
a a A 2 A 2R
sen 2 R sen
2 2
A B C
donde S es el área de ABC y se ha utilizado que r = 4 R sen sen sen .
2 2 2
Sea ahora el punto P en el que la paralela a BC por I corta a la altura AD.
Claramente,
DI 2 = r 2 + IA 2 − ( ha − r ) 2 = IA 2 − ha ( ha − 2r ) = ( 2 R − ha ) ( ha − 2r ).
Fase Nacional de la XLV Olimpiada Matemática Española
Sant Feliu de Guixols (Girona), 27 de marzo de 2009

PRIMERA SESIÓN
SOLUCIONES

PROBLEMA 3.- Se pintan de rojo algunas de las aristas de un poliedro regular. Se dice
que una coloración de este tipo es buena, si para cada vértice del poliedro, existe una
arista que concurre en dicho vértice y no está pintada de rojo. Por otra parte, se dice que
una coloración donde se pintan de rojo algunas de las aristas de un poliedro regular es
completamente buena, si además de ser buena, ninguna cara del poliedro tiene todas sus
aristas pintadas de rojo. ¿Para qué poliedros regulares es igual el número máximo de
aristas que se pueden pintar en una coloración buena y en una completamente buena?
Justifica la respuesta.

Solución:
Claramente, las coloraciones completamente buenas son un subconjunto de las
coloraciones buenas, con lo que si el máximo número de aristas que se pueden
pintar de rojo para obtener una coloración buena se puede alcanzar con una
coloración completamente buena, la pregunta del enunciado tiene respuesta
afirmativa.

NOTA: En cada caso véase el recuadro de las figuras al final de la solución.

En el caso de un tetraedro, existen 6 aristas, tales que en cada vértice confluyen


3 de ellas. El número máximo de aristas pintadas de rojo en una coloración
2
buena sería por lo tanto × 6 = 4, pues en caso contrario existiría algún vértice
3
2
donde más de de las aristas estuvieran pintadas de rojo, es decir, todas las
3
aristas estarían pintadas de rojo. La figura muestra una coloración
completamente buena de un tetraedro con 4 aristas rojas (el tetraedro ha sido
deformado para poder ser dibujado en el plano).

De igual forma, en el cubo existen 12 aristas, tales que en cada vértice


confluyen 3 de ellas. El número máximo de aristas pintadas de rojo en una
2
coloración buena sería, por lo tanto, × 12 = 8. La figura muestra una
3
coloración completamente buena con 8 aristas rojas.

Finalmente, en el dodecaedro existen 30 aristas, tales que en cada vértice


confluyen 3 de ellas. El número máximo de aristas pintadas de rojo en una
2
coloración buena sería, por lo tanto, × 30 = 20. La figura muestra una
3
coloración completamente buena de un dodecaedro con 12 aristas rojas.
De lo anterior se deduce que para el tetraedro, el cubo y el dodecaedro, el
número máximo de aristas rojas en una coloración buena se alcanza con una
coloración completamente buena.
En el octaedro existen 12 aristas, tales que en cada vértice confluyen 4 de
ellas. El número máximo de aristas pintadas de rojo en una coloración buena es
3
por lo tanto × 12 = 9. La figura muestra una coloración buena de un octaedro
4
con 9 aristas rojas. Ahora bien, como cada arista pertenece a dos caras,
sumando para las 8 caras el número de aristas pintadas de rojo en dicha cara,
obtenemos 18 = 2 × 8 + 2, con lo que hay, al menos, dos caras con 3 aristas
pintadas de rojo y una coloración buena con el número máximo de 9 aristas
pintadas de rojo nunca puede ser completamente buena.
Finalmente, en el icosaedro existen 30 aristas, tales que en cada vértice
confluyen 5 de ellas. El número máximo de aristas pintadas de rojo en una
4
coloración buena es por lo tanto × 30 = 24. La figura muestra una
5
coloración buena de un icosaedro con 24 aristas rojas. Ahora bien, eso quiere
decir, que sumando el número de aristas rojas de cada cara para todas las caras,
obtenemos 48 = 2 × 20 + 8, es decir, existen, al menos, 8 caras con todas sus
aristas rojas, y una coloración buena con el número máximo de 24 aristas
pintadas de rojo nunca puede ser completamente buena.

Por lo tanto, los poliedros regulares que tienen la propiedad descrita en el


enunciado son el tetraedro, el cubo y el dodecaedro (es decir, los poliedros
tales que en cada vértice confluyen exactamente 3 aristas) y los que no la
tienen son el octaedro e icosaedro (en cuyos vértices confluyen más de 3
aristas).
FIGURAS

FIGURAS
Fase Nacional de la XLV Olimpiada Matemática Española
Sant Feliu de Guixols (Girona), 28 de marzo de 2009

SEGUNDA SESIÓN
SOLUCIONES

PROBLEMA 4. - Determina justificadamente todos los pares de números enteros


( x, y ) que verifican la ecuación x 2 − y 4 = 2009.

Solución:

Dada una solución ( x, y ) cualquiera, es claro que también son soluciones


( x,− y ), (− x, y ) y (− x,− y ), con lo que se puede asumir sin pérdida de
generalidad que x, y ≥ 0. Supongamos entonces que es así. Es claro
que ( x − y 2 ) ( x + y 2 ) = 7 2 ⋅ 41.
Si x − y 2 y x + y 2 no son primos entre sí, su máximo común divisor al
cuadrado divide a 2009 = 7 2 ⋅ 41, luego es 7 y divide a
( x + y 2 ) + ( x − y 2 ) = 2 x y a ( x + y 2 ) − ( x − y 2 ) = 2 y 2 , con lo que existen
enteros no negativos u y v tales que x = 7u , y = 7v y
(u + 7v )(u − 7v ) = 41. Como ambos factores han de ser enteros, se tiene que
2 2

10
u + 7v 2 = 41 y u − 7v 2 = 1, con lo que u = 21 y v 2 = . No existen pues
7
soluciones enteras en este caso.

Si x − y 2 y x + y 2 son primos entre sí, un posible caso es que x − y 2 = 1 y


x + y 2 = 2009, con lo que y 2 = 1004, absurdo pues 312 = 961 < 1004 < 322.
Resta entonces tan sólo el caso en que x − y 2 = 41 y x + y 2 = 49, que produce
x = 45, y 2 = 4, con lo que la única solución con enteros no negativos es
x = 45 e y = 2, y las únicas soluciones en enteros son ( x, y ) = (± 45, ± 2).
Fase Nacional de la XLV Olimpiada Matemática Española
Sant Feliu de Guixols (Girona), 28 de marzo de 2009

SEGUNDA SESIÓN
SOLUCIONES

PROBLEMA 5. - Sean a, b, c números reales positivos tales que abc = 1. Prueba la


desigualdad siguiente

2 2 2
⎛ a ⎞ ⎛ b ⎞ ⎛ c ⎞ 3
⎜ ⎟ +⎜ ⎟ +⎜ ⎟ ≥
⎝ 1 + ab ⎠ ⎝ 1 + bc ⎠ ⎝ 1 + ca ⎠ 4

Solución:

2 2 2
⎛ a ⎞ ⎛ ca ⎞ ⎛ ca ⎞
Como abc = 1, entonces ⎜ ⎟ =⎜ ⎟ =⎜ ⎟ . Análogamente se
⎝ 1 + ab ⎠ ⎝ abc + c ⎠ ⎝1+ c ⎠
2 2 2 2
⎛ b ⎞ ⎛ ab ⎞ ⎛ c ⎞ ⎛ bc ⎞
obtienen ⎜ ⎟ =⎜ ⎟ y ⎜ ⎟ =⎜ ⎟ . Por tanto la
⎝ 1 + bc ⎠ ⎝1+ a ⎠ ⎝ 1 + ca ⎠ ⎝1+ b ⎠
desigualdad requerida se convierte en
2 2 2
⎛ ab ⎞ ⎛ bc ⎞ ⎛ ca ⎞ 3
⎜ ⎟ +⎜ ⎟ +⎜ ⎟ ≥ , equivalente a
⎝1+ a ⎠ ⎝1+ b ⎠ ⎝1+ c ⎠ 4
1 ⎡⎛ ab ⎞ ⎛ bc ⎞ ⎛ ca ⎞ ⎤ 1
2 2 2

⎢⎜ ⎟ +⎜ ⎟ +⎜ ⎟ ⎥≥ .
3 ⎣⎢⎝ 1 + a ⎠ ⎝ 1 + b ⎠ ⎝ 1 + c ⎠ ⎦⎥ 2
Usando ahora la desigualdad entre las medias aritmética y cuadrática, se
obtiene

1 ⎡⎛ ab ⎞ ⎛ bc ⎞ ⎛ ca ⎞ ⎤ 1 ⎡ ⎛ ab ⎞ ⎛ bc ⎞ ⎛ ca ⎞ ⎤
2 2 2

⎢⎜ ⎟ +⎜ ⎟ +⎜ ⎟ ⎥≥ ⎜ ⎟+⎜ ⎟+⎜ ⎟ .
3 ⎣⎢⎝ 1 + a ⎠ ⎝ 1 + b ⎠ ⎝ 1 + c ⎠ ⎦⎥ 3 ⎢⎣ ⎝ 1 + a ⎠ ⎝ 1 + b ⎠ ⎝ 1 + c ⎠ ⎥⎦
ab bc ca 3
Así es suficiente demostrar que + + ≥ o equivalentemente
1+ a 1+ b 1+ c 2
abc abc abc 3
+ + ≥ , que a su vez equivale a que
c(1 + a) a(1 + b) b(1 + c) 2

1 1 1 3
+ + ≥ .
c(1 + a) a (1 + b) b(1 + c) 2
x y z
Poniendo a= ,b= y c= en la última desigualdad resulta
y z x
−1 −1 −1
⎛ x x⎞ ⎛ y y⎞ ⎛z z⎞ 3 1 1
⎜⎜ + ⎟⎟ + ⎜ + ⎟ + ⎜⎜ + ⎟⎟ ≥ . Sustituyendo ahora α = , β = y
⎝ y z⎠ ⎝ z x⎠ ⎝x y⎠ 2 x y
1 α β γ 3
γ = , se llega a la desigualdad de Nessbit + + ≥ .
z β +γ γ +α α + β 2
La igualdad se alcanza si y sólo si a = b = c = 1.
Fase Nacional de la XLV Olimpiada Matemática Española
Sant Feliu de Guixols (Girona), 28 de marzo de 2009

SEGUNDA SESIÓN
SOLUCIONES

PROBLEMA 6. - En el interior de una circunferencia de centro O y radio r , se toman


dos puntos A y B, simétricos respecto de O. Se considera un punto variable P sobre
esta circunferencia y se traza la cuerda PP´, perpendicular a AP. Sea C el punto
simétrico de B respecto de PP´. Halla el lugar geométrico del punto Q, intersección de
PP´ con AC , al variar P sobre la circunferencia.

Primera solución:

Establezcamos primero que AC es constante.


Método 1.
Se obtiene C a partir de A aplicando un giro de 180º con centro en O
seguido de la simetría de eje PP´.
P Descomponiendo el giro en producto de dos
C
M simetrías de ejes perpendiculares e1 paralelo a AP y
Q
e2 perpendicular a AP, resulta que el triángulo
P' e2
AA´C es rectángulo en A´ y además:
A´C = 2 OM ; AA´= 2 MP, de donde
B O A
AC = 4 OM + 4 MP = 4 OP = 4 r ; es
2 2 2 2 2
decir
e1 AC = 2 r , con independencia de la posición de P.
A'

C Método 2
P

Prolongamos PA hasta que corte de nuevo a la


P' circunferencia en P´´. Se tiene CP´= P´B = AP´´.
Además P´B es paralelo a PP´´; luego el segmento
CA es la imagen del segmento P´P´´ mediante la
B O A
traslación de vector P´´A P y como ∠P´PP´´ es
recto y P´P´´ es un diámetro, resulta
P '' AC = P´P´´= 2 r.
Finalmente, al ser PP ´ la mediatriz de BC ,
QC = QB; se deduce entonces que QB + QA = QC + QA = AC = 2 r y Q
describe la elipse de focos A y B y constante 2 r. La recta PP´ es la tangente
en Q a la elipse.

Segunda solución:

Tomamos r = 1 y unos ejes de coordenadas en los que la ecuación de la


circunferencia es x 2 + y 2 = 1, y las coordenadas de A (a,0), B (− a,0), con
0 < a < 1.
En vez de empezar por P, sea P´ ( x 0 , y 0 ) con la condición x02 + y 02 = 1. Por
las condiciones del problema, P´ es el punto medio de BC; llamando ( x1 , y1 )
⎧ x1 = 2 x 0 + a
a las coordenadas de C , se tiene ⎨ . Entonces la ecuación de la
⎩ y1 = 2 y 0
y
recta CA es y = 0 ( x − a ), es decir x0 y − y 0 x + y 0 a = 0.
x0
y0 x +a
Las pendientes de P´B y de P´P son respectivamente y − 0 . Por
x0 + a y0
tanto la ecuación de P´P es y 0 y + x ( x 0 + a) − ax0 + 1 = 0.
Las coordenadas del punto Q, intersección de AC y P´P son:

⎛ x 0 + a y 0 (1 − a 2 )
Q ⎜⎜ , ⎟. Denotando por x, y a las coordenadas de Q y

⎝ 1 + x0 a 1 + x0 a⎠
a−x −y
despejando los valores de x0 e y 0 se obtiene x0 = , y0 = .
ax − 1 ax − 1
Imponiendo ahora la condición x02 + y 02 = 1, se llega a
(a − x) 2 y2
+ =1
(ax − 1) 2 (ax − 1) 2
y2
y mediante operaciones se transforma en la ecuación x 2 + = 1, que es la
1− a2
ecuación de una elipse.

Tercera solución:

Demostraremos en primer lugar que, dados dos puntos A, B del plano, el


conjunto de los puntos P (del mismo plano) tales que PA 2 + PB 2 es constante
y mayor que AB 2 , es una circunferencia de centro el punto medio de AB y
que tiene a los puntos A y B en su interior.
En efecto, supongamos A = (d ,0), B = (−d ,0) y sea P = ( x, y ) cualquier
punto. Se tiene entonces
PA 2 + PB 2 = 2 x 2 + 2 y 2 + 2 d 2 .
k
Así que si PA 2 + PB 2 = k ≥ 4 d 2 , se tiene que x 2 + y 2 = − d 2 ≥ d 2 .
2
Sea entonces ahora R el punto donde BC corta a PP´ (que es perpendicular a
BC ). Este punto R satisface
PR 2 = BP 2 − BR 2 = AR 2 − AP 2 ,
luego R está en la circunferencia y es distinto de P , con lo que R = P´.
Ahora bien, se tiene
AP 2 + BP´2 −( AP − BP´) 2 k − PP´2 −( AP − BP´)2 k − AB 2
AP ⋅ BP´= = = ,
2 2 2
donde k = AP 2 + BP 2 = AP´2 + BP´2 .
Además, la potencia de A respecto de la circunferencia es
k k − AB 2
r2 − d 2 = − 2d 2 = ,
2 2
con lo que el segundo punto S en el que AP corta a la circunferencia es tal
que AS = BP´= CP´.
Como CP´⊥ PP´⊥ AP, se tiene que AS es paralelo a CP´ y ASP´C es un
paralelogramo.
Finalmente,
PP´2 + PS 2 = AP´2 + AS 2 + 2 ⋅ AS ⋅ AP = AP´2 + BP´2 + k − AB 2 = 2 k − 4 d 2 = 4 r 2 ,

es decir, P ' S = AC = 2r. Como AQ + BQ = AC = 2 r , el lugar de Q es la


elipse interiormente tangente a la circunferencia dada, con A y B como
focos.
XLIV Olimpiada Matemática Española
Fase nacional 2008 (Valencia)
PRIMERA SESIÓN (28 de marzo)

1.- Halla dos enteros positivos a y b conociendo su suma y su mínimo


común múltiplo. Aplícalo en el caso de que la suma sea 3972 y el mínimo
común múltiplo 985928.

SOLUCIÓN:
Sea p un número primo que divide a la suma a + b y a su mínimo común múltiplo
[ a,b ]. Como p [ a, b ] al menos divide a uno de los dos enteros a ó b. Si p a, al
dividir p a la suma a + b, también p b. (Obviamente el mismo razonamiento vale si
hubiéramos supuesto que p b). Por tanto podemos dividir los dos números a y b por
p y también su mínimo común múltiplo [ a,b ], para obtener dos enteros a1 y b1
a
tales que a1 = , b1 =
b
y [ a1 , b1 ]=
[ a, b ] . Sea el máximo común divisor de a y b,
p p p
d = ( a,b ). Repitiendo el proceso anterior llegaremos a obtener dos enteros A y B
tales que a = dA, b = dB y ( A, B ) = 1. Entonces [ A, B ] = AB. Ahora es fácil
⎧ a+b
⎪⎪ A + B = d
determinar A y B a partir del sistema de ecuaciones ⎨ . Es decir A y
⎪ AB = [ a, b ]
⎪⎩ d
B son las raíces de la ecuación de segundo grado d t − (a + b) t + [ a, b ]= 0.
2

Observamos que el discriminante de esta ecuación es no negativo. En efecto:


∆ = (a + b) 2 − 4 d [ a, b ] = (a + b) 2 − 4ab = (a − b) 2 ≥ 0.

Si a y b son distintos, la ecuación anterior tiene por soluciones los dos enteros
a b
positivos A = y B= .
d d
En particular cuando a + b = 3972 y [ a, b ] = 985928, tenemos que
d = ( 3972, 985928 ) = 4. Por tanto a = 4 A y b = 4 B siendo A y B las raíces de la
ecuación 4 t 2 − 3972 t + 985928 = 0.
Es decir A = 491 y B = 502 y los números buscados son a = 1964 (año de la primera
OME) y b = 2008 (año de la actual edición de la OME).

1
XLIV Olimpiada Matemática Española
Fase nacional 2008 (Valencia)
PRIMERA SESIÓN (28 de marzo)

2.- Prueba que para cualesquiera números reales a, b tales que 0 < a, b < 1, se
cumple la desigualdad siguiente:

ab 2 + a 2 b + (1 − a)(1 − b) 2 + (1 − a) 2 (1 − b) < 2 .

SOLUCIÓN:
a+b
Se verifica que x < 3 x para todo x ∈ (0,1). Teniendo en cuenta que 0 < < 1,
2
utilizando la desigualdad anterior y aplicando la desigualdad entre las medias aritmética
y geométrica, se tiene:
⎛a+b⎞
a+b+⎜ ⎟
⎛a+b⎞ 3 ⎛a+b⎞ ⎝ 2 ⎠ a+b
ab ⎜ ⎟ < ab ⎜ ⎟≤ =
⎝ 2 ⎠ ⎝ 2 ⎠ 3 2
y

⎛ a+b⎞ 3 ⎛ a+b⎞
(1 − a) (1 − b) ⎜1 − ⎟ < (1 − a ) (1 − b) ⎜1 − ⎟≤
⎝ 2 ⎠ ⎝ 2 ⎠
a+b
1− a +1− b +1−
≤ 2 =1 − a + b .
3 2
Sumando las expresiones anteriores resulta

⎛a+b⎞ ⎛ a+b⎞
ab ⎜ ⎟ + (1 − a ) (1 − b) ⎜1 − ⎟ < 1,
⎝ 2 ⎠ ⎝ 2 ⎠

o equivalentemente

1
2
( ab 2 + a 2 b + )
(1 − a)(1 − b) 2 + (1 − a) 2 (1 − b) < 1,

de donde se obtiene inmediatamente la desigualdad del enunciado.

2
XLIV Olimpiada Matemática Española
Fase nacional 2008 (Valencia)
PRIMERA SESIÓN (28 de marzo)

3.- Sea p ≥ 3 un número primo. Se divide cada lado de un triángulo en p


partes iguales y se une cada uno de los puntos de división con el vértice
opuesto. Calcula el número máximo de regiones, disjuntas dos a dos, en
que queda dividido el triángulo.

SOLUCIÓN:
En primer lugar veremos que tres de estos segmentos (cevianas) no pueden ser
concurrentes. Sea el triángulo ABC y X , Y , Z puntos de las divisiones interiores de los
lados BC , AC , AB respectivamente. Si AX , BY y CZ fueran concurrentes aplicando
el teorema de Ceva tendríamos
AZ BX CY
⋅ ⋅ = 1.
ZB XC YA
Por otro lado, por la forma en que hemos construido los puntos de división, existen
enteros positivos k , l , m∈{1,2,..., p − 1 }, tales que
AZ k BX l CY m
= , = , = .
ZB p − k XC p − l YA p − m

Sustituyendo en la expresión anterior

k l m = ( p − k ) ( p − l ) ( p − m),
o equivalentemente,
2 k l m + p m ( p − k − l ) = p ( p − k ) ( p − l ).
De aquí resulta que p divide al producto klm, que es imposible y nuestra afirmación
inicial queda probada.
Dibujando las cevianas desde el vértice A el triángulo ABC queda dividido en
p triángulos. Las cevianas trazadas desde B dividen cada uno de los p triángulos
anteriores en p partes disjuntas, teniendo en total p 2 regiones. Cada ceviana trazada
desde el vértice C aumenta el número de regiones en un número exactamente igual a su
número de intersecciones con las rectas que encuentra (incluido el lado AB ). Es decir,
2 ( p − 1) + 1 = 2 p − 1. Por tanto, el número máximo de regiones disjuntas dos a dos, en
que queda dividido el triángulo ABC es p 2 + ( p − 1)(2 p − 1) = 3 p 2 − 3 p + 1.

3
XLIV Olimpiada Matemática Española
Fase nacional 2008 (Valencia)
SEGUNDA SESIÓN (29 de marzo)

4.- Sean p y q dos números primos positivos diferentes. Prueba que


existen enteros positivos a y b, tales que la media aritmética de todos los
divisores positivos del número n = p a q b es un número entero.

SOLUCIÓN:
La suma de todos los divisores de n viene dada por la fórmula
(1 + p + p 2 + ... + p a ) (1 + q + q 2 + ... + q b ),
como se puede comprobar desarrollando los paréntesis. El número n tiene
(a + 1)(b + 1) divisores positivos y la media aritmética de todos ellos es
(1 + p + p 2 + ... + p a ) (1 + q + q 2 + ... + q b )
m= .
(a + 1) (b + 1)

Si p y q son ambos impares, tomando a = p y b = q, es fácil ver que m es un entero.


Efectivamente: cada factor 1 + p + p 2 + p 3 + ... + p p y 1 + q + q 2 + q 3 + ... + q q tiene un
número par de sumandos y por ejemplo, el primero se puede escribir como sigue
1 + p + p 2 + p 3 + ... + p p = (1 + p ) + p 2 (1 + p ) + ... + p p −1 (1 + p ) =
(1 + p ) (1 + p 2 + ... + p p −1 ).
Análogamente el segundo factor 1 + q + q 2 + q 3 + ... + q q = (1 + q) (1 + q 2 + ... + q q −1 ).
Entonces m = (1 + p 2 + p 4 + ... + p p −1 ) (1 + q 2 + q 4 + ... + q q −1 ), que es un entero
positivo.

Si p = 2 y q es impar, se eligen b = q y a + 1 = 1 + q 2 + q 4 + ... + q q −1 . Entonces


q −1
(1 + 2 + 2 2 + ... + 2 q + q +...+ q ) (1 + q + q 2 + q 3 + ... + q q )
2 4

m= q −1
= 1 + 2 + 2 2 + ... + 2 a , que es
(1 + q + q + ... + q ) (q + 1)
2 4

entero.

Para q = 2 y p impar, análogamente al caso anterior se eligen a = p y


b = p 2 + p 4 + ... + p p −1 y m es entero.

Alternativamente y de una manera casi directa, se obtiene una solución completa


observando que si p y q son primos impares se toman a = b = 1 y si p = 2 y q primo
q −1
impar, entonces se consideran a = y b = 1.
2

1
XLIV Olimpiada Matemática Española
Fase nacional 2008 (Valencia)
SEGUNDA SESIÓN (29 de marzo)

5.- Dada una circunferencia y en ella dos puntos fijos A, B, otro variable P
y una recta r; se trazan las rectas PA y PB que cortan a r en C y D
respectivamente. Determina dos puntos fijos de r , M y N, tales que el
producto CM ·DN sea constante al variar P.

SOLUCIÓN:
Trazamos las paralelas a r por A y B que cortan a la circunferencia en A´ y B´
respectivamente de modo que AA´BB´ es un trapecio isósceles.
Las intersecciones de AB´ y BA´ con r determinan los puntos M y N buscados.
En efecto, los triángulos AMC y DNB (sombreados en la figura) son semejantes ya
que tienen dos ángulos iguales:

A'
A
D
C
r N
M

B'

∠MAC = ∠B´BP = ∠NDB,


donde la primera igualdad es cierta por ser ángulos inscritos en el mismo arco y la
segunda por ser BB´ paralela a r.
∠AMC = ∠AB´B = ∠DNB,
con argumentos análogos a los anteriores.

Estableciendo la proporcionalidad de los lados resulta


AM ND
= ⇔ MC ⋅ ND = AM ⋅ BN ,
MC BN
cantidad que no depende de P.

2
Se observa que si la recta r pasa por el punto A, M = A = C , no se forma el triángulo
AMC. En este caso CM = 0 y el producto CM ⋅ DN = 0, es constante. Análogamente
este producto es cero si la recta r pasa por B o por los puntos A y B en cuyo caso
CM = DN = 0.

XLIV Olimpiada Matemática Española


Fase nacional 2008 (Valencia)
SEGUNDA SESIÓN (29 de marzo)

6.- A cada punto del plano se le asigna un solo color entre siete colores
distintos. ¿Existirá un trapecio inscriptible en una circunferencia cuyos
vértices tengan todos el mismo color?

SOLUCIÓN:
La idea inicial es considerar una circunferencia C de radio r y sobre ella bloques de 8
puntos A1 , A 2 ,..., A 8 igualmente espaciados; es decir que los arcos A i A i +1 i = 1,...,7
tengan igual longitud λ > 0 (que se elegirá convenientemente) para cada uno de los
bloques. Se elige un sentido dado (por ejemplo, el antihorario).
Se disponen entonces, en este sentido antihorario, 7 × 7 + 1 = 50 bloques de 7 + 1 = 8
puntos cada uno en la semicircunferencia superior de C , tales que dos bloques distintos
no se intersequen o solapen, para lo cual se toma λ suficientemente pequeño, por
πr
ejemplo 0 < λ < .
400
Se observa que al menos hay dos puntos del mismo color en cada bloque. Se eligen dos
de esos puntos y su color se le asocia al bloque. Y la distancia entre estos dos puntos
que es uno de los siete números d n = nλ , n ∈ {1,2,3,4,5,6,7 }, se le asigna también al
bloque. De este modo a cada uno de los 50 bloques se le hace corresponder el par
(color, distancia), indicado anteriormente. Como el número total de posibles pares es
49, por el principio el palomar, existirán dos bloques R y Q a los que se les asocia el
mismo par (color, distancia). Por tanto los cuatro puntos determinados por estos dos
bloques tienen el mismo color. Y como los dos puntos del bloque R distan igual que
los dos puntos del bloque Q, estando los cuatro puntos sobre la circunferencia
C , necesariamente, estos cuatro puntos son los vértices de un trapecio inscriptible.

NOTA: Este mismo razonamiento se podría hacer considerando un arco de circunferencia de radio r de
l
longitud l , 0 < l ≤ 2π r. En este caso se tomaría λ, 0 < λ < .
400
Y también se podría generalizar a un número de colores c (c ≥ 2) cualesquiera, considerando c + 1
puntos en vez de 7 + 1 y c + 1 bloques disjuntos de c + 1 puntos cada uno, en vez de 7 + 1 bloques
2 2

l
de 7 + 1 puntos cada uno. Y ahora λ debe cumplir que 0 < λ < .
(c + 1)(c 2 + 1)

También podría gustarte